SMLE mix Flashcards

1
Q

Acetaminophen heptotoxicity stages

A

Stage 1:
- 0.5-24 hours after ingestion.
- Patients may be asymptomatic or report anorexia, nausea or vomiting, and malaise.
- Physical examination may reveal pallor, diaphoresis, malaise, and fatigue.

Stage 2 :
- 18-72 h after ingestion
- Patients develop right upper quadrant abdominal pain, anorexia, nausea, and vomiting.
- Right upper quadrant tenderness may be present.
- Tachycardia and hypotension may indicate volume losses.
- Some patients may report decreased urinary output (oliguria)

Stage 3: Hepatic phase
- 72-96 h after ingestion
- Patients have continued nausea and vomiting, abdominal pain, and a tender hepatic edge
- Hepatic necrosis and dysfunction may manifest as jaundice, coagulopathy, hypoglycemia, and hepatic encephalopathy
- Acute kidney injury develops in some critically ill patients
- Death from multiorgan failure may occur.

Stage 4:Recovery phase
- 4 d to 3 wk after ingestion
- Patients who survive critical illness in phase 3 have complete resolution of symptoms and complete resolution of organ failure

How well did you know this?
1
Not at all
2
3
4
5
Perfectly
2
Q

What is the Minimum toxic doses of acetaminophen for a single ingestion, posing significant risk of severe hepatotoxicity ?

A

In :
- Adults: 7.5-10 g
- Children: 150 mg/kg; 200 mg/kg in healthy children aged 1-6 years

How well did you know this?
1
Not at all
2
3
4
5
Perfectly
3
Q

How to treat acetaminophen toxicity ?

A
  • ABCs
  • Gastric decontamination with Oral Activated charcoal (AC) if the patient has a stable mental and clinical status, patent airway, and presents to the emergency department within 1 hour of ingestion.
  • N-acetylcysteine (NAC) : 100% hepatoprotective when it is given within 8 hours after an acute acetaminophen ingestion or if pt is pregnant. (even if it exceeded 8 hrs you should still give it).
  • Psychosocial, psychological and/or psychiatric evaluation is indicated if the patient intent to do self harm
How well did you know this?
1
Not at all
2
3
4
5
Perfectly
4
Q

What are the criteria for liver transplantation in patients with acetaminophen toxicity ?

A
  • Metabolic acidosis, unresponsive to resuscitaton
  • Renal failure
  • Coagulopathy
  • Encephalopathy
How well did you know this?
1
Not at all
2
3
4
5
Perfectly
5
Q

Stepwise Pharmacological therapy in Asthma

A
  1. SABA
  2. low dose ICS
    Alternatives: Cromolyn/Leukotriene receptor antagonist (LTRA)/Theophylline.
  3. low dose ICS + LABA OR medium-dose ICS
    Alternatives: low-dose ICS + either an LTRA or theophylline.
  4. medium-dose ICS + LABA
    Alternatives: medium-dose ICS + either LTRA or theophylline.
  5. high-dose ICS + LABA
  6. High-dose ICS + LABA + oral corticosteroid.

*SABA: Albuterol (Ventolin)
*LABA: Salmeterol
* Symbicort: ICS + LABA
* ICS : Budesonide (Pulmicort)

How well did you know this?
1
Not at all
2
3
4
5
Perfectly
6
Q

What are the tests used to confirm the diagnosis of asthma ?

A
  • Spirometry
  • Peak airflow
  • FeNO tests (exhaled nitric oxide)
  • Provocation tests
How well did you know this?
1
Not at all
2
3
4
5
Perfectly
7
Q

At which age does colorectal cancer screening is recommended ?

A

50 years old

How well did you know this?
1
Not at all
2
3
4
5
Perfectly
8
Q

What are the screening options of colorectal cancer screening ?

A

Tests that detect adenomatous polyps and cancer:
- Flexible sigmoidoscopy every 5 years
- Colonoscopy every 10 years
- Double-contrast barium enema every 5 years
- Computed tomographic (CT) colonography every 5 years.

Tests that primarily detect cancer:
- Annual guaiac-based fecal occult blood test (FOBT) with high test sensitivity for cancer.

  • Annual fecal immunochemical test (FIT) with high test sensitivity for cancer.
  • Stool DNA test with high sensitivity for cancer, interval uncertain
How well did you know this?
1
Not at all
2
3
4
5
Perfectly
9
Q

What are the 4 typical features of tetralogy of Fallot ?

A
  1. Right ventricular (RV) outflow tract obstruction (RVOTO (infundibular stenosis).
    2.Ventricular septal defect (VSD).
    3.Aorta dextroposition (overriding aorta).
    4.Right ventricular hypertrophy.
How well did you know this?
1
Not at all
2
3
4
5
Perfectly
10
Q

What is the screening age of osteoporosis?

A

Women 65 years or older without previous known fractures or secondary causes of osteoporosis.

How well did you know this?
1
Not at all
2
3
4
5
Perfectly
11
Q

What are the Treatment Options of Neurobrucellosis ?

A

Antimicrobial drugs like Rifampicin, doxycycline, ceftriaxone, or co-trimoxazole for a duration of 3–6 months or until normalization of CSF.

How well did you know this?
1
Not at all
2
3
4
5
Perfectly
12
Q

What is the antibiotic of choice for Campylobacter infections?

A

Azithromycin therapy would be a primary antibiotic choice for Campylobacter infections, when indicated (see Medical Care), with a typical regimen of 500 mg/d for 3 days. If the patient is bacteremic, treatment can be extended to two weeks.
However, erythromycin is the classic antibiotic of choice. Its resistance remains low, [32] and it can be used in pregnant women and children.

How well did you know this?
1
Not at all
2
3
4
5
Perfectly
13
Q

In which conditions does schistocytes occur ?

A

Characteristic feature of : microangiopathic hemolytic anemia(MAHA).[5] The causes of MAHA:
can be:
- Disseminated intravascular coagulation
- Thrombotic thrombocytopenic purpura
- Hemolytic-uremic syndrome
- HELLP syndrome: hemolysis, elevated liver enzymes, low platelets (HELLP) syndrome.
- Malfunctioning cardiac valves

How well did you know this?
1
Not at all
2
3
4
5
Perfectly
14
Q

What is Disseminated intravascular coagulation (DIC)?

A

Systemic activation of blood coagulation, which results in generation and deposition of fibrin, leading to microvascular thrombi in various organs and contributing to multiple organ dysfunction syndrome (MODS).
Consumption of clotting factors and platelets in DIC can result in life-threatening hemorrhage.

How well did you know this?
1
Not at all
2
3
4
5
Perfectly
15
Q

How does the lab values usually be in DIC patients?

A
  • Prolonged coagulation times.
  • Thrombocytopenia.
  • High levels of fibrin degradation products (FDPs).
  • Elevated D-dimer levels.
  • Microangiopathic pathology (schistocytes) on peripheral smears.
How well did you know this?
1
Not at all
2
3
4
5
Perfectly
16
Q

How to appraoch Contrast Medium Reactions ?

A
  • ABCs
  • Vital signs
  • Immediately discontinue ICM administration.
  • Oxygen administration.
  • IV Epinephrine (Adrenaline)
  • H1 antihistamines, such as diphenhydramine, and H2-receptor blockers, such as cimetidine, do not have a major role in the treatment of respiratory reactions, but they may be administered after epinephrine.
  • Vital signs monitoring
  • If pt is hypotensice: intravenous iso-osmolar fluid (ie, normal saline, Ringer lactate solution) in large volumes. Vasopressors should be considered if hypotension is resistence.
    The most specifically effective vasopressor is dopamine; at infusion rates of 2-10 mcg/kg/min.
How well did you know this?
1
Not at all
2
3
4
5
Perfectly
17
Q

Name the muscles of the eye, thier actions and thier nerve supply.

A
  • Lateral rectus - Moves eye laterally - Abducens VI
  • Medial rectus - Moves eye medially - Oculomotor III
  • Superior rectus- Elevate eye + medially - Oculomotor III
  • Inferior rectus- depresses eye + medially + Oculomotor III
  • Inferior oblique - elevates eye+ laterally - Oculomotor III
  • Superior oblique - depressess eye + laterally - Trochlear IV.
How well did you know this?
1
Not at all
2
3
4
5
Perfectly
18
Q

What is the management of ascities ?

A

Sodium restriction and diuretic therapy constitute the standard medical management for ascites:

  • Start with spironolactone at 100 mg/d (aldosterone antagonists)
  • loop diuretics may be necessary in some cases to increase the natriuretic effect.
  • If no response occurs after 4-5 days, the dosage may be increased stepwise up to spironolactone at 400 mg/d plus furosemide at 160 mg/d.
  • Therapeutic paracentesis
  • Supplementing 5 g of albumin per each liter over 5 L of ascitic fluid removed decreases complications of paracentesis, such as electrolyte imbalances and increases in serum creatinine levels secondary to large shifts of intravascular volume.
  • The transjugular intrahepatic portosystemic shunt (TIPS) is an interventional radiologic technique that reduces portal pressure and may be the most effective treatment for patients with diuretic-resistant ascites.
How well did you know this?
1
Not at all
2
3
4
5
Perfectly
19
Q

What is Lewy Body Dementia?
Signs and symptoms ?
management ?

A

It is a progressive, degenerative dementia of unknown etiology. Affected patients generally present with dementia preceding motor signs, particularly with visual hallucinations and episodes of reduced responsiveness..

Signs and symptoms:
- varying levels of alertness and attention.
- Visual hallucinations
- Parkinsonian motor features
- Anterograde memory loss
- Nonvisual hallucinations
- Delusions
- Unexplained syncope
- Rapid eye movement sleep disorder
- Neuroleptic sensitivity

Management:
- Cholinesterase inhibitors (eg, donepezil, rivastigmine, rivastigmine patch, galantamine)
- 2nd-generation antipsychotics (eg, clozapine, quetiapine, aripiprazole)
- Antidepressants (eg, venlafaxine, paroxetine, sertraline, fluoxetine)
- Benzodiazepines (eg, clonazepam)
- Dopamine precursors (eg, levodopa and carbidopa)

How well did you know this?
1
Not at all
2
3
4
5
Perfectly
20
Q

What is Malaria?

A

Parasitic disease caused by infection with Plasmodium protozoa transmitted by an infective female Anopheles mosquito.
Plasmodium falciparum infection carries a poor prognosis with a high mortality if untreated.

How well did you know this?
1
Not at all
2
3
4
5
Perfectly
21
Q

What are the Signs and symptoms of malaria?

A
  • Headache
  • Cough
  • Fatigue
  • Malaise
  • Shaking chills
  • Arthralgia
  • Myalgia
  • Paroxysm of fever, shaking chills, and sweats (every 48 or 72 hours, depending on species)

Less common symptoms include the following:
- Anorexia and lethargy
- Nausea and vomiting
- Diarrhea
- Jaundice

  • Cerebral malaria (sometimes with coma)
  • Severe anemia
  • Respiratory abnormalities: Include metabolic acidosis, associated respiratory distress, and pulmonary edema; signs of malarial hyperpneic syndrome include alar flaring, chest retraction, use of accessory muscles for respiration, and abnormally deep breathing
  • Renal failure (typically reversible)
How well did you know this?
1
Not at all
2
3
4
5
Perfectly
22
Q

What is the treatment of malaria ?

A

Quinine-based therapy is with quinine (or quinidine) sulfate plus doxycycline or clindamycin or pyrimethamine-sulfadoxine;

How well did you know this?
1
Not at all
2
3
4
5
Perfectly
23
Q

What is the classic triad of bacterial meningitis ?

A
  • Fever
  • Headache
  • Neck stiffness
How well did you know this?
1
Not at all
2
3
4
5
Perfectly
24
Q

Name the medications in which can be used as meningitis prophylaxis ?

A

Rifampin, quinolones, and ceftriaxone are the antimicrobials that are used to eradicate meningococci from the nasopharynx.

How well did you know this?
1
Not at all
2
3
4
5
Perfectly
25
Q

Malaria Chemoprophylaxis?

A
  • Atovaquone/proguanil (Malarone)
    50/100 mg per day one to two days before travel through seven days after return.
  • Doxycycline:
    100 mg per day one to two days before travel through four weeks after return.
  • Mefloquine:
    250 mg per week one week before travel through four weeks after return
  • Chloroquine (Aralen):
    500 mg per week one to two weeks before travel through four weeks after return
How well did you know this?
1
Not at all
2
3
4
5
Perfectly
26
Q

How to manage pulmonary embolism ?

A
  • Anticoagulation:
    • LMWH or fondaparinux.
    • If concerns regarding subcutaneous absorption arise, severe renal failure exists, or if thrombolytic therapy is being considered, IV UFH is the recommended form of initial anticoagulation.
    • Three months of treatment with anticoagulation
  • Thrombolytic therapy:
    • In patients with acute PE associated with hypotension (systolic BP< 90 mm HG) who do not have a high bleeding risk.
      Thrombolytic therapy is not recommended for most patients with acute PE not associated with hypotension.
  • Direct thrombin inhibitors and factor Xa inhibitors:
    • Apixaban, dabigatran, rivaroxaban, and edoxaban are alternatives to warfarin for prophylaxis and treatment of PE.
  • Warfarin therapy:
    • A vitamin K antagonist such as warfarin should be started on the same day as anticoagulant therapy in patients with acute PE. [5] Parenteral anticoagulation and warfarin should be continued together for a minimum of at least five days and until the INR is 2.0.
    • The recommended therapeutic range for venous thromboembolism is an INR of 2-3. This level of anticoagulation markedly reduces the risk of bleeding without the loss of effectiveness. Initially, INR measurements are performed on a daily basis; once the patient is stabilized on a specific dose of warfarin, the INR determinations may be performed every 1-2 weeks or at longer intervals.
    • A patient with a first thromboembolic event occurring in the setting of reversible risk factors, such as immobilization, surgery, or trauma, should receive warfarin therapy for at least 3 months.
How well did you know this?
1
Not at all
2
3
4
5
Perfectly
27
Q

How to approach a pregnant patient with high suspicion of PE?

A

D-dimer if -ve but still high suspicion -> bilateral leg Doppler assessment.
If the results are positive, the patient should be treated for pulmonary embolism. If the results are negative, CT pulmonary angiography is the next step.

Pregnant patients diagnosed with DVT or pulmonary embolism may be treated with LMWH throughout their pregnancy. Warfarin is contraindicated, because it crosses the placental barrier and can cause fetal malformations

How well did you know this?
1
Not at all
2
3
4
5
Perfectly
28
Q

What is pheochromocytoma ?

A

It is a rare, catecholamine-secreting tumor derived from chromaffin cells.

How well did you know this?
1
Not at all
2
3
4
5
Perfectly
29
Q

What are the syndromes that are associated with pheochromocytoma?

A
  • Von Hippel-Lindau (VHL) syndrome.
  • Multiple endocrine neoplasia type 2 (MEN 2)
  • Neurofibromatosis type 1 (NF1)
How well did you know this?
1
Not at all
2
3
4
5
Perfectly
30
Q

What are the precintage of malignant pheochromocytoma?

A

10% of pheochromocytomas and 35% of extra-adrenal pheochromocytomas are malignant.

How well did you know this?
1
Not at all
2
3
4
5
Perfectly
31
Q

What are the common metastatic sites of malignant pheochromocytoma ?

A

Bone
Liver
Lymph nodes.

How well did you know this?
1
Not at all
2
3
4
5
Perfectly
32
Q

What is the classic presentation and what are the other clinical signs associated with pheochromocytomas ?

A

Classically:
- Headaches
- Palpitations
- Diaphoresis
- Severe hypertension

Other:
- Tremor
- Nausea
- Weakness
- Anxiety, sense of doom
- Epigastric pain
- Flank pain
- Constipation
- Hypertension: Paroxysmal in 50% of cases
- Postural hypotension: From volume contraction
- Hypertensive retinopathy
- Weight loss
- Pallor
- Fever
- Tremor
- Neurofibromas
- Tachyarrhythmias
- Pulmonary edema
- Cardiomyopathy
- Ileus
- Café au lait spots

How well did you know this?
1
Not at all
2
3
4
5
Perfectly
33
Q

How to Diagnose pheochromocytoma and what are the tests used ?

A
  • Plasma metanephrine testing:
    In patients at high risk (ie, those with predisposing genetic syndromes or a family or personal history of pheochromocytoma)
  • 24-hour urinary collection for catecholamines and metanephrines:
    • In patients at lower risk.

Imaging studies should be performed only after biochemical studies have confirmed the diagnosis of pheochromocytoma:

  • Abdominal CT scanning.
  • MRI: Preferred over CT scanning in children and pregnant or lactating women.
  • Scintigraphy: Reserved for biochemically confirmed cases in which CT scanning or MRI does not show a tumor.
  • PET scanning.

Additional studies to rule out a familial syndrome:
- Serum intact parathyroid hormone level and a simultaneous serum calcium level to rule out primary hyperparathyroidism (which occurs in MEN 2A).

  • Screening for mutations in the ret proto-oncogene (which give rise to MEN 2A and 2B).
  • Genetic testing for mutations causing the MEN 2A and 2B syndromes.
  • Consultation with an ophthalmologist to rule out retinal angiomas (VHL disease)
How well did you know this?
1
Not at all
2
3
4
5
Perfectly
34
Q

How to manage pheochromocytoma ?

A

Surgical resection of the tumor is the treatment of choice.

Careful preoperative treatment with alpha and beta blockers is required to control blood pressure and prevent intraoperative hypertensive crises:

  • Start alpha blockade with phenoxybenzamine 7-10 days preoperatively.
  • Provide volume expansion with isotonic sodium chloride solution.
  • Encourage liberal salt intake
  • Initiate a beta blocker only after adequate alpha blockade, to avoid precipitating a hypertensive crisis from unopposed alpha stimulation.
  • Administer the last doses of oral alpha and beta blockers on the morning of surgery.
How well did you know this?
1
Not at all
2
3
4
5
Perfectly
35
Q

What is Uncomplicated Cystitis ?
usually occur in which type of patients?
what are the Sx ?

A

Uncomplicated cystitis occurs in patients who have a
- Normal, u
- Unobstructed genitourinary tract.
- who have no history of recent instrumentation
- whose symptoms are confined to the lower urinary tract.

Uncomplicated cystitis is most common in:
- young
- sexually active women.

Patients usually present with:
- dysuria
- urinary frequency
- urinary urgency
- suprapubic pain

How well did you know this?
1
Not at all
2
3
4
5
Perfectly
36
Q

What is the management of uncomplicated Cystitis in Nonpregnant Patients ?

A

First-line therapy:
- Trimethoprim/sulfamethoxazole 160 mg/800 mg (Bactrim DS, Septra DS) 1 tablet PO BID for 3d or

  • Nitrofurantoin monohydrate/macrocrystals (Macrobid) 100 mg PO BID for 5-7d or
    -Nitrofurantoin macrocrystals (Macrodantin) 50-100 mg PO QID for 7d or
  • Fosfomycin (Monurol) 3 g PO as a single dose with 3-4 oz of water.

Second-line therapy:

  • Ciprofloxacin (Cipro) 250 mg PO BID for 3d or
  • Ciprofloxacin extended release (Cipro XR) 500 mg PO daily for 3d or

-Levofloxacin (Levaquin) 250 mg PO q24h for 3d or

  • Ofloxacin 200 mg PO q12h for 3d

Alternative therapy:
- Amoxicillin-clavulanate (Augmentin) 500 mg/125 mg PO BID for 3-7d or

  • Amoxicillin-clavulanate (Augmentin) 250 mg/125 mg PO TID for 3-7d or
  • cefdinir 300 mg PO BID for 7d or
  • cefaclor 500 mg PO TID for 7d or
  • cefpodoxime 100 mg PO BID for 7d or
  • cefuroxime 250 mg PO BID for 7-10d
How well did you know this?
1
Not at all
2
3
4
5
Perfectly
37
Q

How to manage Complicated Cystitis in Nonpregnant Women ?

A

First-line therapy
Oral:
- Ciprofloxacin (Cipro) 500 mg PO BID for 7-14d or

  • Ciprofloxacin extended release (Cipro XR) 1 g PO daily for 7-14d or
  • levofloxacin (Levaquin) 750 mg PO daily for 5d

Parenteral:
- Ciprofloxacin (Cipro) 400 mg IV q12h for 7-14d or

  • Levofloxacin (Levaquin) 750 mg IV daily for 5d or
  • Ampicillin 1-2 g IV q6h + gentamicin 2 mg/kg/dose q8h for 7-14d or
  • piperacillin-tazobactam (Zosyn) 3.375 g IV q6h or
  • Doripenem 500 mg (Doribax) IV q8h for 10d or
  • Imipenem-cilastatin (Primaxin) 500 mg IV q6h for 7-14d or
  • Meropenem (Merrem) 1 g IV q8h for 7-14d

Second-line therapy
- cefepime (Maxipime) 2 g IV q12h for 10d or

  • ceftazidime (Fortaz, Tazicef) 500 mg IV or IM q8-12h for 7-14d
  • Duration of therapy: shorter courses (7d) are reasonable if patient improves rapidly; longer courses (10-14d) are reasonable if patient has a delayed response or is hospitalized.
  • Parenteral therapy can be switched to oral therapy once clinical improvement is observed.
How well did you know this?
1
Not at all
2
3
4
5
Perfectly
38
Q

How to manage Urinary Tract Infections in Pregnancy ?

A
  • In pregnancy, treating asymptomatic and symptomatic bacteriuria is important.
  • Behavioral methods include the following:
  • Avoid baths
  • Wipe front-to-back after urinating or defecating
  • Wash hands before using the toilet
  • Use washcloths to clean the perineum
  • Use liquid soap to prevent colonization from bar soap
  • Clean the urethral meatus first when bathing.
  • Tx in asymptomatic bacteruria during pregnancy + symptomatic:
  • Nitrofurantoin monohydrate/macrocrystals 100 mg orally twice daily for 5-7 days or
  • Amoxicillin 875 mg orally twice daily (alternative: 500 mg orally three times daily) for 5-7 days or
  • Amoxicillin-clavulanate 500/125 mg orally three times daily for 5-7 days (alternative: 875/125 mg orally two times daily for 5-7 days) or
  • Cephalexin 500 mg orally four times daily for 5-7 days or
  • Fosfomycin 3 g orally as a single dose with 3-4 oz. of water
How well did you know this?
1
Not at all
2
3
4
5
Perfectly
39
Q

What are the side effects of Angiotensin-converting enzyme (ACE) inhibitors ?

A
  • Dry cough
  • Hyperkalemia
  • Fatigue
  • Dizziness from blood pressure going too low
  • Headaches
  • Loss of taste
How well did you know this?
1
Not at all
2
3
4
5
Perfectly
40
Q

What is the chance of transmission of the following virusis, in needle stick injury :
- HCV
- HIV
- HBV ?

A
  • HCV : 3%
  • HIV: 0.3 %
  • HBV: 30%
How well did you know this?
1
Not at all
2
3
4
5
Perfectly
41
Q

What is Cardiac Tamponade Treatment ?

A
  • Pericardiocentesis.
  • Patients should be monitored in an intensive care unit.
  • Oxygen
  • Volume expansion with blood, plasma, dextran, or isotonic sodium chloride solution.
  • Bed rest with leg elevation.
How well did you know this?
1
Not at all
2
3
4
5
Perfectly
42
Q

What is the classification of pulmonary hypertension ?

A
  • Group 1 - Pulmonary arterial hypertension (PAH)
  • Group 2 - Pulmonary hypertension due to left-sided heart disease
  • Group 3 - Pulmonary hypertension due to lung diseases and/or hypoxia
  • Group 4 - Chronic thromboembolic pulmonary hypertension (CTEPH)
  • Group 5 - Pulmonary hypertension with unclear or multifactorial etiologies,
How well did you know this?
1
Not at all
2
3
4
5
Perfectly
43
Q

Guillain-Barre Syndrome effective treatment ?

A

Intravenous immunoglobulin (IVIG) and plasma exchange have proved equally effective.

How well did you know this?
1
Not at all
2
3
4
5
Perfectly
44
Q

Status Epilepticus diagnostic workup ?

A
  • Check ABCs
  • Insert IV
  • STAT labs:
    • Electrolytes, Ca, Mg.
    • CBC
    • LFT + renal function tests.
    • Toxicology screen
    • Anticonvulsant level
    • ABG
  • Insert urinary catheter.
  • Urinanalysis + urine toxicology
  • Cardiac O2 saturation + monitors
  • Consider:
    • Trauma
    • Stroke
    • Infection
    • Drug ingestion
  • Do As indicated:
    • CXR
    • CT scan / MRI
    • Lumbar puncture
    • Blood cultures
    • Blood toxicology screen
  • Treat underlying cause
  • Admit to hospital.
How well did you know this?
1
Not at all
2
3
4
5
Perfectly
45
Q

Status Epilepticus treatment?

A
  • Start IV line
  • Administer a 50 mL bolus of 50% dextrose IV + 100mg Thiamine. (Add Naloxone 0.4 - 2 mgIV to dextrose bag if OD is suspected).
  • Start anticonvulsant:
    • Diazepam 0.15 mg/Kg OR Lorazepam 0.1mg/kg IV over 5 mins OR IM midazolam.
    • Followed by: Fosphyntoin 15-20mg (not to exceed 150mg PE/min. OR phenytoin 18-20 mg/kg (not to exceed 50mg/min)
    • Never mix Phenytoin with 5% dextrose solution, put it in normal saline.
  • Intubate if indicated
  • Control hyperthermia
  • If seizure continued after 20 mins give additional Fosphyntoin 10mg PE/Kg IV. OR Phenytoin 10mg/Kg IV.
  • If seizure continued after 20 mins, give Phenobarbital 15mg/Kg IV
  • If seizure continued consider general anesthesia.

*PE = phenytoin equivalents

How well did you know this?
1
Not at all
2
3
4
5
Perfectly
46
Q

What are the types of urinary Incontinence ?

A
  • Stress:
    Urine leakage associated with increased abdominal pressure from laughing, sneezing, coughing, climbing stairs, or other physical stressors on the abdominal cavity and, thus, the bladder
  • Urge:
    An occasional sudden need to urinate with large volume urine loss; can also exist without incontinence.
    May be associated with pregnancy, childbirth, menopause, pelvic trauma, and neurologic diseases such as Parkinson’s disease and multiple sclerosis.
  • Mixed:
    Combination of stress and urge incontinence.
  • Overflow:
    A frequent dribble of urine due to inefficient bladder emptying; symptoms are similar to stress incontinence.
    Many causes, such as spinal cord injury, diabetes, neurological damage, Parkinson’s disease, multiple sclerosis, or an enlarged prostate
  • Functional:
    Urine loss not associated with any pathology or problem in the urinary system.
    Associated with physical or cognitive impairment such as immobility, Alzheimer’s disease, or head injury.
  • Reflex:
    Reflex (spastic bladder) incontinence happens when the bladder fills with urine and an involuntary reflex causes it to contract in an effort to empty.
    Usually occurs when the spinal cord is injured above the area medically labeled as the “T12” level.
How well did you know this?
1
Not at all
2
3
4
5
Perfectly
47
Q

What is Celiac disease ?

A

also known as celiac sprue or gluten-sensitive enteropathy, is a chronic disorder of the digestive tract that results in an inability to tolerate gliadin, the alcohol-soluble fraction of gluten. Gluten is a protein commonly found in wheat, rye, and barley.

When patients with celiac disease ingest gliadin, an immunologically mediated inflammatory response occurs that damages the mucosa of their intestines, resulting in maldigestion and malabsorption of food nutrients.

How well did you know this?
1
Not at all
2
3
4
5
Perfectly
48
Q

What is the best first test for suspected celiac disease, ?

A

Antibody testing, especially immunoglobulin :
Anti-tissue transglutaminase antibody (IgA TTG). although biopsies are needed for confirmation

How well did you know this?
1
Not at all
2
3
4
5
Perfectly
49
Q

How to investigate in acromegaly ?

A
  • Oral glucose tolerance test is the definitive test for the diagnosis of acromegaly; a positive result is the failure of GH to decrease to < 1 mcg/L after ingesting 50-100 g of glucose.
  • Serum insulinlike growth factor 1 (IGF-1) level is the best endocrinologic test for acromegaly. IGF-1 reflects GH concentration in the last 24 hours.
  • Thyrotrophin-releasing hormone (TRH), 200 mcg, can be given to increase the test’s accuracy. A GH level > 5 mcg/L suggests acromegaly.
How well did you know this?
1
Not at all
2
3
4
5
Perfectly
50
Q

How to investigate in Cushing disease and Cushing syndrome ?

A
  • Dexamethasone suppression test: The physiological basis of this test is a decrease in adrenocorticotropic hormone (ACTH) secretion by the pituitary because of exogenous glucocorticoid administration. One mg of dexamethasone is administered. Serum cortisol level is measured the next morning; it should be < 138 nmol/L (ie, < 5 mcg/dL).
  • Standard low-dose dexamethasone:
    The patient is then given 4 doses of 0.5 mg of dexamethasone at 6-hour intervals. Normal suppression is a serum cortisol level of < 138 nmol/L or a urine level of less than 55 nmol/L.
  • High-dose dexamethasone suppression confirms diagnosis of a pituitary adenoma. It suppresses the pituitary gland even in the presence of an adenoma. If cortisol levels remain unchanged, the cause of increased cortisol is not a pituitary adenoma.
How well did you know this?
1
Not at all
2
3
4
5
Perfectly
51
Q

What are the types of pleural effusion ?

A
  • Transudative pleural effusion: is caused by fluid leaking into the pleural space. This is from increased pressure in the blood vessels or a low blood protein count. Heart failure is the most common cause.
  • Exudative effusion: is caused by blocked blood vessels or lymph vessels, inflammation, infection, lung injury, and tumors.
How well did you know this?
1
Not at all
2
3
4
5
Perfectly
52
Q

Light’s criteria ?

A

These criteria classify an effusion as exudate if one or more of the following are present:
- (1) the ratio of pleural fluid protein to serum protein is greater than 0.5,

  • (2) the ratio of pleural fluid lactate dehydrogenase (LDH) to serum LDH is greater than 0.6, or
  • (3) the pleural fluid LDH level is greater than two thirds of the upper limit of normal for serum LDH.
How well did you know this?
1
Not at all
2
3
4
5
Perfectly
53
Q

What are the main indications for splenectomy in SCD?

A

1) acute splenic sequestration crisis
2) hypersplenism
3) splenic abscess.

How well did you know this?
1
Not at all
2
3
4
5
Perfectly
54
Q

What is the mainstay of treatment for patients with primary melanoma ?

A

Wide local excision with completion lymph node dissection (CLND) in patients with positive sentinel lymph node biopsy results is considered the mainstay of treatment for patients with primary melanoma.

How well did you know this?
1
Not at all
2
3
4
5
Perfectly
55
Q

Is surgery appropriate in melanoma in patients with solitary or acutely symptomatic brain metastases ?

A

Surgical management may alleviate symptoms and provide local control of disease.

How well did you know this?
1
Not at all
2
3
4
5
Perfectly
56
Q

What are the guidelines recommendation regarding adjuvant treatment in melanoma ?

A

In patients with no BRAF mutation:
- Single-agent immunotherapy with the programmed cell death–1 (PD-1) inhibitor pembrolizumab or
- Nivolumab or
- Combination therapy with nivolumab plus ipilimumab.

For patients with a BRAF mutation:
- Targeted combination therapy with dabrafenib/trametinib or
- Vemurafenib/cobimetinib.

**Targeted therapy is preferred if clinically needed for early response. Current targeted therapies can slow tumor growth (eg, BRAF inhibition) or release the brakes on the immune response, resulting in tumor lysis (eg, PD-1 inhibition).

How well did you know this?
1
Not at all
2
3
4
5
Perfectly
57
Q

What is Nitrous oxide (N2O)?
what are the indications ?
What are the contra-indications ?

A

It is known as laughing gas or happy gas used as anesthetics in both dental and medical applications.

Indications:
- General anesthesia, in combination with other anesthetics.
- In Dentistry:
to decrease the pain and anxiety associated with procedures. It is commonly delivered by a nasal mask in combination with oxygen.

Indications in adult dental patients include anxiety, low pain tolerance, underlying psychiatric disorders, and mental retardation.
In prolonged dental procedures + in patients with hyperresponsive gag reflexes.

Contraindications :
- Significant respiratory compromise.
- Nitrous oxide can leave the bloodstream and enter air-filled cavities 34 times faster than nitrogen. As a result, nitrous is contraindicated in patients in whom expansion of these air-filled cavities could compromise patient safety. This includes patients with pneumothorax, pulmonary blebs, air embolism, bowel obstruction, and those undergoing surgery of the middle ear.
- Nitrous oxide is absolutely contraindicated in patients who have had eye surgery that uses an intraocular gas.

How well did you know this?
1
Not at all
2
3
4
5
Perfectly
58
Q

Positive seatbelt sign indicates which type of injuries ?

A

A positive seatbelt sign, in combination with abdominal pain or tenderness, results in a higher likelihood of intra-abdominal injuries, in particular bowel/mesenteric injury

How well did you know this?
1
Not at all
2
3
4
5
Perfectly
59
Q

How to treat Echinococcosis Hydatid Cyst ?

A
  • Albendazole alone (if < 5 cm in diameter).
  • Percutaneous treatment in association with medical therapy (if cysts are 5-10 cm in diameter).
  • For cysts larger than 10 cm, continuous catheterization may be a viable option.
  • Percutaneous treatment:
    The puncture of echinococcal cysts has long been discouraged because of risks of anaphylactic shock and spillage of the fluid.
How well did you know this?
1
Not at all
2
3
4
5
Perfectly
60
Q

What are the most common malignancies associated with HCV ?

A
  • Hepatocellular carcinoma (HCC).
  • Non-Hodgkin lymphoma.
How well did you know this?
1
Not at all
2
3
4
5
Perfectly
61
Q

How to manage Mallory-Weiss Tear ?

A
  • Resuscitative measures as appropriate, performing endoscopy promptly, and triaging patients to intensive care, hospital inpatient, or outpatient management, depending on the severity of bleeding, comorbidities, and risk of rebleeding and complications.
  • Monitor the patient’s vital signs.
  • Obtain serial hemoglobin and hematocrit values (q6h initially).
  • Watch for clinical signs of rebleeding.
  • Correct coagulopathy, and maintain hemodynamic support with fluid and blood replacement.
  • Control or eliminate precipitating factors, such as nausea and vomiting.
  • Acid suppression (eg, omeprazole) and antiemetic drug therapy (eg, prochlorperazine).
  • Transfuse, generally, for hemoglobin levels less than 8 g/dL (< 10 g/dL for patients with cardiopulmonary disease).
  • Endoscopic Mx:
    • contact thermal modality.
    • Epinephrine injection.
    • Sclerosant injection.
    • Argon plasma coagulation.
    • Band ligation
    • Hemoclip placement
    • Balloon tamponade
  • Angiotherapy with either selective vasopressin infusion or embolization of the left gastric artery.
  • Outpatient Monitoring:
    • Watch for recurrent symptoms or signs of rebleeding.
    • Acid suppressant (eg, proton pump inhibitor; omeprazole, 20 mg PO qd) or a mucosal protectant (eg, sucralfate, 1 g PO qid) for 1-2 weeks to accelerate healing by reducing injurious factors, such as acid, pepsin, or bile, that impair the healing of the mucosal tear.
    • An antiemetic (eg, prochlorperazine) is useful for controlling nausea and vomiting, common precipitating factors for Mallory-Weiss tears.
How well did you know this?
1
Not at all
2
3
4
5
Perfectly
62
Q

What is the medullary thyroid carcinoma management?

A

Total thyroidectomy

How well did you know this?
1
Not at all
2
3
4
5
Perfectly
63
Q

What are the contents of the jugular foramen?

A
  • Ascending pharyngeal and occipital arteries enter the jugular foramen.
  • The glossopharyngeal, vagus, and accessory nerves pass through the jugular foramen on the medial side of the jugular bulb.
How well did you know this?
1
Not at all
2
3
4
5
Perfectly
64
Q

What is the presentation of jugular foramen injury ?

A
  • Vagus: Vagal compression initially causes paralysis of the laryngeal muscles leading to hoarseness and a nasal pitch. If the nerve gets compressed further, it results in unilateral paralysis of the soft palate and deviation of the uvula to the normal side.
  • Glossopharyngeal nerve leads to the following:
    • Loss of sensation to the posterior ipsilateral aspect of the tongue.
    • Reduced secretions from the ipsilateral parotid gland.
      *Loss of the ipsilateral gag reflex.
  • Accessory nerve: This manifests as drooping of the shoulder, difficulties in abducting the ipsilateral arm and rotating the head contralaterally due to weakness of sternocleidomastoid and trapezius muscles.
  • Obstruction of the traversing venous sinuses and veins: Headache and papilledema due to intracranial venous congestion leading to cerebral edema and raised intracranial pressure.
How well did you know this?
1
Not at all
2
3
4
5
Perfectly
65
Q

What is a Phyllodes tumor?

A

cystosarcoma phyllodes is a rare, predominantly benign tumor that occurs almost exclusively in the female breast.

How well did you know this?
1
Not at all
2
3
4
5
Perfectly
66
Q

What are the characteristics of a malignant phyllodes tumor ?

A
  • Recurrent malignant tumors seem to be more aggressive than the original tumor.
  • The lungs are the most common metastatic site, followed by the skeleton, heart, and liver.
  • Symptoms of metastatic involvement can arise from as early as a few months to as late as 12 years after the initial therapy.
  • Most patients with metastases die within 3 years of the initial treatment.
  • No cures for systemic metastases exist
    Roughly 30% of patients with malignant phyllodes tumors die of the disease.
How well did you know this?
1
Not at all
2
3
4
5
Perfectly
67
Q

What is the definitive methods for diagnosing phyllodes tumors?

+ how to manage ?

A

Open excisional breast biopsy for smaller lesions and incisional biopsy for large lesions are the definitive methods for diagnosing phyllodes tumors.

No absolute rules regarding margin size have been established. [25] However, a 2-cm margin for small (< 5 cm) tumors and a 5-cm margin for large (>5 cm) tumors have been advocated.

If the tumor-to-breast ratio is sufficiently high to preclude a satisfactory cosmetic result with segmental excision, total mastectomy, with or without reconstruction, is an alternative.

Axillary lymph node dissection should be performed only for clinically suspicious nodes.

There is no proven role for adjuvant chemotherapy or radiation therapy in the treatment of phyllodes tumors.

How well did you know this?
1
Not at all
2
3
4
5
Perfectly
68
Q

What is the commonest cause of acute abdomen in pregnancy ?

A

The commonest cause of acute abdomen in pregnancy is acute appendicitis followed by acute cholecystitis.

How well did you know this?
1
Not at all
2
3
4
5
Perfectly
69
Q

Which area does the pudendal nerve supplies?

A
  • Sensation to external genitalia of both sexes.
  • Skin around the anus, anal canal and perineum through its branches.
How well did you know this?
1
Not at all
2
3
4
5
Perfectly
70
Q

rectum innervation ?

A

The rectum receives sensory and autonomic innervation.

  • Sympathetic nervous supply to the rectum is from the lumbar splanchnic nerves and superior and inferior hypogastric plexuses.
  • Parasympathetic supply is from S2-4 via the pelvic splanchnic nerves and inferior hypogastric plexuses. Visceral afferent (sensory) fibres follow the parasympathetic supply.
How well did you know this?
1
Not at all
2
3
4
5
Perfectly
71
Q

What are the indications for initiation of long- term O2 therapy ?

A
  • Room air arterial partial pressure of O2 =/< 55 mmHg, OR 56-59 mmHg with cor pulmonate or signs of tissue hypoxia.
  • Room air O2 saturation =/< 88%, OR =/< 89% with cor pulmonate or signs of tissue hypoxia.
  • Nocturnal SpO2 =/< 88% (use O2 only at night).
  • Exercise hypoxemia with arterial partial pressure of O2 =/< 55mmHg or SpO2 =/< 88% (use O2 only with exercise).
How well did you know this?
1
Not at all
2
3
4
5
Perfectly
72
Q

Ophthalmology screening guidelines in DM ?

A
  • DM1 ——> 5 years after diagnosis, then Q 1 year.
  • DM2 —–> Q1 year
How well did you know this?
1
Not at all
2
3
4
5
Perfectly
73
Q

What is Turner syndrome and what is the clinical manifestation of the syndrome?

A

It is caused by the absence of one set of genes from the short arm of one X chromosome.

  • Swollen hands and feet because of lymphedema.
  • Dysplastic or hypoplastic nails and lymphedema gives a characteristic sausage-like appearance to the fingers and toes.
  • Higher incidence of congenital hip dislocation.
  • Short stature.
  • Adrenarche, the beginning of pubic hair growth, occurs at a normal age.
  • Breast development is absent when ovarian failure occurs before puberty.
  • Primary or secondary amenorrhea occurs with ovarian failure.
  • High arched palate.
  • Webbed neck.
  • Short fourth and fifth metacarpals and metatarsals.
  • Shield chest: The chest appears to be broad with widely spaced nipples.
  • Ears: Serous otitis media is more common [4] ; the auricles may be posteriorly rotated or low set; hearing loss due to otosclerosis is common in adults.
  • GI bleeding: due to intestinal vascular malformations, + high incidence of Crohn disease and ulcerative colitis.
  • Scoliosis.
  • Hypertension.
  • Coarctation of the aorta.
  • Renal anomalies.
  • Cardiac murmurs
    • Hypoplastic left heart
    • Coarctation of the aorta.
    • Bicuspid aortic valve.
    • Aortic dissection in adulthood
  • Thyroid: Hypothyroidism +/- thyroid enlargement
How well did you know this?
1
Not at all
2
3
4
5
Perfectly
74
Q

What is the antidote to acetaminophen overdose ?

A

N-acetylcysteine (NAC).

How well did you know this?
1
Not at all
2
3
4
5
Perfectly
75
Q

baby’s reflex smile at which age ?

A

2 months

How well did you know this?
1
Not at all
2
3
4
5
Perfectly
76
Q

What is the causitive agent of viral and bacterial gastroenteritis ?

A

Viral :
- Group A rotavirus.
- Calicivirus

Bacterial :
- Salmonella,
- Shigella, and
- Campylobacter species
are the top three leading causes of bacterial diarrhea worldwide, followed closely by Aeromonas species

How well did you know this?
1
Not at all
2
3
4
5
Perfectly
77
Q

Minimum daily exercise for children ?

A

180 minutes (three hours) a day.

CDC : 60 mins

How well did you know this?
1
Not at all
2
3
4
5
Perfectly
78
Q

What is the name of the medication that can be given in iron toxicity ?

A

Deferoxamine.
used to chelate iron

How well did you know this?
1
Not at all
2
3
4
5
Perfectly
79
Q

What is the classic Sx of measles ?

A
  • High fever
  • Cough
  • Runny nose
  • Watery eyes.

Measles rash appears 3 to 5 days after the first symptoms.

How well did you know this?
1
Not at all
2
3
4
5
Perfectly
80
Q

What is Transient tachypnea of the newborn (TTN) ?

A

It is a benign, self-limited condition that can present in infants of any gestational age, shortly after birth. It is caused due to delay in clearance of fetal lung fluid after birth which leads to ineffective gas exchange, respiratory distress, and tachypnea.

How well did you know this?
1
Not at all
2
3
4
5
Perfectly
81
Q

What is the causitive agent of Syphilis ?
And how does the transmission occur ?

A

Spirochete Treponema pallidum.

Transmissible by
- Sexual contact with infectious lesions.
- From mother to fetus in utero.
- Via blood product transfusion.
- Occasionally through breaks in the skin that come into contact with infectious lesions.

If untreated, it progresses through 4 stages: primary, secondary, latent, and tertiary.

How well did you know this?
1
Not at all
2
3
4
5
Perfectly
82
Q

What are the indications of Cesarean delivery ?

A
  • Previous cesarean delivery.
  • Breech presentation
  • Dystocia.
  • Fetal distress.
  • Repeat cesarean delivery
  • Obstructive lesions in the lower genital tract.
  • Pelvic abnormalities that preclude engagement or interfere with descent of the fetal presentation in labor.
  • Malpresentations (eg, preterm breech presentations, non-frank breech term fetuses)
  • Certain congenital malformations or skeletal disorders.
  • Infection.
  • Prolonged acidemia.
  • Abnormal placentation (eg, placenta previa, placenta accreta).
  • Abnormal labor due to cephalopelvic disproportion
How well did you know this?
1
Not at all
2
3
4
5
Perfectly
83
Q

Abruptio Placentae Clinical Presentation ?

A

Triad: bleeding, uterine contractions, and fetal distress.

  • Vaginal bleeding - 80%
  • Abdominal or back pain and uterine tenderness - 70%
  • Fetal distress - 60%
  • Abnormal uterine contractions (eg, hypertonic, high frequency) - 35%
  • Idiopathic premature labor - 25%
  • Fetal death - 15%
How well did you know this?
1
Not at all
2
3
4
5
Perfectly
84
Q

What is a Cystocele?

A
  • Simply: It is a protrusion of the bladder into the vagina due to defects in pelvic support.
  • Radiographic definition of a cystocele is descent of the bladder base below the inferior margin of the symphysis pubis.
How well did you know this?
1
Not at all
2
3
4
5
Perfectly
85
Q

How to diagnose cystocele ?

A
  • Physical examination.
  • Upright resting and straining cystogram.

During fluorourodynamic studies to evaluate incontinence, the type and degree of cystocele also can be assessed.

How well did you know this?
1
Not at all
2
3
4
5
Perfectly
86
Q

What is the clinical presentation of cystocele ?

A
  • Stress urinary incontinence during activity.
  • Difficulty emptying the bladder.
  • The presence of pelvic prolapse creates a feeling of incomplete bladder emptying or fullness.
How well did you know this?
1
Not at all
2
3
4
5
Perfectly
87
Q

What is the most common types of vulvar cancer?

A

Vulvar squamous cell carcinoma.

How well did you know this?
1
Not at all
2
3
4
5
Perfectly
88
Q

What is the treatment of mastitis in routine casess (outpatient) ?

A
  • Amoxicillin-clavulanate 875 mg PO BID for 10-14 days or
  • Dicloxacillin 500 mg PO QID for 10-14 days or
  • Flucloxacillin 250-500 mg PO QID for 5-7 days

If penicillin intolerance (not allergy):
- Cephalexin 500 mg PO QID for 10-14 days

If beta-lactam allergy:
- Clarithromycin 500 mg PO BID for 10-14 days

If suspected community-acquired methicillin-resistant Staphylococcus aureus (CA-MRSA) infection:
- Clindamycin 300 mg PO TID for 10-14 days or
- Trimethoprim-sulfamethoxazole 1 DS tablet PO BID for 10-14 days [4] (caution if nursing preterm infant or child with known or suspected glucose-6-phosphate dehydrogenase [G6PD] deficiency) or
- Doxycycline 100 mg PO BID for 10-14 days ( do not use in pregnancy or if breastfeeding ) or
- Ciprofloxacin 500 mg PO BID for 10-14 days (use caution if pregnant or breastfeeding).

How well did you know this?
1
Not at all
2
3
4
5
Perfectly
89
Q

What is the guidlines for Cervical Cancer Screening?

A
  • Recommended in women aged 21 to 65 years.
    21- 29 :
    Every 3 years with cervical cytology alone.

30 - 65:
- Every 3 years with cervical cytology alone OR
- Every 5 years with high-risk human papillomavirus (hrHPV) testing alone OR
- Every 5 years with hrHPV testing in combination with cytology (cotesting).

How well did you know this?
1
Not at all
2
3
4
5
Perfectly
90
Q

Definition of Gestational hypertension ?

A

Having a blood pressure higher than 140/90 measured on two separate occasions, more than 6 hours apart, without the presence of protein in the urine and diagnosed after 20 weeks of gestation(Pre-eclampsia)..

How well did you know this?
1
Not at all
2
3
4
5
Perfectly
91
Q

What is endometriosis ?

A

Endometriosis is defined as the presence of normal endometrial mucosa (glands and stroma) abnormally implanted in locations other than the uterine cavity.

How well did you know this?
1
Not at all
2
3
4
5
Perfectly
92
Q

What is the definitive method of diagnosis of endometriosis ?

A

Gross visualization of endometrial implants remains the definitive method of diagnosis:
- laparoscopy is the procedure of choice.
- Laparotomy can be another method of diagnosis. This is usually performed when another cause of patient pain is suspected.

How well did you know this?
1
Not at all
2
3
4
5
Perfectly
93
Q

What is the most common ectopic site of ectopic pregnncy ?

A
  • Fallopian tube (96% of cases): ampulla&raquo_space; isthmus > fimbriae > interstitial pregnancy.
  • Ovary (3% of cases).
  • Abdomen (1% of cases).
  • Cervix (very rare).
How well did you know this?
1
Not at all
2
3
4
5
Perfectly
94
Q

What is the initial test for ectopic pregnancy ?

A

β-hCG test + Transvaginal ultrasound (TVUS).

How well did you know this?
1
Not at all
2
3
4
5
Perfectly
95
Q

What are the risk factors of ectopic pregnancy ?

A

Anatomic alteration of the fallopian tubes is the main cause of ectopic pregnancy. It may be due to:
- A history of PID
- Previous ectopic pregnancy
- Past surgeries involving the fallopian tubes
- Endometriosis
- Exposure to DES (diethylstilbestrol) in utero
- Bicornuate uterus

Non‑anatomical risk factors
- Intrauterine device (IUD)
- History of infertility
- Hormone therapy

How well did you know this?
1
Not at all
2
3
4
5
Perfectly
96
Q

What is the clinical presentation of ectopic pregnancy ?

A
  • Lower abdominal pain and guarding.
  • Vaginal bleeding.
  • Signs of pregnancy: amenorrhea, nausea, breast tenderness, frequent urination.
  • Tenderness in the area of the ectopic pregnancy.
  • Cervical motion tenderness, closed cervix.
  • Enlarged uterus.
  • Interstitial pregnancies tend to present late, at 7–12 weeks of gestation, because of myometrial distensibility.
How well did you know this?
1
Not at all
2
3
4
5
Perfectly
97
Q

How frequent we should measure β-hCG if we’re suspecting ectopic pregnancy ?

A

Every 48 hours,

How well did you know this?
1
Not at all
2
3
4
5
Perfectly
98
Q

What are the findings of Transvaginal ultrasound (TVUS) in ectopic pregnancy ?

A
  • Empty uterine cavity in combination with a thickened endometrial lining.
  • Possible free fluid within the pouch of Douglas.
  • Possible extraovarian adnexal mass.
  • Tubal ring sign (blob sign) : an echogenic ring that surrounds an unruptured ectopic pregnancy .
  • Interstitial line sign: an echogenic line that extends from the gestational sac into the upper uterus (thought to be the echogenic appearance of the interstitial portion of the tube).
  • A thin myometrial layer (< 5 mm) surrounding the gestational sac [17]
How well did you know this?
1
Not at all
2
3
4
5
Perfectly
99
Q

What are te indications of Exploratory laparoscopy in ectopic pregnancy ?

A
  • Unstable patients suspected of having an ectopic pregnancy.
  • In pregnancy of unknown location if the location is still uncertain after 7–10 days.

Consider earlier laparoscopic exploration for high-risk patients (e.g., previous ectopic pregnancy).

How well did you know this?
1
Not at all
2
3
4
5
Perfectly
100
Q

What is the medical treatment of ectopic pregnancy + it mechanism of action + its indication + contraindications ?

A

The treatment of choice is methotrexate (MTX).

Mechanism of action: inhibits folate-dependent steps in DNA synthesis to terminate the rapidly dividing ectopic pregnancy.

Indications:
- Uncomplicated ectopic pregnancies.
- Hemodynamically stable patients
- Unruptured mass
- β-hCG ≤ 2,000 mlU/mL .
- Mass size < 3.5 cm .
- No fetal heartbeat

Absolute contraindications
- Pulmonary , renal , hepatic , or hematologic disease
- Breastfeeding
- Methotrexate sensitivity
- Immunodeficiency
- Peptic ulcer disease
- Ruptured ectopic pregnancy

How well did you know this?
1
Not at all
2
3
4
5
Perfectly
101
Q

What are the contraindications of External cephalic version ?

A

Absolute CI :
- Previous scar on the uterus.
- Placenta praaevia
- Unexplained APH (Antipartum hemorrhage)
- APH within last 7 days.
- Pre-eclampsia
- Multiple pregnancy.
- If C-section is required
- Abnormal cardiotocography
- Major uterine anamoly
- Ruptured membranes

Relative CI:
- Rhesus isoimmunisation
- Elderly primigravida.
- IUGR
- Oligohydramnios
- Polyhydramnios
- Protienuric pre-eclampsia
- Major fetal anomalies
- Unstable lie

How well did you know this?
1
Not at all
2
3
4
5
Perfectly
102
Q

What is the classification of perineal tear ?

A
  • First-degree tear: Injury to perineal skin and/or vaginal mucosa.
  • Second-degree tear: Injury to perineum involving perineal muscles but not involving the anal sphincter.
  • Third-degree tear: Injury to perineum involving the anal sphincter complex:
    • Grade 3a tear: Less than 50% of external anal sphincter (EAS) thickness torn.
    • Grade 3b tear: More than 50% of EAS thickness torn.
      *Grade 3c tear: Both EAS and internal anal sphincter (IAS) torn.
  • Fourth-degree tear: Injury to perineum involving the anal sphincter complex (EAS and IAS) and anorectal mucosa.
How well did you know this?
1
Not at all
2
3
4
5
Perfectly
103
Q

What is the most common ovarian mass in young women?

A

Follicular cyst of the ovary.

How well did you know this?
1
Not at all
2
3
4
5
Perfectly
104
Q

Patients with PCOS has an increase risk of which cancer ?

A

Endometrial cancer.

How well did you know this?
1
Not at all
2
3
4
5
Perfectly
105
Q

What are the clinical features of PCOS ?

A
  • Onset typically during adolescence.
  • Menstrual irregularities (primary or secondary amenorrhea, oligomenorrhea).
  • Difficulties conceiving or infertility
  • Obesity and possibly other signs of metabolic syndrome
  • Hirsutism
  • Androgenic alopecia
  • Acne vulgaris and oily skin
  • Acanthosis nigricans: hyperpigmented, velvety plaques (axilla, neck)
  • Premature adrenarche
How well did you know this?
1
Not at all
2
3
4
5
Perfectly
106
Q

How is the Blood hormone levels in PCOS ?

A
  • ↑ Testosterone (both total and free) or free androgen index
  • ↑ LH (LH:FSH ratio > 2:1)
  • Estrogen is normal or slightly elevated
How well did you know this?
1
Not at all
2
3
4
5
Perfectly
107
Q

How to treat PCOS ?

A

The therapeutic approach in PCOS is broadly based on whether or not the patient is seeking treatment for infertility.

If treatment for infertility is not sought:
- Therapy aimed at controlling menstrual, metabolic, and hormonal irregularities.
- If the patient is overweight (BMI ≥ 25 kg/m2):
* First-line: weight loss via lifestyle changes (e.g., dietary modifications, exercise).
* Second-line (as an adjunct): combined oral contraceptive therapy.

  • If the patient is not overweight: combined oral contraceptive therapy

If seeking treatment for infertility
- First-line
* Ovulation induction with clomiphene citrate or letrozole
Clomiphene inhibits hypothalamic estrogen receptors, thereby blocking the normal negative feedback effect of estrogen → increased pulsatile secretion of GnRH → increased FSH and LH, which stimulates ovulation.

If the patient is overweight: advise weight loss.

 * Second-line: ovulation induction with exogenous gonadotropins or laparoscopic ovarian drilling
How well did you know this?
1
Not at all
2
3
4
5
Perfectly
108
Q

HTN Tx in pregnancy in acute + nonemergent ?

A
  • Acute HTN : Hydralazine
  • HTN in pregnancy : Nifedipine (CCB).
  • Nonemergent HTN in pregnancy: Methyldopa.

Hypertensive Moms Need Love :
Hydralazine
Methyldopa
Nifedipine
Labetalol

How well did you know this?
1
Not at all
2
3
4
5
Perfectly
109
Q

Hyperthyroidism Tx in pregnancy ?

A
  • First trimester : Propylthiouracil
  • After first trimester: Switch to Methimazole for the rest of the pregnancy.
How well did you know this?
1
Not at all
2
3
4
5
Perfectly
110
Q

What are the types of Breech Presentation ?

A
  • Complete (flexed) breech – both legs are flexed at the hips and knees (fetus appears to be sitting ‘crossed-legged’).
  • Frank (extended) breech – both legs are flexed at the hip and extended at the knee. This is the most common type of breech presentation.
  • Footling breech – one or both legs extended at the hip, so that the foot is the presenting part.
How well did you know this?
1
Not at all
2
3
4
5
Perfectly
111
Q

When it is recommended to screen for DM in pregnant women ?

A

After 24 weeks of pregnancy.

How well did you know this?
1
Not at all
2
3
4
5
Perfectly
112
Q

What is Kartagener syndrome (KS) ?

A

It is an autosomal recessive syndrome, presents with the following triad:
1. Situs inversus
2. Chronic sinusitis
3. Bronchiectasis

How well did you know this?
1
Not at all
2
3
4
5
Perfectly
113
Q

Fanconi anemia mode of inheritance ?

A

Autosomal recessive pattern.

How well did you know this?
1
Not at all
2
3
4
5
Perfectly
114
Q

What are the causes of essential tremors ?

A

Positive family history (50–70%; autosomal dominant inheritance) or sporadic; benign form.

How well did you know this?
1
Not at all
2
3
4
5
Perfectly
115
Q

What is the clinical features of Essential tremor ?

A
  • Localization: hands (about 90%), head (about 30%; “yes-yes” or “no-no” motion), voice (about 15%)
  • Mostly bilateral postural tremor with a frequency of 5–10 Hz
  • Slowly progressive
  • Worse with voluntary movement , stress, fatigue, and caffeine
  • Resolves at rest
  • Improves with alcohol consumption
How well did you know this?
1
Not at all
2
3
4
5
Perfectly
116
Q

What is the treatment of Essential tremor ?

A
  • Drugs of choice: propranolol or primidone

Alternatives (if propanolol and primidone are unresponsive or contraindicated)
- Other beta blockers (e.g., atenolol, sotalol)
- Other anticonvulsants (e.g., gabapentin, topiramate) including certain benzodiazepines (e.g., alprazolam, clonazepam)

In drug-resistant cases
- Deep brain stimulation (DBS)
- Thalamotomy

How well did you know this?
1
Not at all
2
3
4
5
Perfectly
117
Q

What is Medullary sponge kidney ?

A

It is a benign congenital disorder characterized by dilatation of collecting tubules in one or more renal papillae, affecting one or both kidneys.
Usually is not diagnosed until the second or third decade of life or later

How well did you know this?
1
Not at all
2
3
4
5
Perfectly
118
Q

Interpret ankle - brachial index (ABI).

A

> 1.3 —> Calcified.
<0.9 —> Significant arterial disease.
0.4 - 0.9 —> Claudication
<0.4 —> critical limb ischemia.

How well did you know this?
1
Not at all
2
3
4
5
Perfectly
119
Q

What is the auscultation features of aortic regurgitation?

A
  • Site: Aortic area
  • Timing: Early diastolic
  • Radiates to the lower left sternal edge
  • Character: Decrescendo
  • Manoevres: Expiration , pt leaning forward.
How well did you know this?
1
Not at all
2
3
4
5
Perfectly
120
Q

What is the auscultation features of aortic stenosis ?

A
  • Site: Aortic area
  • Timing: Systolic
  • Radiates to the carotids
  • Character: ejection
  • Manoevres: Expiration
How well did you know this?
1
Not at all
2
3
4
5
Perfectly
121
Q

What is the
- Gross motor
- Fine motor adaptive
- Personal - social
- Language
developemental milestones in a 2 weeks old?

A
  • Gross motor: Moves head side to side.
  • Personal - social : regards face
  • Language: alerts to bell.
How well did you know this?
1
Not at all
2
3
4
5
Perfectly
122
Q

What is the
- Gross motor
- Fine motor adaptive
- Personal - social
- Language
developemental milestones in a 2 months old?

A
  • Gross motor: Lift shoulder while prone.
  • Fine motor adaptive : Tracks past midline.
  • Personal - social : smiles responsively.
  • Language : cooing and searches for sound with eyes.
How well did you know this?
1
Not at all
2
3
4
5
Perfectly
123
Q

What is the
- Gross motor
- Fine motor adaptive
- Personal - social
- Language
developemental milestones in a 4 months old?

A
  • Gross motor: Lifts up on hands, rolls front to back, If pulled to sit from supine there is no head lag.
  • Fine motor adaptive : Reaches for object, raking grasp.
  • Personal - social: Looks at hand, begins to work toward toys.
  • Language: laughs and squeals
How well did you know this?
1
Not at all
2
3
4
5
Perfectly
124
Q

What is the
- Gross motor
- Fine motor adaptive
- Personal - social
- Language
developemental milestones in a 6 months old?

A
  • Gross motor : sits alone
  • Fine motor adaptive: Transfers object hand to hand.
  • Personal - social: feeds self, hold bottle.
  • Language: babbles.
How well did you know this?
1
Not at all
2
3
4
5
Perfectly
125
Q

What is the
- Gross motor
- Fine motor adaptive
- Personal - social
- Language
developemental milestones in a 9 months old?

A
  • Gross motor: pulls to stand, gets into sitting position.
  • Fine motor adaptive: starts to pincer grasp, bangs 2 blocks togather.
  • Personal - social : waves bye-bye , plays pat a cack.
  • Language : Says dada and mama but nonspecific, 2 syllable sounds.
How well did you know this?
1
Not at all
2
3
4
5
Perfectly
126
Q

What is the
- Gross motor
- Fine motor adaptive
- Personal - social
- Language
developemental milestones in a 12 months old?

A
  • Gross motor: Walks, stoops and stands
  • Fine motor adaptive: Puts block in cup.
  • Personal - social: drink from a cup, imitates others.
  • Language : Says mama and dada specific, and says to other words.
How well did you know this?
1
Not at all
2
3
4
5
Perfectly
127
Q

What is the
- Gross motor
- Fine motor adaptive
- Personal - social
- Language
developemental milestones in a 15 months old?

A
  • Gross motor: walks backward
  • Fine motor adaptive: scribbles, stacks 2 blocks.
  • Personal - social: Uses spoon and fork, helps in housework
  • Language: 3 -6 words , follows command.
How well did you know this?
1
Not at all
2
3
4
5
Perfectly
128
Q

What is the
- Gross motor
- Fine motor adaptive
- Personal - social
- Language
developemental milestones in a 18 months old?

A
  • Gross motor: Runs.
  • Fine motor adaptive : stacks 4 blocks, kicks a ball.
  • Personal - social: Removes clothes, feeds doll.
  • Language : says at least 6 words.
How well did you know this?
1
Not at all
2
3
4
5
Perfectly
129
Q

What is the
- Gross motor
- Fine motor adaptive
- Personal - social
- Language
developemental milestones in a 2 years old?

A
  • Gross motor: walks up and down stairs, throws overhand.
  • Fine motor adaptive: stacks 6 blocks, copies line
  • Personal - social : washes and dries hand, brushes teeth, puts on clothes.
  • Language: puts 2 words togather, points to pics, knows body parts.

+ understands the concept of today.

How well did you know this?
1
Not at all
2
3
4
5
Perfectly
130
Q

What is the
- Gross motor
- Fine motor adaptive
- Personal - social
- Language
developemental milestones in a 3 years old?

A
  • Gross motor: walks steps alternating feets, broad jump.
  • Fine motor adaptive : Stacks 8 blocks , wiggles thumb
  • Personal - social: uses spoon well, put on T-shirt
  • Language: names pics, speech understandable to stranger 75%, says 3 words sentence.

+ understands the concept of tomorrow and yesterday

How well did you know this?
1
Not at all
2
3
4
5
Perfectly
131
Q

What is the
- Gross motor
- Fine motor adaptive
- Personal - social
- Language
developemental milestones in a 4 years old?

A
  • Gross motor: Balances well on each foot, hops on one foot
  • Fine motor adaptive: copies O + draws a person with 3 parts.
  • Personal - social : brushes teeth without help, dresses without help.
  • Language : name colors and understand adjectives.
How well did you know this?
1
Not at all
2
3
4
5
Perfectly
132
Q

What is the
- Gross motor
- Fine motor adaptive
- Personal - social
- Language
developemental milestones in a 5 years old?

A
  • Gross motor : skips, heels to toe walks
  • Fine motor adaptive: copies a square.
  • Language : counts , understands opposites.
How well did you know this?
1
Not at all
2
3
4
5
Perfectly
133
Q

What is the
- Gross motor
- Fine motor adaptive
- Personal - social
- Language
developemental milestones in a 6 years old?

A
  • Gross motor: balance on each foot 6 sec
  • Fine motor adaptive: Copies a trangle
  • Language : define words

+ begins to understand left and right

How well did you know this?
1
Not at all
2
3
4
5
Perfectly
134
Q

What are the stages of change for smoking cessation?

A
  1. Precontemplation “ما قبل التأمل”:
    current smoker not planning to quit in the next 6 months.
  2. Contemplation “the action of looking thoughtfully at something for a long time”:
    Current smoker, planning on quitting in next 6 months but never tried to quit during last year.
  3. Preparation:
    current smoker who is definitely planning to quit within next 30 days and have made a quit attempt in the past year.
  4. Action:
    not currently a smoker and have stopped within the past 6 months.
  5. Maintenance:
    Not currently a smoker, and stopped more than 6 months but less than 5 years.
How well did you know this?
1
Not at all
2
3
4
5
Perfectly
135
Q

What is the Canadian Cardiovascular Society grading scale for the classification of angina pectoris severity?

A
  • Class I - Angina only during strenuous or prolonged physical activity.
  • Class II - Slight limitation, with angina only during vigorous physical activity.
  • Class III - Symptoms with everyday living activities, ie, moderate limitation.
  • Class IV - Inability to perform any activity without angina or angina at rest, ie, severe limitation
How well did you know this?
1
Not at all
2
3
4
5
Perfectly
136
Q

Define Unstable angina.

A

Unstable angina is defined as new-onset angina (ie, within 2 mo of initial presentation) of at least class III severity, significant recent increase in frequency and severity of angina, or angina at rest.

How well did you know this?
1
Not at all
2
3
4
5
Perfectly
137
Q

Define Benign prostatic hyperplasia (BPH).

A

Benign prostatic hyperplasia (BPH): benign glandular and stromal hyperplasia of the transitional zone of the prostate.

How well did you know this?
1
Not at all
2
3
4
5
Perfectly
138
Q

What are the clinical features of Benign prostatic hyperplasia (BPH)?

A

Irritative symptoms of BPH:
- Urinary frequency
- Urinary urgency and urge incontinence
- Nocturia
- Occasionally dysuria

Obstructive symptoms of BPH
- Hesitancy
- Straining to urinate
- Poor and/or intermittent stream (not continuous)
- Prolonged terminal dribbling
- Sensation of incomplete voiding

Often gross hematuria

Digital rectal examination (DRE) findings: symmetrically enlarged, smooth (no nodules), firm, nontender prostate with rubbery or elastic texture .

*** To remember the symptoms of BPH, think “FUNWISE”: Frequency, Urgency, Nocturia, Weak stream /hesitancy, Intermittent stream, Straining to urinate, and Emptying (not emptying completely, terminal dribbling).

How well did you know this?
1
Not at all
2
3
4
5
Perfectly
139
Q

How to confirm the diagnosis of Benign prostatic hyperplasia (BPH)?

A

Diagnosis can be confirmed on abdominal ultrasound which demonstrates an enlarged prostate and increased post-void residual urine in the bladder

How well did you know this?
1
Not at all
2
3
4
5
Perfectly
140
Q

How to treat Benign prostatic hyperplasia (BPH)?

A
  1. Conservative management:
    - Restrict fluid intake before bedtime or before going out.
    - Avoid/reduce caffeine and alcohol intake .
    - Ensure complete bladder.
  2. Medical therapy:
    - Alpha-blockers (first-line):
    Examples: tamsulosin, doxazosin, terazosin, alfuzosin
  • 5-alpha-reductase inhibitors:
    Examples: finasteride, dutasteride
  • Parasympatholytics/anticholinergics:
    Examples: oxybutynin, darifenacin
  • Phosphodiesterase type 5 inhibitors:
    Example: tadalafil.
How well did you know this?
1
Not at all
2
3
4
5
Perfectly
141
Q

What is the mechanism of action of Alpha-blockers in the Tx of Benign prostatic hyperplasia (BPH)?

A

Mechanism of action: α1 receptors (α1A receptors) inhibition of the bladder neck and the prostatic urethra → relaxation of the smooth muscle of the bladder neck and the urethra → decreased resistance to urinary outflow → symptomatic improvement.

How well did you know this?
1
Not at all
2
3
4
5
Perfectly
142
Q

What is the mechanism of action of 5-alpha-reductase inhibitors in the Tx of Benign prostatic hyperplasia (BPH)?

A

Mechanism of action: ↓ conversion of testosterone to DHT (Dihydrotestosterone) → lower intraprostatic DHT levels → decreased prostatic growth and increased prostatic apoptosis and involution → improvement of LUTS (Lower urinary tract symptoms).

How well did you know this?
1
Not at all
2
3
4
5
Perfectly
143
Q

Male patient with androgenetic alopecia + Benign prostatic hyperplasia (BPH), which medication is appropriate to use ?

A

5-alpha-reductase inhibitors:
Examples: finasteride, dutasteride

How well did you know this?
1
Not at all
2
3
4
5
Perfectly
144
Q

What are the side effects of 5-alpha-reductase inhibitors (e.g finasteride, dutasteride) ?

A

Adverse effects:
- sexual dysfunction (erectile dysfunction, Decreased libido, Ejaculatory dysfunction)
- Gynecomastia

How well did you know this?
1
Not at all
2
3
4
5
Perfectly
145
Q

What are the surgical interventions used in Benign prostatic hyperplasia (BPH) ?

A
  1. Transurethral resection of the prostate (TURP):
    Procedure: resection of the hyperplastic prostatic tissue under cystoscopic guidance, using a cautery resectoscope.
  2. Transurethral incision of the prostate (TUIP):
    (used in pt with small prostates with obstructive symptoms or those at high risk for surgical complications).
  3. Open/laparoscopic/robotic prostatectomy.
How well did you know this?
1
Not at all
2
3
4
5
Perfectly
146
Q

HBV Postexposure Prophylaxis ?

A

Hepatitis B immune globulin is recommended in addition to vaccine for added protection.

How well did you know this?
1
Not at all
2
3
4
5
Perfectly
147
Q

How to treat infective endocarditis ?

A
  • Native valve endocarditis:
    penicillin G + gentamicin for coverage of streptococci.
  • History of IV drugs:
    Nafcillin + gentamicin.
  • History of prosthetic valves:
    Vancomycin + gentamicin + refampin
How well did you know this?
1
Not at all
2
3
4
5
Perfectly
148
Q

How to diagnose Subacute thyroiditis ?

A

Thyroid function tests
- Thyrotoxic phase: ↑ T3 and T4 , ↑ thyroglobulin, ↓ TSH
- Hypothyroid phase: ↓ T3 and T4, ↑ TSH

Confirmatory test
- ↑ ESR

Radioiodine uptake study : ↓ iodine uptake (< 5%)

Ultrasound: thyroid with poorly defined hypoechoic regions and decreased vascularity, giving rise to a cobblestone appearance

Histologic features :
Subacute granulomatous thyroiditis: granulomatous inflammation, multinucleated giant cells
Subacute lymphocytic thyroiditis: lymphocytic infiltration, sparse germinal centers

How well did you know this?
1
Not at all
2
3
4
5
Perfectly
149
Q

What is Familial Mediterranean fever ?

A

Familial Mediterranean fever (FMF), also known as recurrent polyserositis, is an autosomal recessive autoinflammatory disorder characterized mainly by brief recurrent episodes of peritonitis, pleuritis, and arthritis, usually with accompanying fever.

How well did you know this?
1
Not at all
2
3
4
5
Perfectly
150
Q

What are the absolute and relative contraindications for fibrinolytic use in ST-segment elevation myocardial infarction (STEMI)?

A

Absolute contraindications for fibrinolytic use in STEMI include the following:

  1. Prior intracranial hemorrhage (ICH).
  2. Known structural cerebral vascular lesion.
  3. Known malignant intracranial neoplasm.
  4. Ischemic stroke within 3 months.
  5. Suspected aortic dissection.
  6. Active bleeding or bleeding diathesis (excluding menses).
  7. Significant closed head trauma or facial trauma within 3 months.
  8. Intracranial or intraspinal surgery within 2 months.
  9. Severe uncontrolled hypertension (unresponsive to emergency therapy).
  10. For streptokinase, prior treatment within the previous 6 months.

Relative contraindications (not absolute) to fibrinolytic therapy include:

  1. Uncontrolled hypertension (BP > 180/110), either currently or in the past.
  2. Intracranial abnormality not listed as absolute contraindication (i.e. benign intracranial tumor).
  3. Ischemic stroke more than 3 months prior.
  4. Bleeding within 2 to 4 weeks (excluding menses).
  5. Traumatic or prolonged cardiopulmonary resuscitation
  6. Major surgery within 3 weeks.
  7. Pregnancy
  8. Current use of anticoagulants
  9. Non-compressible vascular puncture
  10. Dementia
How well did you know this?
1
Not at all
2
3
4
5
Perfectly
151
Q

What is the precipitating cause of rheumatic fever ?

A

Group A streptococcus (GAS) infections of the pharynx .

How well did you know this?
1
Not at all
2
3
4
5
Perfectly
152
Q

How to confirm rheumatic fever caused by Group A streptococcus (GAS) infection ?

A

By throat culture or rapid antigen detection test (RADT).

How well did you know this?
1
Not at all
2
3
4
5
Perfectly
153
Q

What is the treatment of rheumatic fever ( Group A streptococcus GAS pharyngitis) ?

A
  • Intramuscular penicillin G benzathine
  • oral penicillin V potassium
  • oral amoxicillin.

For patients with Penicillin allergy:

  • Narrow-spectrum cephalosporin (cephalexin [Keflex], cefadroxil [formerly Duricef]).
  • Azithromycin (Zithromax)
  • Clarithromycin (Biaxin)
  • Clindamycin (Cleocin)

** Patients are no longer considered contagious after 24 hours of antibiotic therapy.

How well did you know this?
1
Not at all
2
3
4
5
Perfectly
154
Q

What is the Duration of Secondary Prophylaxis for Rheumatic Fever ?

A
  • Rheumatic fever with carditis and residual heart disease (persistent valvular disease†):
    10 years or until age 40 years (whichever is longer); lifetime prophylaxis may be needed.
  • Rheumatic fever with carditis but no residual heart disease (no valvular disease†):
    10 years or until age 21 years (whichever is longer).
  • Rheumatic fever without carditis:
    5 years or until age 21 years (whichever is longer).
How well did you know this?
1
Not at all
2
3
4
5
Perfectly
155
Q

What is Dengue fever ?

A
  • Dengue is a viral disease caused by dengue virus (Serotype: DENV 1–4).
  • RNA virus of the genus Flavivirus
  • Transmission route: vector-borne: mosquitoes most commonly from the species Aedes aegypti .
How well did you know this?
1
Not at all
2
3
4
5
Perfectly
156
Q

What is the Clinical features of dengue fever ?

A
  • Incubation period: 2–14 days.
  • Children are usually asymptomatic.
  • Starts with fever and malaise that lasts ∼ 1 week.
  • Severe arthralgia and myalgia (often referred to as “break-bone fever”).
  • Severe headache and retro-orbital pain.
  • Maculopapular, measles-like exanthem (typically appears 2–5 days following fever).
  • Generalized lymphadenopathy.

**If symptoms appear more than 2 weeks after returning from a dengue-endemic region, it is very unlikely that dengue is the cause!

How well did you know this?
1
Not at all
2
3
4
5
Perfectly
157
Q

What is Dengue hemorrhagic fever (DHF) ?

A
  • Occurs in 1–2% of cases.
  • Generally develops as the initial fever subsides (∼ 1 week after onset).

Clinical manifestations:
- Temperature change: ranges from hypothermia to a second spike in fever.
- Abdominal pain, vomiting.
- Changes in mental status (e.g., confusion).
- Hemorrhagic manifestations (e.g., petechiae, epistaxis, gingival bleeding) .
- Positive capillary fragility test [7]
- Increased vascular permeability → signs of pleural effusion and/or ascites.

** Dengue shock syndrome (DSS): DHF + shock.

** Dengue hemorrhagic fever is more frequent in individuals who experience a repeat infection with a second serotype, especially serotype 2!

How well did you know this?
1
Not at all
2
3
4
5
Perfectly
158
Q

How to diagnose dengue fever ?

A

Laboratory tests
- Leukopenia
- Thrombocytopenia
- ↑ AST
- Hct elevated ≥ 20% of normal values if vascular permeability (in DHF)

Best test for confirming infection: serology (IgM, IgG)

Alternatives
- PCR
- Molecular methods (ELISA): detection of viral antigen

How well did you know this?
1
Not at all
2
3
4
5
Perfectly
159
Q

What is the treatmnet of dengue fever ?

A

SUPPORTIVE MANAGEMENT :

Symptomatic treatment
- Fluid administration to avoid dehydration
- Acetaminophen

Dengue hemorrhagic fever
- Blood transfusions in case of significant internal bleeding (e.g., epistaxis, gastrointestinal bleeding, or menorrhagia)
- IV fluids

How well did you know this?
1
Not at all
2
3
4
5
Perfectly
160
Q

What is the treatment of chronic HBV ?

A

Antiviral treatment:
- Indication: chronic active hepatitis B with evidence of liver inflammation (ALT ≥ 2 times upper limit) or cirrhosis.

Goals:
- Reduce HBV DNA below detectable levels
- Seroconversion of HBeAg to anti‑HBe
- Reverse liver disease

A.Nucleoside/nucleotide analogs : indicated for patients with both decompensated and compensated liver disease and nonresponders to interferon treatment:

 * Tenofovir is commonly the drug of choice 
 * Entecavir

B. Pegylated interferon alfa (PEG-IFN-a) :
- Especially in younger patients with compensated liver disease.
- Regimen is shorter than nucleoside/nucleotide analogs

Contraindications:
* Decompensated cirrhosis
* Psychiatric conditions
* Pregnancy
* Autoimmune conditions
* Leukopenia or thrombocytopenia
* Coinfection with HDV is best treated with PEG-IFN-a.

Surgical treatment:
- Liver transplantation
* In cases of end-stage liver disease due to HBV
* In cases of fulminant hepatic failure (emergent transplantation).

How well did you know this?
1
Not at all
2
3
4
5
Perfectly
161
Q

What is Hypochondriasis or hypochondria ?

A

It is a condition in which a person is excessively and unduly worried about having a serious illness.

How well did you know this?
1
Not at all
2
3
4
5
Perfectly
162
Q

What is the treatment of toxoplasmosis ?

A

Pyrimethamine and sulfadiazine, plus folinic acid.

How well did you know this?
1
Not at all
2
3
4
5
Perfectly
163
Q

Anorexia nervosa definition ?

A

It is an eating disorder characterized by an abnormally low body weight, an intense fear of gaining weight and a distorted perception of weight. People with anorexia place a high value on controlling their weight and shape, using extreme efforts that tend to significantly interfere with their lives.

How well did you know this?
1
Not at all
2
3
4
5
Perfectly
164
Q

Site of thoracentesis ?

A
  • The visceral and parietal pleura are thin layers of connective tissue, and the space between the two linings is the pleural space. The pleural space extends inferiorly to level of approximately the 10th intercostal space. The intercostal vascular bundles are located along the inferior aspect of the ribs which is an important consideration during needle insertion due to the potential risk of injury to this bundle.
  • The intercostal neurovascular bundle is located along the lower edge of each rib. Therefore, the needle must be placed over the upper edge of the rib to avoid damage to the neurovascular bundle.
  • The liver and spleen rise during exhalation and can go as high as the 5th intercostal space on the right (liver) and 9th intercostal space on the left (spleen).
How well did you know this?
1
Not at all
2
3
4
5
Perfectly
165
Q

What is Bulimia nervosa ?

A

It is a serious eating disorder, you eat large amounts of food (binge eating) and then purge to get rid of extra calorie.

How well did you know this?
1
Not at all
2
3
4
5
Perfectly
166
Q

What are indications for methacholine challenge testing?

A

It is used diagnose asthma, confirm a diagnosis of asthma, document the severity of hyperresponsiveness, and follow changes in hyperresponsiveness.

How well did you know this?
1
Not at all
2
3
4
5
Perfectly
167
Q

What is the DDx of microcytic anemia ?

A

Defective heme synthesis:
- Iron deficiency anemia (the most common)
- Lead poisoning
- Anemia of chronic disease (late phase)
- Sideroblastic anemia
Defective globin chain
- Thalassemia

***The causes of microcytic anemia can be remembered with IRON LAST: IRON deficiency, Lead poisoning, Anemia of chronic disease, Sideroblastic anemia, Thalassemia.

How well did you know this?
1
Not at all
2
3
4
5
Perfectly
168
Q

What is the DDx of Normocytic anemia ?

A

Hemolytic anemia:
A. Intrinsic defects
- Hemoglobinopathies
* Sickle cell anemia
* HbC disease
- Enzyme deficiencies
* Pyruvate kinase deficiency
* G6PD deficiency
- Membrane defects
* Paroxysmal nocturnal hemoglobinuria
* Hereditary spherocytosis
B. Extrinsic defects
- Autoimmune hemolytic anemia
- Microangiopathic hemolytic anemia
- Macroangiopathic hemolytic anemia
- Infections
- Mechanical destruction

Nonhemolytic anemia:
- Blood loss
- Aplastic anemia
- Anemia of chronic kidney disease
- Iron deficiency anemia (early phase)
- Anemia of chronic disease (early phase)

How well did you know this?
1
Not at all
2
3
4
5
Perfectly
169
Q

What is the DDx of Macrocytic anemia ?

A

Megaloblastic anemia: impaired DNA synthesis and/or repair with hypersegmented neutrophils

  • Vitamin B12 deficiency
  • Folate deficiency
  • Medications
    • Phenytoin
    • Sulfa drugs
    • Trimethoprim
    • Hydroxyurea
    • MTX
    • 6-mercaptopurine
  • Fanconi anemia
  • Orotic aciduria

Nonmegaloblastic anemia: normal DNA synthesis without hypersegmented neutrophils

  • Liver disease
  • Alcohol use
  • Diamond-Blackfan anemia
  • Myelodysplastic syndrome
  • Multiple myeloma
  • Hypothyroidism
How well did you know this?
1
Not at all
2
3
4
5
Perfectly
170
Q

Tennis elbow site of injury ?
name of test ?

A

Lateral epicondylitis
Mill’s test + Cozen’s test

How well did you know this?
1
Not at all
2
3
4
5
Perfectly
171
Q

Golfer’s elbow site of injury ?

A

Medial epicondylitis

How well did you know this?
1
Not at all
2
3
4
5
Perfectly
172
Q

What is the most common humerus fractures ?

A

Proximal humerus fractures.

How well did you know this?
1
Not at all
2
3
4
5
Perfectly
173
Q

What is the classification of humerus fractures ?

A
  1. Proximal humerus fracture (common in the elderly):
    - The proximal humerus has four major segments: the anatomical neck, the humeral shaft, the greater tuberosity, and the lesser tuberosity (the surgical neck is distal to the lesser and greater tuberosity).
    - Commonly used classification (Neer) is based on whether one or more of these four segments have been displaced.
  2. Humeral shaft fracture
    Classified according to location: proximal third, middle third (most common location), distal third.
    Or according to comminution: type A (no comminution), type B (butterfly fragment), and type C (comminution is present).
  3. Distal humerus fracture
    Classification according to anatomical site
    - Lateral/medial fractures
    - Supracondylar fractures (supracondylar fractures are the most common pediatric elbow fracture )
How well did you know this?
1
Not at all
2
3
4
5
Perfectly
174
Q

Which nerve is at risk in fractures of the middle third (midshaft) of the humerus ?

A

The radial nerve runs through the radial sulcus of the upper arm and is especially at risk in fractures of the middle third (midshaft) of the humerus!

How well did you know this?
1
Not at all
2
3
4
5
Perfectly
175
Q

How to treat Humerus fractures ?

A

A.Conservative therapy:
- Indication: nondisplaced, closed fractures
- Procedures:
* Hanging-arm cast or coaptation splint and sling for approx. one to two weeks; subsequent follow‑up X‑ray and brace.
* Early physical therapy to restore function.

B.Surgical treatment
- Indication: open fractures, displaced fractures that cannot be reduced, associated injuries (nerves, blood vessels), floating elbow (simultaneous humerus and forearm fracture), pseudarthrosis.
- Procedures:
* Internal fixation using plates and screws, or intramedullary implants (especially supracondylar fractures).
* External fixation (e.g., open fracture, polytrauma)
* Arthroplasty of humeral head or elbow (complex fractures or poor quality bone), especially in elderly patients.

How well did you know this?
1
Not at all
2
3
4
5
Perfectly
176
Q

In carpal tunnel syndrome which muscles will be paralyzed ?

A

The first and second lumbricals are supplied by the median nerve .

How well did you know this?
1
Not at all
2
3
4
5
Perfectly
177
Q

What is the motor function of the radial nerve ?

A

The radial nerve innervates the muscles located in the posterior arm and posterior forearm.

  • In the arm, it innervates the three heads of the triceps brachii, which acts to extend the arm at the elbow.
  • The radial nerve also gives rise to branches that supply the brachioradialis and extensor carpi radialis longus (muscles of the posterior forearm).
  • As a generalisation, these muscles act to extend at the wrist and finger joints, and supinate the forearm.
How well did you know this?
1
Not at all
2
3
4
5
Perfectly
178
Q

Which nerve innervates the medial aspect of the lower leg and the medial foot as far as the first metatarsal phalangeal joint ?

A

The saphenous nerve.

How well did you know this?
1
Not at all
2
3
4
5
Perfectly
179
Q

What are the Steps of nonoperative management in appendiceal mass ?

A
  • Empiric parenteral antibiotic therapy for 2–3 days.
  • Supportive care:
    • Bowel rest (NPO)
    • Intravenous fluids.
    • Electrolyte repletion as needed
    • IV analgesics (see pain management).
    • IV antiemetics as needed
    • Antipyretic therapy
  • Periappendiceal abscess > 4 cm: image-guided percutaneous drainage; send aspirate for cultures
  • Monitor vitals and serial abdominal examinations every 6–12 hours.
  • Insignificant improvement/worsening of symptoms: urgent surgical intervention.
  • Symptomatic improvement within 24–48 hours
    • Slow introduction of enteral nutrition.
    • Switch to oral antibiotics for 7-day course.
  • Schedule interval colonoscopy in patients > 40 years of age following NOM of acute appendicitis to rule out early colonic malignancy.

Medscape :
- Patients with a phlegmon or a small abscess: After intravenous (IV) antibiotic therapy, an interval appendectomy can be performed 4-6 weeks later.
- Patients with a larger well-defined abscess: After percutaneous drainage with IV antibiotics is performed, the patient can be discharged with the catheter in place. Interval appendectomy can be performed after the fistula is closed.
- Patients with a multicompartmental abscess: These patients require early surgical drainage.

How well did you know this?
1
Not at all
2
3
4
5
Perfectly
180
Q

What is Amebiasis ?
Pathogen + route of transmission ?

A

It is an infectious disease caused by the anaerobic protozoan Entamoeba histolytica.

Transmission
- Fecal-oral
- Amebic cysts are excreted in stool and can contaminate drinking water or food.
- Transmission may also occur through sexual contact.

How well did you know this?
1
Not at all
2
3
4
5
Perfectly
181
Q

What is the clinical features of Amebiasis ?

A

A. Intestinal amebiasis (Amebic dysentery)
- Loose stools with mucus and bright red blood.
- Painful defecation, tenesmus, abdominal pain, cramps, weight loss, and anorexia.
- Fever in 10–30% of cases and possible systemic symptoms (e.g., fatigue).
- High risk of recurrence, e.g., through self-inoculation (hand to mouth).
- A chronic form is also possible, which is clinically similar to inflammatory bowel disease.

B.Extraintestinal amebiasis
- In 95% of cases: amebic liver abscess, usually a solitary abscess in the right lobe
- Fever in 85–90% of cases (compared to amebic dysentery)
- RUQ pain or pressure sensation
- Chest pain, pleuralgia
- Diarrhea precedes only a third of all cases of amebic liver abscesses.
- In 5% of cases: abscesses in other organs (e.g., especially the lungs; in rare cases, the brain), with accompanying organ-specific symptoms.

How well did you know this?
1
Not at all
2
3
4
5
Perfectly
182
Q

How to treat Amebiasis ?

A

A.Medical therapy
Asymptomatic intestinal amebiasis
- No treatment in endemic areas
- In nonendemic areas:
- Luminal agents such as paromomycin, diloxanide, or iodoquinol
- Goal: To prevent the development of invasive disease and the shedding of cysts.

Symptomatic intestinal amebiasis and invasive extraintestinal amebiasis:
- Initial treatment with a nitroimidazole derivative such as metronidazole or tinidazole to eradicate invasive trophozoites.
- Followed by a luminal agent (e.g., paromomycin, diloxanide, or iodoquinol) to eradicate intestinal cysts and prevent relapse

B. Invasive procedures
Aspiration: ultrasound or CT-guided puncture of
complicated liver abscesses at risk for perforation
Indications:
* Localized in the left lobe
* Pyogenic abscess
* Multiple abscesses
* Failure to respond to pharmacotherapy

Surgical drainage: should generally be avoided, but may be indicated for inaccessible abscesses or ruptured abscesses in combination with peritonitis

***To ensure successful treatment, the patient’s stool must be analyzed regularly!

How well did you know this?
1
Not at all
2
3
4
5
Perfectly
183
Q

How to manage asymptomatic umbilical hernia in pediatrics ?

A

The literature does suggest that expectant management of asymptomatic hernias until age 4-5years, regardless of size of hernia defect, is both safe and the standard practice of many pediatric hospitals.

How well did you know this?
1
Not at all
2
3
4
5
Perfectly
184
Q

Where to insert the needle in tension pneumothorax ?

A

Treatment of tension pneumothorax is immediate needle decompression by inserting a large-bore (eg, 14- or 16-gauge) needle into the 2nd intercostal space in the midclavicular line.

How well did you know this?
1
Not at all
2
3
4
5
Perfectly
185
Q

Chest Tube Thoracostomy Anatomical Placement ?

A

5th intercostal space at the midaxillary line (note: in most patients this is lateral to the nipple at the point of the midaxillary line).

How well did you know this?
1
Not at all
2
3
4
5
Perfectly
186
Q

What is the management of Hepatocellular adenoma ?

A
  • Discontinue oral contraceptives.
  • Women with symptoms or tumor > 5 cm: indication for surgical resection because of increased risk of rupture, bleeding, or malignant transformation.
  • Men with hepatocellular adenoma: indication for surgical resection irrespective of the size of the lesion because of an increased risk of malignant transformation.
How well did you know this?
1
Not at all
2
3
4
5
Perfectly
187
Q

What is the treatment of varicose veins ?

A

A. Conservative measures
- Indications
* Superficial disease with no correctable cause of reflux.
* Postoperative period.
- Measures
* Compression therapy with compression stockings
* Frequent elevation of the legs
* Physical therapy, manual lymphatic drainage
* Avoid long periods of standing and sitting (with bent legs) and heat

B. Definite treatment:
- Indications:
* Symptomatic venous disease with correctable cause of reflux
* In case of complications such as bleeding, ulcers, or recurrent superficial thrombophlebitis.

  • Technique: vein ablation therapies
    • Interventional:
      **First-line: endovenous thermal ablation (laser and radiofrequency).
      ** Alternative: chemical ablation (sclerotherapy)
    • Open surgery with partial or complete removal of a vein: only for veins that are not accessible by interventional techniques .
How well did you know this?
1
Not at all
2
3
4
5
Perfectly
188
Q

How to classify Clinical signs of vascular injury ?

A

Hard signs:
- active hemorrhage,
- rapidly expanding hematomas,
- absent pulses,
- pallor,
- paresthesia,
- pain,
- paralyses,
- poikilothermia,
- palpable thrill or audible bruit.

Soft signs:
- history of arterial bleeding at the scene of injury,
- diminished distal unilateral pulse,
- small hematoma,
- neurological deficit,
- abnormal flow velocity wave on Doppler examination,
- abnormal ankle-brachial pressure index (ABI, <0.9).

***The presence or not of hard clinical signs may decide whether the patient needs an immediate operation, can undergo further investigation, or may only need continuous observation.

How well did you know this?
1
Not at all
2
3
4
5
Perfectly
189
Q

What are the recommendations of breast cancer screening ?

A
  • Women who are 50 to 74 years old and are at average risk for breast cancer get a mammogram every two years.

***Women who are 40 to 49 years old should talk to their doctor or other health care professional about when to start and how often to get a mammogram. Women should weigh the benefits and risks of screening tests when deciding whether to begin getting mammograms before age 50.

How well did you know this?
1
Not at all
2
3
4
5
Perfectly
190
Q
  1. What is the definition of Pancreatic pseudocysts?
  2. How to diagnose it ?
  3. How to treat it ?
  4. What are the complicaitons ?
A
  1. It is an encapsulated collection of pancreatic fluid that develops 4 weeks after an acute attack of pancreatitis; can occur in both acute and chronic pancreatitis.
  2. Diagnostics: abdominal ultrasound/CT/MRI → extrapancreatic fluid collection within well-defined wall/capsule; no solid cyst components detectable.
  3. Treatment : surgical/endoscopic cystogastrostomy/cystoduodenostomy/cystojejunostomy ; ultrasound/CT-guided percutaneous drainage.
  4. Complications
    - Infection → fever, abdominal pain, sepsis
    - Rupture → pancreatic ascites/pancreaticopleural fistula
    - Erosion into an abdominal vessel with hemorrhage into the cyst → sudden abdominal pain, signs of hemorrhagic shock .
How well did you know this?
1
Not at all
2
3
4
5
Perfectly
191
Q
  1. What is the definition of achalasia ?
  2. Clinical features ?
  3. initial work up ?
  4. Confirmation test ?
A
  1. Achalasia is a failure of the lower esophageal sphincter (LES) to relax that is caused by the degeneration of inhibitory neurons within the esophageal wall.
    • Dysphagia to solids and liquids; can be progressive or paradoxical
    • Regurgitation
    • Retrosternal pain and cramps
    • Weight loss.
  2. upper endoscopy and/or esophageal barium swallow;.
  3. Esophageal manometry.
How well did you know this?
1
Not at all
2
3
4
5
Perfectly
192
Q

What are the findings of the following studies in patients with achalasia :
- Chest x-ray.
- Esophageal barium swallow.
- Upper endoscopy.
- Esophageal manometry. ?

A
  • Esophageal barium swallow: supportive and/or confirmatory test:
    * Bird-beak sign: dilation of the proximal esophagus with stenosis of the gastroesophageal junction
    * Delayed barium emptying or barium retention
  • Upper endoscopy: to rule out pseudoachalasia
    * Usually normal.
    * May show retained food in esophagus or increased resistance of LES during passage with endoscope.
    * If malignancy is suspected, biopsy and endoscopic ultrasound are indicated.
  • Esophageal manometry: confirmatory test of choice
    * Peristalsis is absent or uncoordinated in the lower two-thirds of the esophagus.
    * Incomplete or absent LES relaxation
    * High LES resting pressure
    * No evidence of mechanical obstruction
  • Chest x-ray
    * Widened mediastinum
    * Air-fluid level on lateral view
    * Possible absence of gastric air bubble
How well did you know this?
1
Not at all
2
3
4
5
Perfectly
193
Q

What is the treatment of achalasia ?

A

If a low surgical risk:
- Pneumatic dilation:
Endoscope-guided graded dilation of the LES that tears the surrounding muscle fibers with the help of a balloon
- LES myotomy (Heller myotomy) (2nd choice).

If a high surgical risk:
Botulinum toxin injection in the LES

If other measures are unsuccessful:
- nitrates or calcium channel blockers.

How well did you know this?
1
Not at all
2
3
4
5
Perfectly
194
Q

What is the rare form of Down syndrome ?

A

Mosaic Down syndrome, or mosaicism.

How well did you know this?
1
Not at all
2
3
4
5
Perfectly
195
Q
  1. What is Osgood-Schlatter disease ?
  2. Clinical features ?
  3. Dx ?
  4. Tx ?
A
  1. It is an avascular necrosis thought to arise from overuse of the quadriceps muscle during periods of growth.
    • Progressive anterior knee pain that is worse with activity and is reproducible with extension against resistance.
    • Proximal tibial swelling.
    • X-ray:
      *Anterior soft tissue swelling.
      *Lifting of tubercle from the shaft
      *Irregularity or fragmentation of the tubercle
    • Ultrasound: soft tissue swelling.

4.
- Mostly conservative (rest, ice, NSAIDs ).
- Strengthening and stretching of the quadriceps muscle
- Generally resolves once full bone maturity is reached
- Surgical excision of intratendinous ossicles in severe cases.

How well did you know this?
1
Not at all
2
3
4
5
Perfectly
196
Q
  1. What is Iliotibial band syndrome (runner’s knee) ?
  2. Clinical features ?
  3. Tx ?
A
  1. Common overuse injury of the distal portion of the iliotibial band (over the lateral femoral epicondyle)
    Etiology: repetitive flexion and extension of the knee (e.g., during running).
    • Pain in the lateral knee (due to friction of iliotibial band against femoral epicondyle)
    • Sharp pain when the foot strikes the ground
    • Dull, constant pain at rest
    • Noble test: patient lies on their side and the examiner passively flexes the patient’s leg while exerting constant pressure on the lateral femoral epicondyle with his thumb → test is positive if pain is elicited.
    • Conservative treatment:
      • Rest
      • Cooling/ice for the first 24–48 hours
      • Topical/oral NSAIDs
      • Physiotherapy: stretching and strength training once the pain has subsided
      • Orthotic braces and bands
  • Corticosteroid and lidocaine injections:
    • Only considered if conservative treatment has failed
    • Short-term relief (no long-term benefit).
    • Injection into surrounding tissue, not directly into tendons.
  • Surgery :
    • Indicated in patients with persistent symptoms despite 6 months of conservative treatment.
    • Excision of abnormal tendon tissue.
    • Longitudinal incisions (tenotomies) to release scarred and fibrotic areas

***Corticosteroid injections are generally avoided in insertional tendinopathy since they may cause tendon rupture!

How well did you know this?
1
Not at all
2
3
4
5
Perfectly
197
Q

What are the common caustive agent of bronchiolitis?

A
  • RSV: Respiratory syncytial virus
  • rhinovirus,
  • human metapneumovirus (hMPV),
  • parainfluenza virus,
  • adenovirus,
  • coronavirus,
  • influenza virus
  • human bocavirus
How well did you know this?
1
Not at all
2
3
4
5
Perfectly
198
Q

What is the
1. transmission route
2. Clinical features
3. Tx
4. Complications

of Salmonella enterica ?

A
  1. Transmission: foodborne (poultry, raw eggs, and milk.
  2. Clinical features
    - Duration: 3–7 days
    - Fever (usually resolves within 2 days), chills, headaches, myalgia
    - Severe vomiting and inflammatory (watery-bloody) diarrhea.
  3. Treatment (antibiotic therapy in severe cases):
    - Fluoroquinolones (e.g., ciprofloxacin)
    - Alternative: TMP-SMX or cephalosporins (e.g., ceftriaxone), depending on the antimicrobial susceptibility test.
    - Antibiotic treatment prolongs fecal excretion of the pathogen; only indicated for systemic manifestations or diarrhea > 9/day.
  4. Complications: (especially in immunocompromised patients, e.g., HIV):
    - Bacteremia
    - Reactive arthritis
    - Systemic disease: osteomyelitis, meningitis, myocarditis
How well did you know this?
1
Not at all
2
3
4
5
Perfectly
199
Q

What is the
1. transmission route
2. Clinical features
3. Tx
4. Complications

of Shigellosis ?

A
  1. Transmission:
    - Fecal-oral (especially a concern in areas with poor sanitation)
    - Oral-anal sexual contact
    - Foodborne (unpasteurized milk products and raw, unwashed vegetables)
    - Contaminated water.
  2. Clinical features:
    - Duration: 2–7 days
    - High fever
    - Tenesmus, abdominal cramps
    - Profuse inflammatory, mucoid-bloody diarrhea.
  3. Treatment: in severe cases, antibiotic therapy with fluoroquinolones or 3rd generation cephalosporins.
  4. Complications:
    - HUS
    - Intestinal complications (e.g., toxic megacolon, colonic perforation, intestinal obstruction, proctitis, rectal prolapse) .
    - Febrile seizures
    - Reactive arthritis
How well did you know this?
1
Not at all
2
3
4
5
Perfectly
200
Q

What is the definition of short stature ?

A

It is a height that is at least two standard deviations (SDs) below the mean for children of the same age and sex.

How well did you know this?
1
Not at all
2
3
4
5
Perfectly
201
Q

How to differentiate between Familial short stature, Constitutional growth delay and Idiopathic short stature ?

A

Familial short stature:
- Hereditary short stature
- Most common cause of proportionate short stature.
- Normal development
- Skeletal age consistent with chronological age.

Constitutional growth delay :
- Inherited type of developmental delay
- Second most common cause of short stature
- Normal adult height
- Delayed skeletal age
- Delayed onset of puberty

Idiopathic short stature:
-Diagnosis of exclusion in the absence of an underlying condition

How well did you know this?
1
Not at all
2
3
4
5
Perfectly
202
Q

What is the peak incidence of Intussusception ?

A

3–12 months; otherwise commonly occurs in children 5 months to 6 years of age.

How well did you know this?
1
Not at all
2
3
4
5
Perfectly
203
Q

What are the casues of Intussusception ?

A

Mostly idiopathic (usually in children 3 months to 5 years of age)
∼ 75% of cases have no identifiable lead point
Disorganized peristalsis with enlarged Peyer’s patches may underlie idiopathic cases.

Pathological lead points: more likely the underlying cause in patients with recurrent episodes of intussusception; more common in children < 3 months or > 5 years of age.
- Meckel’s diverticulum (most common)
- Intestinal polyps or other tumors (2nd most common)
- Bowel wall thickening in Henoch-Schönlein purpura
- Recent rotavirus immunization
- Cystic fibrosis

How well did you know this?
1
Not at all
2
3
4
5
Perfectly
204
Q

How to classify Intussusception (according to site) ?

A
  • Ileocecal invagination (most common).
  • Ileoileal invagination
  • Ileocolic invagination
  • Colosigmoidal invagination
  • Appendicocecal invagination (very rare)
How well did you know this?
1
Not at all
2
3
4
5
Perfectly
205
Q

What are the clinical features of Intussusception ?

A

** Less than 15% of patients with intussusception present with the classic triad of abdominal pain, a palpable sausage-shaped abdominal mass, and blood per rectum!

  • Child typically looks healthy.
  • Acute cyclical colicky abdominal pain (sudden screaming or crying spells), often with legs drawn up, with asymptomatic intervals.
  • Acute attacks occur approx. every 15–30 min.
  • Vomiting (initially nonbilious)
  • Lethargy , pallor, and other symptoms of shock or altered mental status may be present.
  • Abdominal tenderness, palpable sausage-shaped mass in the RUQ , and an “emptiness” or retraction in the right lower quadrant (Dance sign) during palpation.
  • High-pitched bowel sounds on auscultation
  • “Currant jelly stool” (usually a late sign) may be noticed in passed stool or during digital rectal examination .
How well did you know this?
1
Not at all
2
3
4
5
Perfectly
206
Q

What is the best initial test for Dx Intussusception ?

A

Abdominal ultrasound (best initial test): often sufficient to confirm diagnosis :

  • Target sign (transverse view): The invaginated portion of bowel appears as rings on a target in transverse view on ultrasound.
  • Pseudokidney sign (longitudinal view): the lead point of the invagination in the distal loop of bowel resembles a kidney. This “pseudokidney” is made up of longitudinal layers of bowel wall.
  • Possible pendulous peristalsis
How well did you know this?
1
Not at all
2
3
4
5
Perfectly
207
Q

What is the best confirmatory test for Intussusception ?

A

Contrast or pneumatic enema using ultrasound or fluoroscopy (best confirmatory test).

Interruption of contrast or air at the site of invagination.

How well did you know this?
1
Not at all
2
3
4
5
Perfectly
208
Q

How to Tx Intussusception ?

A

Initial steps: nasogastric decompression and fluid resuscitation.

Nonsurgical management (performed under continuous ultrasound or fluoroscopic guidance):
- Air (pneumatic) enema: treatment of choice
- Hydrostatic reduction: normal saline (or water-soluble contrast enema)
- Observe for 24 hours post-reduction, as there is a small risk of perforation and recurrence is common during this period.

Surgical reduction
- Indications:
* When a pathological lead point is suspected
* Failed conservative management
* Suspected gangrenous or perforated bowel
* Critically ill patient (e.g., shock)
- Open or laparoscopic method:
* Hutchinson maneuver: manual proximal bowel compression and reduction of intussusception.
* For necrotic bowel segments: Resection and end-to-end anastomosis.

How well did you know this?
1
Not at all
2
3
4
5
Perfectly
209
Q

How to calculate incidence ?

A

New cases devided by total population at risk x 100

How well did you know this?
1
Not at all
2
3
4
5
Perfectly
210
Q

How to calculate Prevalence

A

New cases + old cases devided by total population at risk x 100

How well did you know this?
1
Not at all
2
3
4
5
Perfectly
211
Q
  1. What is the definition of Lactose intolerance ?
  2. What are the clinical features of Lactose intolerance ?
A
  1. Lactose intolerance is the inability to absorb lactose, caused by lactase deficiency.
  2. Clinical features:
    - Symptoms occur about an hour to several hours following consumption of milk products. The intensity of symptoms correlates with the amount of lactose consumed.
    - Diarrhea (often watery, bulky, and frothy)
    - Cramping abdominal pain (often periumbilical or in the lower abdomen)
    - Abdominal bloating
    - Flatulence
    - Nausea
How well did you know this?
1
Not at all
2
3
4
5
Perfectly
212
Q

What are the causes of Lactose intolerance ?

A
  • Primary (lactase non‑persistence): most common type of lactose intolerance; a decrease in lactase activity is primarily observed during childhood or adolescence.
  • Secondary (acquired): due to underlying disorders of the small intestine that result in mucosal damage (e.g., gluten‑sensitive enteropathy following gastroenteritis).
  • Developmental: occurs in children born prematurely, as lactase activity develops late during pregnancy.
  • Congenital: autosomal recessive gene defect (extremely rare).
How well did you know this?
1
Not at all
2
3
4
5
Perfectly
213
Q

Where does the lactose digestion occur ?

A

In the small intestine (particularly the jejunum).

How well did you know this?
1
Not at all
2
3
4
5
Perfectly
214
Q

How to Dx Lactose intolerance ?
How to Tx Lactose intolerance ? ?

A
  • Stool analysis: ↑ stool osmotic gap , stool pH < 6.
  • Hydrogen breath test:
    The amount of hydrogen in the expired air increases after administering lactose in the fasting state.
    Measurement at baseline and at 30‑minute intervals over 3 hours following the ingestion of 50 g of lactose (children: 2 g/kg lactose (max. amount 25 g)).
  • Lactose tolerance test: Following the administration of lactose, the normal rise in blood glucose levels is pathologically reduced and symptoms appear → rarely used, as the test has low sensitivity and specificity.
  • Trial lactose‑free diet: to see if symptoms resolve.
  • Biopsy of the small intestine: qualitative and quantitative assessment of lactase via an endoscopic tissue biopsy (conclusive, although rarely used). Histologic analysis shows normal intestinal architecture.

Tx:
Either by avoidance or Oral lactase supplements.

How well did you know this?
1
Not at all
2
3
4
5
Perfectly
215
Q

Name the most common viable numerical chromosomal aberrations.

A

Autosomal chromosomal aberrations:
- Trisomy 13 (Patau syndrome)
- Trisomy 18 (Edwards syndrome)
- Trisomy 21 (Down syndrome)

Gonosomal chromosomal aberrations
- Klinefelter syndrome
- Turner syndrome

How well did you know this?
1
Not at all
2
3
4
5
Perfectly
216
Q

What are the clinical features of Trisomy 13 (Patau syndrome) ?

Dx and Prognosis ?

A
  • Microcephaly, holoprosencephaly
  • Characteristic facial anomalies:
    • Cleft lip and palate
    • Low-set, malformed ears
    • Bulbous nose
    • Small chin
    • Eyes: microphthalmia (small orbits, which may be unilateral or bilateral), possibly coloboma , ocular hypotelorism.
  • Polydactyly, primarily hexadactyly, flexed fingers
  • Congenital heart defects (particularly ventricular septal defect, patent ductus arteriosis)
  • Rocker-bottom feet
  • Visceral and genital anomalies, especially of the kidneys and ureters (e.g., polycystic kidney disease)
  • Aplasia cutis congenita: congenital absence of skin; most commonly scalp lesions with a punched-out appearance that may extend to the bone or the dura.
  • Omphalocele
  • Capillary hemangioma

Diagnosis :
- Usually detected during first trimester screening with combined ultrasound and maternal serum testing (↓↓ PAPP-A, ↑ nuchal translucency).

Prognosis:
Infants usually die before the age of 1, only approx. 11% of infants survive past 12 months of age.

How well did you know this?
1
Not at all
2
3
4
5
Perfectly
217
Q

What are the clinical features of Trisomy 18 (Edwards syndrome)?

Dx and Prognosis ?

A
  • Characteristic facial anomalies:
    • Prominent occiput
    • Microcephaly
    • Broad nose
    • Low-set ears (malformed auricle)
    • Micrognathia (congenital mandibular hypoplasia)
    • Cleft lip and palate, high palate
  • Clenched fists with flexion contractures and overlapping fingers
  • Rocker-bottom feet: convex deformity of the plantar side of the foot, with a vertical talus, and prominent calcaneus
  • Congenital heart defects (particularly VSD, ASD, tetralogy of Fallot)
  • Malformations of internal organs: diaphragmatic hernia, ureter, and kidneys (horseshoe kidneys).
  • Myelomeningocele
  • Omphalocele
  • Severe intellectual disability

Diagnosis
- Quadruple test in the second trimester shows
* ↓ free estriol,
* ↓ AFP,
* ↓ Inhibin A
* ↓ β-HCG

Prognosis:
Only approx.13% of patients survive past 12 months of age.

How well did you know this?
1
Not at all
2
3
4
5
Perfectly
218
Q

What is the most common pathogen of Pelvic inflammatory disease (PID) ?

A
  • Chlamydia trachomatis
  • Neisseria gonorrhoeae

***Less common (consider coinfections):
- E. coli
- Ureaplasma
- Mycoplasma
- other anaerobes

How well did you know this?
1
Not at all
2
3
4
5
Perfectly
219
Q

What are the risk factors of Pelvic inflammatory disease (PID) ?

A
  • Multiple sexual partners, unprotected sex
  • History of prior STIs and/or adnexitis
  • Intrauterine devices.
  • Vaginal dysbiosis (bacterial vaginosis).
  • Risk is lower during pregnancy; PID development during pregnancy increases the risk of maternal morbidity and preterm births.
How well did you know this?
1
Not at all
2
3
4
5
Perfectly
220
Q

What are the clinical features of Pelvic inflammatory disease (PID) ?

A
  • Lower abdominal pain (generally bilateral), which may progress to acute abdomen.
  • Nausea, vomiting
  • Fever
  • Dysuria, urinary urgency
  • Menorrhagia, metrorrhagia
  • Dyspareunia
  • Abnormal vaginal discharge
How well did you know this?
1
Not at all
2
3
4
5
Perfectly
221
Q

What are the investigations ordered in Pelvic inflammatory disease (PID) ?

A
  • History:
    • Most often a sexually active young woman.
    • Lower abdominal pain.
  • PEx: Vaginal examination:
    • Cervical motion tenderness: severe cervical pain elicited by pelvic examination.
    • Uterine and/or adnexal tenderness
    • Purulent, bloody cervical and/or vaginal discharge.
  • Blood tests: elevated ESR, leukocytosis.
  • Pregnancy test: to rule out an (ectopic) pregnancy
  • Cervical and urethral swab:
    • Gonococcal and chlamydial DNA (PCR) and cultures.
    • Giemsa stain of discharge can show cytoplasmic inclusions in C. trachomatis infections, but not in N. gonorrhoeae infection.
  • Imaging
    • Ultrasound: free fluid, abscesses, pyosalpinx/hydrosalpinx
  • Exploratory laparoscopy:
    Indicated in ambiguous cases and if patient does not respond to treatment.
    Characteristic findings include tubal edema, erythema, and purulent exudate.
  • Endometrial biopsy: to confirm the presence of endometritis.
  • Culdocentesis: aspiration of intraperitoneal fluid from the pouch of Douglas(no longer a routine procedure ,replaced by ultrasound).
How well did you know this?
1
Not at all
2
3
4
5
Perfectly
222
Q

What is the Tx of Pelvic inflammatory disease (PID) ?

A

Outpatient regimen:
- One single dose of IM ceftriaxone and oral therapy with doxycycline.
- If signs of vaginitis or recent gynecological instrumentation add oral metronidazole.

Inpatient regimen (parenteral antibiotics):
- Indications:
* No response to or inability to take outpatient oral regimen
* Non-compliance concerns (e.g., teenagers)
*High fever

  • Possible combinations (should be administered for 14 days)
  • Cefoxitin or cefotetan plus doxycycline.
  • Clindamycin plus gentamicin
  • Switch to oral therapy with doxycycline after clinical improvement.
How well did you know this?
1
Not at all
2
3
4
5
Perfectly
223
Q

What are the complications of Pelvic inflammatory disease (PID) ?

A

Short-term complications:
- Pelvic peritonitis
- Fitz-Hugh-Curtis syndrome (perihepatitis):
* Inflammation of the liver capsule
*Characterized by violin-string-like adhesions extending from the peritoneum to the liver
- Tubo-ovarian abscess.
* A confined pus collection of the uterine adnexa
* May spread to adjacent organs (e.g., bladder, bowel)

Long-term complications:
- Infertility: caused by adnexitis, adhesions of the fallopian tubes and ovaries, and tubal scarring, which result in impaired ciliary function and tubal occlusion
- Ectopic pregnancy
- Chronic pelvic pain
- Hydrosalpinx/pyosalpinx: accumulation of fluid/puss in the fallopian tubes due to chronic inflammation and consequent stenosis.
- Chronic salpingitis

How well did you know this?
1
Not at all
2
3
4
5
Perfectly
224
Q

Hirschsprung’s disease (congenital aganglionic megacolon) common site?

A

rectosigmoid region.

How well did you know this?
1
Not at all
2
3
4
5
Perfectly
225
Q

What is the Pathophysiology of Hirschsprung’s disease (congenital aganglionic megacolon) ?

A

-It is caused by defective caudal migration of parasympathetic neuroblasts (precursors of ganglion cells) from the neural crest to the distal colon.

  • Affected segments are histologically characterized by the absence of the Meissner plexus and Auerbach plexus (submucosal and myenteric plexus ganglion) beginning at the anorectal line, leading to::
    • Inability of the myenteric plexus to control the intestinal wall muscles → uncoordinated peristalsis and slowed motility.
    • Spastic contraction of intestinal muscles → stenosis
    • Expansion of the colon segment proximal to the aganglionic section (possible megacolon).
How well did you know this?
1
Not at all
2
3
4
5
Perfectly
226
Q

What are the clinical features of Hirschsprung’s disease (congenital aganglionic megacolon) ?

A

Early presentation:
- Delayed passage of meconium.
- Distal intestinal obstruction: abdominal distention and bilious vomiting.
- Tight anal sphincter with explosive release of stools and air upon removal of the finger.
- Failure to thrive/poor feeding.
- Palpation of feces via the abdominal wall.

Late presentation
- Chronic constipation with possible inability to pass gas

How well did you know this?
1
Not at all
2
3
4
5
Perfectly
227
Q

What is the gold standard confirmatory test for Hirschsprung’s disease (congenital aganglionic megacolon) ?

A

rectal biopsy is the gold standard for confirming the diagnosis

How well did you know this?
1
Not at all
2
3
4
5
Perfectly
228
Q

What is the initial test for Hirschsprung’s disease (congenital aganglionic megacolon) ?

A

Abdominal x-ray.

How well did you know this?
1
Not at all
2
3
4
5
Perfectly
229
Q

What are the diagnostic test and thier findings of Hirschsprung’s disease (congenital aganglionic megacolon) ?

A

Abdominal x-ray
- Best initial test
- Findings:
* Decreased or absent air in rectum.
* Dilated colon segment immediately proximal to aganglionic region.
* Distal intestinal obstruction.

Barium enema
- Indication: Usually performed in addition to x-ray, to localize and determine the length of the aganglionic segment prior to surgery.
- Findings:
* Change in caliber along the affected intestinal segment .
* Retention of barium for 24–48 hours.

Anorectal manometry
- Findings:
* Absent relaxation reflex of the internal sphincter after stretching of the rectum (paradoxical increase in pressure).
* Typical ‘mass contractions’: lack of physiological, propulsive waves.

Rectal biopsy
- Confirmatory test
- Procedure:
* Full-thickness biopsy under general anesthesia
* Bedside suction biopsy
- Findings
* Absence of ganglion cells in an adequate tissue sample.
* Elevated acetylcholinesterase activity
* Hyperplasia of the parasympathetic nerve fibers
* Absent calretinin immunostained fibers.

How well did you know this?
1
Not at all
2
3
4
5
Perfectly
230
Q

What is the treatment of Hirschsprung’s disease (congenital aganglionic megacolon) ?

A

Surgical treatment is usually performed in two stages:
- First stage: diverting colostomy to relieve the dilated bowel .
- Second stage:
* Resection of the aganglionic segment
* Anastomoses of the normal ganglionic colon segment to either the distal modified rectum or normal (unmodified) distal rectum.
* Preservation of internal anal sphincter function is of the atmost importance.

How well did you know this?
1
Not at all
2
3
4
5
Perfectly
231
Q

Preterm labor definition ?

A

Preterm labor: Regular uterine contractions with cervical effacement, dilation, or both before 37 weeks gestation.

How well did you know this?
1
Not at all
2
3
4
5
Perfectly
232
Q

What are the risk factors of preterm labor ?

A

High risk factors:
- History of preterm birth
- Cervical insufficiency
- Multiple gestation

Low risk factors:
- Maternal and fetal medical conditions:
* Infections
* Polyhydramnios
* Malaria
* Hypertensive pregnancy disorders
* Diabetes mellitus, gestational diabetes
* Uterine anomalies (e.g., anomalies of Mullerian duct fusion, uterine fibroids)
* Placenta previa
* Placental abruption
* Congenital abnormalities of the fetus

  • Lifestyle and environmental factors:
    • Smoking
    • Substance use (e.g., alcohol or drugs)
    • Maternal or fetal stress
    • Maternal age (≤ 18 years, > 35 years)
    • Low maternal pre-pregnancy weight
    • African-american race
How well did you know this?
1
Not at all
2
3
4
5
Perfectly
233
Q

What are the clinical features of preterm labor ?

A
  • Regular uterine contractions and associated symptoms of labor .
  • Cervical dilation ≥ 3 cm, effacement, or both.
  • Premature rupture of membranes.
How well did you know this?
1
Not at all
2
3
4
5
Perfectly
234
Q

How to Dx preterm labor ?

A
  • Clinical diagnosis based on preterm contractions and cervical changes.
  • Supportive tests:
    • Transvaginal cervical ultrasound: for diagnosis of short cervix
    • Cervicovaginal fetal fibronectin detection test: a positive test supports the diagnosis of preterm labor
How well did you know this?
1
Not at all
2
3
4
5
Perfectly
235
Q

How to Tx preterm labor ?

A
  1. Single course of antenatal steroids (IM betamethasone or IM dexamethasone ):
    - Indication: 24–34 weeks gestation with a risk of delivery within the next 7 days .
    - Improves fetal lung maturity and surfactant production.
    - Repeat the course if the last dose of corticosteroids was > 14 days previously.

2.Tocolysis: administration of tocolytics to inhibit uterine contractions and prolong pregnancy:
- Recommended for up to 48 hours to enable administration of antenatal corticosteroids in preterm labor .
- First-line: beta-adrenergic agonists, NSAIDs, or calcium-channel blockers.
- Second-line: Magnesium sulfate
- Contraindications:
* Maternal drug contraindications
* Nonreassuring fetal CTG
* Intrauterine fetal demise
* Chorioamnionitis
* Antepartum hemorrhage with hemodynamic instability
* Severe pre-eclampsia or eclampsia
* Lethal fetal anomaly

  1. Fetal neuroprotection: Magnesium sulfate
    Indication: if birth < 32 weeks is anticipated.

*** Antibiotics for group B streptococcus (GBS) prophylaxis is recommended in preterm labor, preterm premature rupture of membranes and when GBS infection is evident.

How well did you know this?
1
Not at all
2
3
4
5
Perfectly
236
Q

What are the complications of preterm labor ?

A
  1. Neonatal respiratory distress syndrome (RDS)
  2. Bronchopulmonary dysplasia (BPD)
  3. Patent ductus arteriosus (PDA)
  4. Retinopathy of prematurity (ROP)
  5. Necrotizing enterocolitis (NEC)
  6. Periventricular leukomalacia (PVL).
  7. Neurological disorders (e.g., cerebral palsy, learning disabilities, developmental delays, ADHD).
  8. Problems of homeostasis: apnea, bradycardia, hypothermia.
  9. Infection and sepsis (e.g., pneumonia) .
  10. Anemia of prematurity: impaired ability to produce adequate erythropoietin (EPO).
  11. Intraventricular hemorrhage (IVH) .
How well did you know this?
1
Not at all
2
3
4
5
Perfectly
237
Q

What are the risk factors of Premature rupture of membranes (PROM) ?

A
  • Ascending infection (common)
  • Cigarette smoking
  • Multiple pregnancy
  • Previous preterm delivery
  • Previous PROM
How well did you know this?
1
Not at all
2
3
4
5
Perfectly
238
Q

What are the complications of Premature rupture of membranes (PROM) ?

A
  • Umbilical cord prolapse
  • Placental abruption
  • Chorioamnionitis
How well did you know this?
1
Not at all
2
3
4
5
Perfectly
239
Q

When to perform Cervical cerclage in Cervical insufficiency ?

A

< 24 weeks gestation; most commonly performed at 13–16 weeks gestation

How well did you know this?
1
Not at all
2
3
4
5
Perfectly
240
Q

What is the quadruple test?

A

Second trimester quadruple test (15–18 weeks) to Dx down syndrome :
- ↓ Free estriol
- ↓ Alpha-fetoprotein (AFP)
- ↑ Inhibin A
- ↑ β-hCG

How well did you know this?
1
Not at all
2
3
4
5
Perfectly
241
Q

Can a HIV infected mother breastfeed her baby ?

A

to prevent HIV transmission, HIV-infected mothers should not breastfeed their infants. HIV is a virus that attacks the body’s immune system and is spread through certain body fluids, including breast milk. Mother-to-child transmission can occur during pregnancy, birth, or breastfeeding.

How well did you know this?
1
Not at all
2
3
4
5
Perfectly
242
Q

The crown-rump length (CRL) is a measurement of the embryo, usually identified at which weeks of gestation ?

A

7 and 13 weeks’ gestation.

How well did you know this?
1
Not at all
2
3
4
5
Perfectly
243
Q

Head circumference is measured at which weeks of gestation ?

A

13–25 completed weeks

How well did you know this?
1
Not at all
2
3
4
5
Perfectly
244
Q

Femur length is measured at which weeks of gestation ?

A

13–25 completed weeks

How well did you know this?
1
Not at all
2
3
4
5
Perfectly
245
Q

How to calculate the expected date of delivery ?

A
  1. First, determine the first day of your last menstrual period.
  2. Next, count back 3 calendar months from that date.
  3. Lastly, add 1 year and 7 days to that date.
How well did you know this?
1
Not at all
2
3
4
5
Perfectly
246
Q

When does Vitamin K deficiency bleeding of the newborn (VKDB) usually occur ?

A
  • Early-onset (within 24 hours after birth).
  • Classic (within 4 weeks).
  • Late-onset (between 2–8 months).
How well did you know this?
1
Not at all
2
3
4
5
Perfectly
247
Q

Lab results in patints with Vitamin K deficiency bleeding of the newborn (VKDB)

A

Coagulation studies:
- ↑ Prothrombin time (PT)
- Normal or ↑ activated partial thromboplastin time (PTT)
- Normal bleeding time
- ↓ Factors II, VII, IX, and X

How well did you know this?
1
Not at all
2
3
4
5
Perfectly
248
Q

What is the treatment of Vitamin K deficiency bleeding of the newborn (VKDB ?

A
  • Transfusions as necessary
  • Administration of vitamin K
How well did you know this?
1
Not at all
2
3
4
5
Perfectly
249
Q

How to estimate a child’s adult height ?

A
  • Add the mother’s height and the father’s height in either inches or centimeters.
  • Add 5 inches (13 centimeters) for boys or subtract 5 inches (13 centimeters) for girls.
  • Divide by two.
How well did you know this?
1
Not at all
2
3
4
5
Perfectly
250
Q

What is Wiskott-Aldrich Syndrome ?

A

It is an X-linked disorder characterized by the clinical triad of microthrombocytopenia, eczema, and recurrent infections.

How well did you know this?
1
Not at all
2
3
4
5
Perfectly
251
Q

What is Kawasaki disease ?

A

It is an acute, necrotizing vasculitis of unknown etiology.Usually occur in patient who are 5 years of age or younger.

How well did you know this?
1
Not at all
2
3
4
5
Perfectly
252
Q

What is the clinical features of Kawasaki disease ?

A

Clinical diagnosis requires fever for at least 5 days and one of the following:
* ≥ 4 other specific symptoms
* < 4 specific symptoms and involvement of the coronary arteries.

  • Specific symptoms include:
    • Erythema and edema of hands and feet, including the palms and soles (the first week).
    • Possible desquamation of fingertips and toes after 2–3 weeks.
    • Polymorphous rash, originating on the trunk
    • Painless bilateral “injected” conjunctivitis without exudate.
    • Oropharyngeal mucositis:
      • Erythema and swelling of the tongue (strawberry tongue).
        - Cracked and red lips.
        • Cervical lymphadenopathy (mostly unilateral).
  • Nonspecific symptoms may precede the onset of Kawasaki disease (e.g., diarrhea, fatigue, abdominal pain)
How well did you know this?
1
Not at all
2
3
4
5
Perfectly
253
Q

What labs and other investigation used to evlauate Kawasaki disease and how are the results ?

A
  • Laboratory findings
    • ↑ ESR and CRP
    • Leukocytosis
    • Thrombocytosis
  • Echocardiography:
    • For evaluating coronary artery aneurysms
    • Minimal evaluation: should be performed at diagnosis, at 2 weeks, and at 6–8 weeks after onset .
How well did you know this?
1
Not at all
2
3
4
5
Perfectly
254
Q

How to treat Kawasaki disease ?

A
  • IV immunoglobulin (IVIG)
    • High single-dose to reduce the risk of coronary artery aneurysms.
    • Most effective if given within the first 10 days following disease onset.
  • High-dose oral aspirin
  • IV glucocorticoids: may be considered in addition to standard treatment, esp. in cases of treatment-refractory disease, as they lower the risk of coronary involvement

*** To avoid the risk of Reye syndrome, children should not be treated with aspirin, especially if a viral infection is suspected. Kawasaki disease is an exception to this rule.

How well did you know this?
1
Not at all
2
3
4
5
Perfectly
255
Q

What are the clinical features of Meckel diverticulum ?

A

Asymptomatic (∼ 96%).
Symptomatic (2–4%):
- Painless lower gastrointestinal bleeding (most common presentation)
- Hematochezia
- Tarry stools
- Currant jelly stools

How well did you know this?
1
Not at all
2
3
4
5
Perfectly
256
Q

How to Dx Meckel diverticulum ?

A
  • Same protocol as that for lower gastrointestinal bleeding and/or acute abdomen +
  • Meckel scintigraphy scan (Meckel scan): a noninvasive nuclear medicine imaging technique using radiolabelled technetium (99mTc), which is preferentially absorbed by the gastric mucosa and can identify ectopic gastric mucosa.
  • CT angiography: may demonstrate the vitelline artery or even contrast extravasation from a bleeding Meckel diverticulum.
  • Double balloon enteroscopy: A long endoscope is advanced, either through the mouth or the rectum, into the small intestine. The sequential inflation and deflation of the two balloons helps advance the scope into the intestine by pleating the bowel onto the scope.
  • Capsule endoscopy: The patient is asked to swallow a small, encapsulated camera, which takes multiple images of the bowel as it passes through the gastrointestinal tract.
  • Diagnostic laparoscopy
How well did you know this?
1
Not at all
2
3
4
5
Perfectly
257
Q

What is the Tx of Meckel diverticulum ?

A

Asymptomatic Meckel diverticulum:
- Incidentally detected on imaging studies: no treatment necessary
- Incidentally detected on laparotomy/laparoscopy:
* Children or young adults: surgical resection of all incidentally detected Meckel diverticuli.
* Adults < 50 years: surgical resection only for Meckel diverticuli that have a high risk of developing complications.
* Adults > 50 years: no treatment necessary.

  • Symptomatic or complicated Meckel diverticulum:
    • Initial stabilization of the patient
    • Surgical resection of all symptomatic/complicated Meckel diverticuli
  • Surgical procedures:
    • Segmental resection : Indicated for a Meckel diverticulum that is bleeding, has a broad base, or a palpable abnormality.
    • Diverticulectomy: Meckel diverticulum is resected at the base.
How well did you know this?
1
Not at all
2
3
4
5
Perfectly
258
Q

What are the major classes of antihypertensive agents?

A
  1. Diuretics
  2. Beta- blockers
  3. ACEIs (Angiotensin-converting-enzyme inhibitors)
  4. ARBs (Angiotensin II receptor blockers)
  5. CCBs (Calcium channel Blockers)
  6. Vasodilators
  7. Alpha 1 adrenergic blockers
  8. Centrally acting adrenergic agonist.
How well did you know this?
1
Not at all
2
3
4
5
Perfectly
259
Q

What are the
1. Types
2. Mechanism of action.
3. Side effects.

Of diuretics ?

A
  1. Thiazide - loop (Furosemide) - K sparing (spironolactone).
  2. Decreases extracellular fluid volume and thereby decreases vascular resistance.
  3. Hypokalemia (not with K sparing) - hyperglycemia - hyperlipidemia- hyperurecemia - azotemia.
How well did you know this?
1
Not at all
2
3
4
5
Perfectly
260
Q

What are the
1. Types
2. Mechanism of action.
3. Side effects.

Of beta- blockers ?

A
  1. Propranolol - Metoprolol - Nadolol- Carvedilol
  2. Decrease cardiac contractility and renin release
  3. Bronchospasm in asthma - bradycardia - CHF exacerbation - impotence - fatigue - depression.
How well did you know this?
1
Not at all
2
3
4
5
Perfectly
261
Q

What are the
1. Types
2. Mechanism of action.
3. Side effects.

Of ACEIs ?

A
  1. Captopril - Enalopril - Benazepril
  2. Block aldosterone formation , decreases peripheral resistance and salt/water retention
  3. Cough - Rash - leukopenia - hyperkalemia
How well did you know this?
1
Not at all
2
3
4
5
Perfectly
262
Q

What are the
1. Types
2. Mechanism of action.
3. Side effects.

Of ARBs ?

A
  1. Losartan - Valsartan
  2. Block aldosterone effect, decreases peripheral resistance and salt/water retention
  3. Rash - leukopenia - hyperkalemia
How well did you know this?
1
Not at all
2
3
4
5
Perfectly
263
Q

What are the
1. Types
2. Mechanism of action.
3. Side effects.

Of CCBs ?

A
  1. Dihydropyridines ( Nifedipine- Felodipine- Amlodipine) Nondigydropyridines ( Diltiazem - verapmil).
  2. Decreases smooth muscle tone => vasodilation , decreases cardiac output.
  3. Dihydropyridines: headache, flushing, peripheral edema.

Nondigydropyridines: decreases contractility.

How well did you know this?
1
Not at all
2
3
4
5
Perfectly
264
Q

What are the
1. Types
2. Mechanism of action.
3. Side effects.

Of Vasodilators ?

A
  1. Hydralazine - Minoxidil
  2. Decreases peripheral resistance by dilating arteries/ arterioles
  3. Hydralazine : headache - lupus like syndrome.
    Monoxidil: orthostasis - hirsutiam
How well did you know this?
1
Not at all
2
3
4
5
Perfectly
265
Q

Which tendon makes patient unable to stand on his toe if ruptured?

A

Achilles / Calcaneal tendon

How well did you know this?
1
Not at all
2
3
4
5
Perfectly
266
Q

What is galactosemia and what is the type of inheritance ?

A

Galactosemia, which means “galactose in the blood,” refers to a group of inherited disorders that impair the body’s ability to process and produce energy from a sugar called galactose. When people with galactosemia injest foods or liquids containing galactose, undigested sugars build up in the blood.

Type of inheritance: Autosomal recessive

How well did you know this?
1
Not at all
2
3
4
5
Perfectly
267
Q

What is pheochromocytoma ? And what is the management?

A

Its a catecholamine secreting tumor that causes life threatening HTN.

Mx:
- Alpha blockers (Phenoxybenzamine)
- Beta blockers
- surgical removal.

How well did you know this?
1
Not at all
2
3
4
5
Perfectly
268
Q

Name the cranial nerves that arise from the jugular foramen ?

A

IX Glossopharyngeal nerve
X Vagus nerve
XI Accessory nerve

How well did you know this?
1
Not at all
2
3
4
5
Perfectly
269
Q

What are the side effects of Haloperidol?

A
  • involuntary movement: Akathisia, dystonia , Parkinsonism
  • wight gain
  • erectile dysfunction
  • Amenorrhea
How well did you know this?
1
Not at all
2
3
4
5
Perfectly
270
Q

What are the muscles supplied by tibial nerve ?

A

2 T, 2 P, 2 F
- Triceps Surae
- Tibialis posterior
- Plantaris
- Popliteus
- Flexor digitorum longus
- Flexor hallicus longus

How well did you know this?
1
Not at all
2
3
4
5
Perfectly
271
Q

What is Pericardial tamponade (Becks Triad) ?

A
  • Distant (Muffled) heart sound
  • Decreased arterial BP
  • Distended jugular veins.
How well did you know this?
1
Not at all
2
3
4
5
Perfectly
272
Q

Can mothers with HCV and HBV breastfeed ?

A

Yes

How well did you know this?
1
Not at all
2
3
4
5
Perfectly
273
Q

What is the equation of sensitivity?

A

True Positive/ True positive + False negative.

( have disease)

How well did you know this?
1
Not at all
2
3
4
5
Perfectly
274
Q

What is the equation of Specificity ?

A

True Negative/ True negative + False positive

(Don’t have disease)

How well did you know this?
1
Not at all
2
3
4
5
Perfectly
275
Q

What are the vaccines that are contraindicated in pregnancy?

A
  • Measles
  • Mumps
  • Rubella
  • Zoster
  • Varicella
  • Typhoid
  • Influenza “Live attenuated)
  • BCG: Bacille calmette- Guerin
  • smallpox (pre-exposure)
How well did you know this?
1
Not at all
2
3
4
5
Perfectly
276
Q

Total body surface area rule of 9s?

A
  • Head and neck 9%
  • Anterior Trunk 18%
  • posterior trunk 18%
  • Arm 9%
  • leg 18%
  • Genitalia 1%
How well did you know this?
1
Not at all
2
3
4
5
Perfectly
277
Q

Parkland formula in burns ?

A

Total fluid requirements in 24 hours:

4 mL x TBSA (%) x body weight (Kg).

  • 50 % given in first 8 hours
  • 50 % given in next 16 hours.
How well did you know this?
1
Not at all
2
3
4
5
Perfectly
278
Q

How to diagnose bacterial vaginosis?

A

By Amsel criteria ( 3/4):

  1. Demonstration of clue cells on a saline smear.
  2. A pH > 4.5
  3. Thin, grey and homogeneous discharge
  4. Positive whiff test ( add potassium hydroxide to a slide that contains vaginal discharge, positive when fishy odor occurs).
How well did you know this?
1
Not at all
2
3
4
5
Perfectly
279
Q

What is the treatment of bacterial vaginosis ?

A

Metronidazole or Clindamycin

How well did you know this?
1
Not at all
2
3
4
5
Perfectly
280
Q

How to diagnose subclinical hypothyroidism by labs and what is the management?

A

TSH : high
T4 or Free Thyroxine Index FTI : Normal

Start levothyroxine 75-125 ug if TSH > 10 uU

How well did you know this?
1
Not at all
2
3
4
5
Perfectly
281
Q

What are the signs and symptoms of myxedema coma ?

A
  • Altered mental status
  • hypothermia
  • Bradycardia
  • Hyponatremia
  • Hypercarbia
  • cardiomegaly, pericardial effusion, cardiogenic shock
  • Ascites
How well did you know this?
1
Not at all
2
3
4
5
Perfectly
282
Q

How to diagnose hypothyroidism by labs ?

A

TSH : high
T4: low
Free thyroxine index FTI : low

How well did you know this?
1
Not at all
2
3
4
5
Perfectly
283
Q

When to screen for Thyroid function ?

A
  • Pregnancy
  • Female > 60 years old
  • DM type 1
  • any autoimmune disease
  • History of neck radiation
How well did you know this?
1
Not at all
2
3
4
5
Perfectly
284
Q

Name the medications that needs serum TSH re-assessment when started ?

A
  • Estrogen
  • Androgen
  • Tyrosine Kinase Inhibitors
  • Phenobarbitals
  • Phenytoin
  • Carbamazepine
  • Rifampin
  • Sertraline
How well did you know this?
1
Not at all
2
3
4
5
Perfectly
285
Q

How to treat myxedema coma ?

A
  • IV levothyroxine 200 - 250 ug bolus
  • after 24 hours —> levothyroxine 100 ug IV
  • then 50 ug /day IV
  • Give stress dose of IV glucocorticoids
How well did you know this?
1
Not at all
2
3
4
5
Perfectly
286
Q

How to calculate pediatric fluid maintenance ?

A
  • 1st 10 kg : 100
  • 2nd 10 Kg: 50
  • rest 10 kg : 25

Divid total by 24 hours to get per hour

How well did you know this?
1
Not at all
2
3
4
5
Perfectly
287
Q

Labs in tumor lysis syndrome

A

High :
- Urea
- K
- P

Low : Calcium

+ Acute Kidney Injury

How well did you know this?
1
Not at all
2
3
4
5
Perfectly
288
Q

Reiter’s syndrome / Reactive arthritis classic triad ?

A
  • Conjunctivitis
  • Urethritis
  • Arthritis

Usually occur after infection ( urogenital / GI infections)

How well did you know this?
1
Not at all
2
3
4
5
Perfectly
289
Q

What are the relative indications for surgery in primary hyperparathyroidism ?

A
  1. Age < 50 years
  2. Marked decrease in bone mass
    3.Nephrolithiasis , renal insufficiency
  3. Markedly elevated serum Ca or episodes of severe hypercalcemia
  4. Urine Ca > 400 mg in 24 hours
How well did you know this?
1
Not at all
2
3
4
5
Perfectly
290
Q

What are the common triggers of C.difficile infection?

A
  • Abx: Clindamycin- Cephalosporins
  • Gastric acid suppression: PPIs
How well did you know this?
1
Not at all
2
3
4
5
Perfectly
291
Q

How to differentiate between Nephrotic syndrome and Nephritic syndrome?

A

Nephrotic:
- Proteinuria > 3.5 g / 24 hours
- Hypoalbuminemia ( serum Albumin < / = 30 g/L
- Edema
- hyperlipidemia

Nephritic syndrome:
- Hematuria
- Red cell casts
-Proteinuria “small amount”
- HTN
- Oliguria

How well did you know this?
1
Not at all
2
3
4
5
Perfectly
292
Q

How to diagnose HTN in pediatrics ?

A

When BP in 3 separate visits is > 95th percentile

How well did you know this?
1
Not at all
2
3
4
5
Perfectly
293
Q

What are most common area of metastasis in prostate cancer ?

A
  • Lymph nodes
  • Bone
How well did you know this?
1
Not at all
2
3
4
5
Perfectly
294
Q

What is the management of medullary thyroid cancer ?

A

Total thyroidectomy and lymph node dissection

How well did you know this?
1
Not at all
2
3
4
5
Perfectly
295
Q

What is the mode of inheritance in Neurofibromas?

A

Autosomal dominant

How well did you know this?
1
Not at all
2
3
4
5
Perfectly
296
Q

What are the most common pathogens in meningitis according to age ?

A

0-6 months : GBS , E.coli , Listeria

6 months - 6 years : S.pneumonia , N.meningitidis , H.influenza

6 - 60 years: N.meningitidis, S.pneumonia , enterovirus, HSV

> 60 years : S.pneumonia , listeria , N.meningitidis.

How well did you know this?
1
Not at all
2
3
4
5
Perfectly
297
Q

What is the treatment of Toxoplasmosis in HIV patients?

A

Cotrimoxazole for 6 months

Alternative: Dapsone + Pyrimethamine

How well did you know this?
1
Not at all
2
3
4
5
Perfectly
298
Q

When to perform sweat chloride test for cystic fibrosis ?

A

If:
- siblings or parents have CF
- A family member is a known carrier of CF gene
- your partner is a carrier.

How well did you know this?
1
Not at all
2
3
4
5
Perfectly
299
Q

DKA biochemically ?

A
  • Serum ketones > 5 meq/ L
  • blood sugar > 250 mg/dl
  • Arterial pH < 7.3
  • Serum HCO3 =< 18
  • Increased anion gap
  • increased serum osmolarity
  • Increased uric acid
How well did you know this?
1
Not at all
2
3
4
5
Perfectly
300
Q

What are the signs of poor prognosis in DKA ?

A
  • Deep coma
  • Oliguria
  • Hypothermia
How well did you know this?
1
Not at all
2
3
4
5
Perfectly
301
Q

What are the most common causes of mortality in DKA?

A
  1. Cerebral edema (results from rapid intracellular fluid shifts + most common).
  2. Severe hypokalemia
  3. Adult respiratory distress syndrome
  4. Comorbid states
How well did you know this?
1
Not at all
2
3
4
5
Perfectly
302
Q

What are the complications associated with DKA ?

A
  • Sepsis
  • Diffuse ischemic process
  • MI
  • DVT
  • acute gastric dilation
  • Erosive gastritis
  • Late hypoglycemia
  • Respiratory distress
  • Infections
  • hypophosphatemia
  • CVA
How well did you know this?
1
Not at all
2
3
4
5
Perfectly
303
Q

How does the labs in DKA patients should be scheduled?

A

Glucose : Q 1-2 hours until stable, then Q4-6 hours

Serum electrolytes: Q1-2 hours until stable, then Q 4-6 hours

How well did you know this?
1
Not at all
2
3
4
5
Perfectly
304
Q

In DKA , which ketone is detected in urine dipstick ?

A

Acetoacetate

How well did you know this?
1
Not at all
2
3
4
5
Perfectly
305
Q

Which ketone in predominant in severe untreated DKA ?

A

Beta - hydroxybutyrate

How well did you know this?
1
Not at all
2
3
4
5
Perfectly
306
Q

What is the plasma osmolarity equation ?

A

Plasma osmolarity = 2 (Na + K ) + BUN/3 + Glucose/18

How well did you know this?
1
Not at all
2
3
4
5
Perfectly
307
Q

What is the action of Insulin ?

A
  1. Suppresses hepatic glucose output.
  2. Enhamces glucose uptake by peripheral tissue.
  3. Suppresses ketogenesis and lipolysis.
  4. Stimulates proper use of glucose by the cells.
  5. Reduces blood sugar levels.
How well did you know this?
1
Not at all
2
3
4
5
Perfectly
308
Q

What are the side effects of Corticosteroids?

A
  1. Osteoprosis.
  2. Cataract, Glucoma, and visual blurring.
  3. Cushinoid appearence.
  4. Truncal obesity.
  5. Hirsutism
  6. Acne
  7. Impaired wound healing.
  8. Striae
  9. Easy brusing and capillary refill.
  10. Immunosuppression
  11. HTN
  12. Glucose intolerance
  13. Electrolytes disturbence
  14. Fluid retention
  15. Peripheral edema
  16. Increases appetite.
  17. Gastrointestinal bleeding.
  18. Avascular necrosis.
  19. Growth retardation.
  20. Steroid myopathy.
  21. Pneumocystis jiroveci pneumonia (PJP).
  22. Tremor.
  23. Behavioral changes.
  24. Insomnia
  25. Cerebral atrophy.
How well did you know this?
1
Not at all
2
3
4
5
Perfectly
309
Q

What is the normal endometrial thickening in postmenopause females?

A

< 5 mm.

How well did you know this?
1
Not at all
2
3
4
5
Perfectly
310
Q

What is the most common HCV genotype in SA ?

A

HCV-4.

How well did you know this?
1
Not at all
2
3
4
5
Perfectly
311
Q

What are the symptoms of Maple syrup urine disease MSUD ? and what’s the Tx?

A
  • Burnt maple sugar smell in diapers after urine has dried.
  • Fussiness
  • Lethargy
  • Decreased feeding
  • Poor weight gain
  • Hypo/hyper tonia
  • High pitched cry
  • seizures

Tx:
Dietry restriction of branched - chain amino acids BCAAs.

How well did you know this?
1
Not at all
2
3
4
5
Perfectly
312
Q

In which age group does Maple syrup urine disease MSUD occurs in ? and whats the complications if not treated ?

A

Maple syrup urine disease occur in neonates aged 3-7 days, if untreated could lead to encephalopahty and progressive neurodegeneration.

How well did you know this?
1
Not at all
2
3
4
5
Perfectly
313
Q

What is Olanzapine and what its one of his improtant side effects?

A

Olanzapine it is an antipsychotic
S/E: weight gain

How well did you know this?
1
Not at all
2
3
4
5
Perfectly
314
Q

What is the role of ACEI in nephrotic syndrome ?

A

protect the kidneys

How well did you know this?
1
Not at all
2
3
4
5
Perfectly
315
Q

What are the side effects of the following Anti-TB meds ?

  1. Isoniazid
  2. Rifampicin
  3. Pyrazinamide
  4. Ethambutol
  5. Streptomycin
A
  1. Isoniazid:
    - Heptotoxicity
    - Peripheral neuritis
    - Drug induced lupus
    - Psychotic changes
  2. Rifampicin:
    - Hepatotoxicity
    - Autoimmune reactions
    - Thromocytopenia
    - Respiratory shock syndrome
    - Acute hemolytic anemia
  3. Pyrazinamide:
    - Heptotoxicity
    - Arthralgia
    - Hyper-uricemia
  4. Ethambutol:
    - Optic neuritis
    - color blindness
    - Hyperuricemia
  5. Streptomycin :
    - Vestibular dysfunction
    - Nephrotoxicity
    - Deafness
    - Peripheral neuritis
How well did you know this?
1
Not at all
2
3
4
5
Perfectly
316
Q

Which of the following in air-borne transmitted and which is droplets ?

  • Meningococal
  • Measles
  • TB
  • RSV
  • Varicella
  • Influenza
  • Smallpox
A

Airborne:
- Measles
- Varicella
- Smallpox
- TB

Droplets:
- RSV
- Influenza
- Meningococal

How well did you know this?
1
Not at all
2
3
4
5
Perfectly
317
Q

What is FELTY syndrome and what are its components ?

A

It is a rare condition that involves RA, decreased WBC and swollen spleen.
its components:
“SANTA “:

Splenomegaly
Anemia
Neutropenia
Thrombocytopenia
Arthritis (RA)

How well did you know this?
1
Not at all
2
3
4
5
Perfectly
318
Q

Contraindications of liver transplantation ?

A
  1. Alcoholism
  2. Extrahepatic malignancy
  3. Severe and uncontrolled extrahepatic infection
  4. Compensated cirrhosis
  5. Advanced cardiopulmonary disease
  6. Multisystem organ failure
  7. Active substance abuse.
How well did you know this?
1
Not at all
2
3
4
5
Perfectly
319
Q

plantar fasciitis clinical presentation ?

A

It is the most commin casue of heel pain:
It is an intense, sharp heel pain with the first couple of steps in the morning.
Pain decreses with walking or athletic warm up, but then increases throughout the day as activity increases.
In severe cases patient complain of pain after prolonged sitting.

How well did you know this?
1
Not at all
2
3
4
5
Perfectly
320
Q

What is the differnece between Radical, modified and simple mastectomy ?

A
  • Radical mastectomy :
    Removes all breast tissue + skin + axillary lymph nodes + pectoralis muscle
  • Modified mastectomy:
    Removes all breast tissue + skin+ axillary lymph nodes.
  • Simple mastectomy:
    Removes all breast tissue + skin.
How well did you know this?
1
Not at all
2
3
4
5
Perfectly
321
Q

What is a brachial cyst ? and how to treat it ?

A

Smooth and painless neck mass, appears after URTI.
Surgical removal , but if it was infected start with Abx first.

How well did you know this?
1
Not at all
2
3
4
5
Perfectly
322
Q

What are the 2 most common tumors in HIV positive patients ?

A
  1. Kaposi sarcoma (skin and oral mucosa)
  2. Non-hodgkin lymphoma (chest and abdomen).
How well did you know this?
1
Not at all
2
3
4
5
Perfectly
323
Q

Burkitt’s lymphoma :

A
  • B cell lymphoma
  • C myc gene dysregulation
  • Starry sky pattern on microscope
  • Associated with EBV infection.
How well did you know this?
1
Not at all
2
3
4
5
Perfectly
324
Q

What are the indications of surgical excision of a thyroid nodule?

A
  • FNA suspicion
  • > 3-4 cm in size (even if FNA is normal).
  • Increasing in size on serial US
  • Hyperthyroidism unmanageable with radio-iodine therapy.
  • Malignancy other anaplastic CA or thyroid lymphoma.
How well did you know this?
1
Not at all
2
3
4
5
Perfectly
325
Q

What is the gold standard study for diagnosing intussusception ?

A

Barium enema

How well did you know this?
1
Not at all
2
3
4
5
Perfectly
326
Q

Meckel’s diverticulum investigations?

A

Abdominal x-ray
Tc 99m

How well did you know this?
1
Not at all
2
3
4
5
Perfectly
327
Q
  1. What is Hirschsprung ?
  2. gold standard for Dx ?
  3. AXR findings?
  4. Tx ?
A
  1. Migration defect of neurocrest cells “aganglionic”. Failure to pass meconium/distal bowel obstruction/abdominal distention.
  2. Rectal biopsy is gold standard : aganglionosis and neuronal hypertrophy in muscularis externa.
  3. AXR: narrow rectum

4: Surgical resection of aganglionic part “anal pull through”

How well did you know this?
1
Not at all
2
3
4
5
Perfectly
328
Q

What is the medication that can treat small renal stones ?

A

Urodeoxycholic acid

How well did you know this?
1
Not at all
2
3
4
5
Perfectly
329
Q

Pancreatic cancer tumor marker ?

A

CA 19- 9

How well did you know this?
1
Not at all
2
3
4
5
Perfectly
330
Q

Digeorge syndrome ?

A
  • 22q11.2 deletion syndrome
  • Syndrome is caused by deletion of small segment of chromosome 22
  • Congenital heart problems
  • Specific facial features
  • Frequent infections
  • Developmental delay and learning problems
  • Cleft palate
How well did you know this?
1
Not at all
2
3
4
5
Perfectly
331
Q

Defenition of pyrexia of unknown origin?

A
  • Temp. > 38.3 on several occasions
  • > 3 weeks duration of illness
  • Failure to reach a diagnosis despite 1 week of inpatient investigations.
How well did you know this?
1
Not at all
2
3
4
5
Perfectly
332
Q

Which type of cancer is associated with hashimoto’s thyroiditis ?

A

Papillary thyroid cancer

How well did you know this?
1
Not at all
2
3
4
5
Perfectly
333
Q

What is the most common cancer in childhood ?

A

Leukemia

How well did you know this?
1
Not at all
2
3
4
5
Perfectly
334
Q

Obturator nerve trauma may cause paralysis to which muscles ?

A
  • Adductor longus
  • Adductor brevis
  • Gracilis
How well did you know this?
1
Not at all
2
3
4
5
Perfectly
335
Q

How often should kidney assessment done in DM patient and how ?

A

Assess kidney in DM annually through urine by microalbuminuria

336
Q

What is horner’s syndrome ?

A

Miosis + Ptosis + anhidrosis (loss of sweat of face).

337
Q

What is the study of choice in evaluating coronary artery aneurysm?

A

Echocardiography.

338
Q

What is Kawasaki disease? and how to diagnose it?

A

It is an acute febrile illness of early childhood characterized by vasculitis of the medium-sized arteries.

Complete disease is diagnosed by:
- Fever for at least 5 days
+ 4-5 of the following:
1. Extremity changes
2. Polymorphus rash
3. Oropharyngeal changes
4. Bilateral, nonexudative , limb sparing, painless, bulbar conjunctival injection.
5. Acute unilateral nonpurulent cervical lymphadenopathy with LN diameter > 1.5 cm.

Incomplete disease : fever > 5 days + 2/3 of features + lab findings:

lab findings:
- Hight ESR
- High CRP
- Hypoalbuminemia
- Anemia
- High ALT
- Thrombocytosis
- Leukcocytosis
- Puria

339
Q

What is the management of Kawasaki disease ?

A

The goal is to prevent coronary artery disease.

  • IVIG + Aspirin is the mainstay of treatment
  • Corticosteroids, infliximab , methotrexate can be used.
  • Other anticoagulants can be used.
340
Q

What are the symptoms of polycystic ovarian syndrome ?

A
  • Menstrual dysfunction
  • Anovulation
  • Signs of hyperandrogenism
  • Hirstusim
  • Infertility
  • Obesity and metabolic syndrome
  • DM
  • Obstructive sleep apnea.
341
Q

Which conditions should be excluded before the diagnosis of poly-cystic ovarian syndrome ?

A
  • Adrenal and ovarian tumors
  • Thyroid dysfunction
  • Congenital adrenal hypoerplasia
  • Hyperprolactinemia
  • Acromegaly
  • Cushing syndrome
342
Q

When to do the quad screen test ? and what are the substances measured ?

A

at the second trimester of pregnancy.

  • Alpha- fetoprotien AFP (hormone made by developing baby).
  • Human chorionic gonadotropin HCG (hormone made by placenta).
  • Estriol (hormone made by placenta and baby’s liver
  • Inhibin A (hormonal made by placenta).
343
Q

Clinical triad for ectopic pregnancy ?

A
  1. Abdominal pain
  2. Amenorrhea
  3. Vaginal bleeding.
344
Q

What is the medical treatment of ectopic pregnancy?

A

Methotrexate

345
Q

What are the indications of methotrexate in ectopic pregnancy ?

A
  1. Hemodynamic stability
  2. No severe / persisting abdominal pain.
  3. The ability of follow up multiple times
  4. Normal baseline liver and renal finction test.
346
Q

What are the absolute contrainidcations to methotrexate therapy in ectopic pregnancy ?

A
  1. Existence of intrauterine pregnancy
  2. Immunodeficiency
  3. Moderate - severe anemia , leukopenia and thrombocytopenia.
  4. Sensitivity to methotrexate.
  5. Active pulmonary or peptic ulcer disease.
  6. Breastfeeding
  7. Clinically important hepatic or renal dysfunction
  8. Evidence of tubal rupture.
347
Q

What is the most common predisposing cause of ectopic pregnancy ?

A

History of pelvic inflammatory disease.

348
Q

How to differentiate between benign nevi and melanoma?

A

Melanoma:

A: asymmetry
B: Border irrgularity
C: Color that’s dark black or blue
D: Diameter >/= 6mm.

349
Q

What is the metformin mechanism of action ?

A
  • Improves glucose tolerence in patients with type 2 DM
  • Lowering basal and postprandial plasma glucose
  • Decrease hepatic glucose production
  • Decrease intestinal absorption of glucose.
  • Improves insulin sensitivity by increasing peripheral glucose uptake and utilization.
350
Q

How to manage cardiac tamponade ?

A
  • Emergency subxiphoid percutaneous drainage.
  • Pericardiocentesis +/- echo guidence
  • Percutaneous balloon pericardiotomy.
351
Q

what is placental abruption / Abruptio placenta ?

A

Premature seperation of placenta from the uterus.
Typical presentation: bleeding - uterine contraction + fetal distress.

352
Q

What is the DDX of non-blaching rash ?

A
  • Meningococcal disease
  • Sepsis
  • Idiopathic thrombocytopenia purpura
353
Q

Initial emergency management of hyperkalemia ?

A
  • Cardiac monitoring
  • IV bicarbonate (NaHCO3)
  • Ca
  • Insulin + 50% of dectrose
  • If digoxin toxicity is suspected give MgSO4
  • Measure K, 1,2,4,6 and 24 hours after Dx and Tx.
354
Q

What is the pharmacological therapy of hyperkalemia ?

A

Step 1:
- IV Ca (to improve cardiac toxicity)
- CaCl or Cagluconate

Step 2:
- Identify and remove sources of K and discontinue K supplements.

Step 3:
- IV glucose + IV insulin (To enhance K uptake by cells + to decrease serum concentration).
- Beta-adrenergic agonist : nebulized Albuterol

Step 4:
- IV saline with loop diuretics “Furosemide”
- Aldosterone analogue : 9 - alpha fluorohydrocortisone acetate (renal excretion of K)
- Resin : sodium polystyrene sulphonate (GI excretion)
- Patiromer sorbitex calcium
- Sodium zirconium cyclosilicate

Step 5:
Emergency dialysis : If not respondinf to meds.

***IF no ECG changes start with step 3

355
Q

What is diabetus insipidus ?

A

It is the passage of large volume > 3L/24 hours of dilute urine <300 mOsm/Kg

Forms:
1. Central (neurogenic, pituitary or neurohypophyseal)
characterized by: Decreased ADH (arganine vasopressin AVP)

  1. Nephrogenic, characterized by decreased ability to concentrate urine because of resistence to ADH action in the kideny.
  2. Gestational DI, due to deficiency of ADH but nor because of reason 1 and 2.
  3. Primary polydypsia , due to deficiency of ADH but nor because of reason 1 and 2.
356
Q

What is the clinical presentation of diabetes insipidus ?

A
  • Hydronephrosis +/- pelvic fullness +/- flank pain / tenderness.
  • Bladder enlargement
  • Dehydration.
357
Q

How to investigate in diabetes insipidus ?

A
  • 24 hour urine collection to determine volume
  • Serum electrolytes
  • Glucose level
  • Urinary specific gravity
  • Plasma ADH level
  • Water deprivation (miller - moses test) to ensure adequate dehydration and maximal stimulation of ADH
  • Pituatry studies.
358
Q

How to investigate TB in pregnant women ?

A
  • All pregnants can undergo Tuberculin skin testing.
  • If PPD test is +ve —> CXR with lead shielding
  • Chest radiograph should not be delyed during first 3 months of pregnancy.
359
Q

What is the main cause of cervical cancer ?

A

HPV

360
Q

How to diagnose genital warts during pregnancy?
and how to treat ?

A

Dx:
1. Pap smear, if abnormal :
2. Colposcopy.

Tx:
Medical:
- Imiquimod
- Podophylin and podofilox
- Trichloroacetic acid

Surgical:
- Electrocautery
- Cryosurgery (freezing warts)
- Laser
- Excision
- Interferon injections

361
Q

What is the clinical triad of Wiskott-Aldrich syndrome ?
How to Tx ?

A

X - linked disorder

Clinical triad :
1. Microthrombocytopenia
2. Eczema
3. Recurrent infections.

Tx include:
- Abx
- IVIG
- Splenectomy in special cases
- Gene therapy
- Early hematopoietic stem cells transplantation
- Immunomoulatory.

362
Q

Steeple / pencil sign in CXR indicates … ?

A

Croup (Acute laryngotracheitis / acute laryngotracheobronchitis).

363
Q

What is the pentad of Thrombotic Thromocytopenic Purpura (TTP) ?

A
  1. Microangiopathic hemolytic anemia
  2. Thrombocytopenia purpura
  3. Neurologic abnormalitis (seizures, altered mental status, hemiplagia , aphasia, paresthesia and visual disturbance)
  4. Fever
  5. Renal disease.
364
Q

How to differentiate between ITP, TTP and DIC ?

A

ITP (Immune thrombocytopenic purpura):
- Antiplatlets and antibodies
- RBC : -
- Coagulation profile : Normal
- Tx : Steroids - IVIG - Splenectomy.

TTP (Thrombotic thrombocytopenic purpura):
- Endothelial defect
- RBC: schistocytes
- Coagulation profile : Normal or high
- Tx: Plasma exchange - Vincristine

DIC (Disseminated intravascular coagulation) :
- Thrombin excess
- +/- schistocytes
- Tx: Plasma - platelets - treatment of the cause.

365
Q

Anal fissure presentation and Tx ?

A
  • Severe pain during bowel movement with pain lasting mins-hours afterward.
  • PAtient become afraid of having bowel movement –> wosening of constipation, hard stool and more pain.
  • Associated with bright red blood (70%).

Tx :
- Treatment of constipation: fiber supplements + stool softner.
- Sitz bath
- High fiber diet
- Nitroglycerine “Glycerol trinitrate” ointment (side effect: headache and dizziness).
- Botulinum toxins
- Lateral internal sphincteretomy
- Sphincter dilation.

366
Q

Trichomoniasis :
1. Etiology
2. Sx
3. Dx
4. Tx

A
  1. Etiology:
    - Motile parasitic protozoen
    - Trichomonas vaginalis
  2. Sx:
    - Frothy yellow - green vafinal discharge
    - Valvular irritation
  3. Dx:
    - Wet mount microscopy
  4. Tx :
    - It is sexulally transmitted , so stop sex during treatment period
    - Oral Metronidazole
367
Q

TB workup ?

A
  • Primary screening method is : Mantoux tuberculin skin test with purified protein derivative PPD.
  • Blood test based on interferon - gamma relaes assay IGRA with antigen specific for mycobacterium tuberculosis can be used for latent TB infections
  • Acid fast bacilli smear and culture
  • HIV serology for unknown HIV status
  • CXR
  • Sputum smear :
    • should be collected in early morning on 3 consecutive days.
    • If hospitalized patients —> Q8hrs
  • AFB cultures in the most specific test for TB.
368
Q

How to treat seizures caused by isoniazid overdose ?

A
  • Diazepam to control seizures.
  • Pyridoxin drug of choice
  • If patient is awake and alert —> activated charcol + sorbital orally.
369
Q

In which weeks of pregnancy is the patient when the Fundal height reaches :
- Near pubic bone
- Between pubic symphesis and umbilicus.
- Umbilicus
- Edge of ribs
- Below sternum.
?

A
  • Near pubic bone : 12 weeks
  • Between pubic symphesis and umbilicus: 18 weeks.
  • Umbilicus : 24 weeks
  • Edge of ribs : 30 weeks.
  • Below sternum : 36 - 38 weeks.
370
Q

What is the most common organism of acute tonsillitis ?

A

Group A beta-hemolytic streptococcus pyogenes GABHS.

371
Q

What are the complications of acute tonsilitis ?

A
  1. Acute poststreptococcal glomerulonephritis.
  2. Scarlet fever
  3. Rheumatic fever
  4. Septic arthritis
372
Q

What is the treatment of Group A beta-hemolytic streptococcus pyogenes GABHS. acute tonsilitis ?

A

Penicillin PO for 10 days.
Tonsillectomy if:
1. > 6 episodes of streptococcus in 1 year
2. 5 episodes in 2-3 years
3. Not responding to antibiotics.

373
Q

What to do if someone exposed to measels and he\she is not immunized ?

A

Give immunoglobulin IM or IV within 6 days of exposure.

374
Q

What is the clinical presentation of measles ?
Tx ?

A
  • 3 Cs : Coryza - Conjunctivitis - Cough
  • High fever of 40 (4-7 days)
  • Photophobia
  • Periorbital edema
  • Myalgias
  • Koplic spots
  • Rash: mild pruritis, blanching, erythematous, macules and papules starts on face –> sides of the neck and behind the ears then spread cephalocaudaly.
  • Most ill during rash period
  • Rash develop 14 days after exposure.

Tx:
- IVF and supportive management
- Vitamin A supplement.

375
Q

Salicylate / Aspirin toxicity management ?

A
  • ABCs
  • GIT decontamination:
    • Gastric lavage within 60 mins with warmed isotonic NaCl solution.
    • Activated charcol within 1 hour of ingestion +/- sorbitol.
    • Whole bowel irrigation with polyethylene glycol.
  • Urinary excretion and alkalization:
    • IV NaHCO3
    • Ringer lactate/ isotonic NaCl.
  • Hemodialysis if:
    • Serum salicylate >120 mg/dL
    • Refractory acidosis or volume overload
    • Coma or seizures or renal failure
    • Noncardiogenic pulmonary edema.
376
Q

Ischemic stroke Tx options ?

A

** ABCs alwayes then:
- Fibrinolytic therapy (Alteplase rt-PA)
- Antiplatelets ( 24-48 hours of ischemic stoke onset).
- Mechanical thrombectomy.

377
Q

When to use Fibrinolytic therapy (Alteplase rt-PA) in ischemic stroke ?

A
  • Within 3 hours of stroke onset
  • Only after CT had ruled out hemorrhagic stroke.
  • 3 - 4.5 hours can improve neurologic outcome.
378
Q

What are the risk factors of congenital cerebral palsy ?

A
  • low birth weight
  • Premature birth (before 32 weeks)
  • Multiple births
  • Infections during pregnancy.
379
Q

What is the initial test for peripheral arterial disease PAD ? and how to interfere with results ?

A

ABI : Ankle - Brachial Index

Abnormal : = 0.9
Borderline : 0.91- 0.99
Normal : 1 - 1.4
Noncompressible : >1.4

380
Q

Milan criteria for liver transplantation in patients with heptocellualr carcinoma and liver cirrhosis ?

A
  • single tumor with diameter = 5 cm OR
  • up to 3 tumors , each with diameter = 3 cm.
    AND
  • No major vessels or extra-hepatic involvement.
381
Q

What are the causes of short stature ?

A
  • Chronic disease
  • Familial short stature
  • Constitutional delay of growth and development
  • Endocrine disease.
382
Q

How to differntiate between familial , constitutional and endocrine short stature ?

A

Familial short stature:
- Bone age is appropriate for chronologic age.
- Normal growth velocity
- Predicted adult height appropriate to the familial pattern.

Constitutional short stature:
- Delayed bone age.
- Normal growth velocity.
- Predicted adult height appropriate to the familial pattern.

Endocrine disease:
- Linear GF that occurs to a greater degree than wight loss.

383
Q

What are the lab studies used to investigate short stature ?

A
  • Serum insulin-like growth factor -1 (IGF-1) “Somatomedin C”
  • IGF binding protein -3 (IGFBP-3).
  • Karyotype by G-binding
  • Serum growth hormone (GH).
384
Q

What is hydatidiform mole ?

A

It is a rare mass that forms inside the uterus at the beggening of a pregnancy.
It is a type of gestational trophoblastic disease that originate in the placenta.

385
Q

What are the types of gestational trophoblastic disease GTD ?

A
  1. Complete mole (No fetal tissue)
  2. Partial moles (with fetal tissue)
  3. Placenta site trophoblastic tumors.
  4. Choriocarcinoma
  5. Invasive moles.
386
Q

What are the risk factors of gestational trophoblastic disease GTD ?

A

Diet deficient of animal fat and carotene.

387
Q

What is the presentation of complete mole ?

A
  • Vaginal bleeding (irregular) –> Uterus may be distended dut to large amount of blood.
  • No fetal tissue
  • Hyperemesis (nasea and vomiting).
  • Hypothyroidism (due to stimulation of thyroid gland by high B-hcg”
  • Uterine size is larger than gestational age (CLASSIC SIGN).
  • Early failed pregnancy.
388
Q

What is the presentation of partial mole ? and how to treat ?

A
  • Missed abortion
  • Vaginal bleeding
  • Absence of fetal heart sound
  • Anemia

Tx:
- D & C is always necessory
- IV Oxytocin should be started after dilatation of cervix to prevent hemorrhage.

389
Q

Breat cancer during pregnancy is treated exactly as if pregnancy didn’t exist, except from :

A
  • No radiotherapy during pregnancy
  • No chemotherapy during first trimester
  • Termination of pregnancy isn’t necessory.
390
Q

Which surgery is appropriate for inflammatory breast cancer ?

A

almost always : Modified radical mastectomy.

391
Q

Most common heart defect in down syndrome is ?

A

Atrioventricular septal defect.

392
Q

Risk factors of ectopic pregnancy ?

A
  • Maternal age of 35-44 years
  • Previous ectopic pregnancy
  • Previous pelvic / abdominal durgery
  • Pelvic inflammatory disease.
  • Induced abortions.
  • Conceiving after tubal ligation or IUD
  • smoking
  • Endometriosis
  • Fertility treatment.
393
Q

What are the casues of spotting between periods ?

A
  • Hormonal contraceptives
  • Taking emergency contraceptive pill
  • Vaginal inury
  • Recent abortion
  • Sexually transmitted infection
  • Vaginal dryness
  • Cervical cancer
  • Uterine cancer
  • Cervical endometrial polyps
394
Q

What investigations to order in dysfunctional/abnormal uterine bleeding? and how to manage ?

A

Investigations:
- HCG
- Pap smear
- Endometrial sampling
- Thyroid function and prolactin
- Liver function test
- Coagulation profile
- U/S

Management:
- Oral contraceptives
- Estrogen “Prolonged uterine bleeding”
- Progestins (chronic management)
- Desmopressin
- Hysterectomy
- Endometrial ablation.

395
Q

What is the normal postmenopausal endometrial thickness?

A
  • Typically < 5 mm.
  • If vaginal bleeding and patient NOT on Tamoxifen < 5 mm is upper limit.
  • If no history of vaginal bleeding : 8-11 mm is acceptable.
  • If abnormal thickening : repeat transvaginal U/S or referr to gyne.
396
Q

Adrenal incidentaloma is usually associated with which conditions ?

A
  • Cushing syndrome/ subclinical hypercortisolism.
  • Hyperaldosteronism “Pheochromocytoma”.
397
Q

What is a carcinoid tumor ?

A

Carcinoid tumors are neuroendocrine in origin. Derived from primitive stem cells in the gut wall espically appendix, but it can be seen in other organs such as; lungs, mediastinum , liver , bile duct, ovarian , prostate etc.

398
Q

What are the five As of smoking cessation intervention ?

A
  1. Ask about tobacco use
  2. Advise to quit through personalized messages
  3. Assess willingness to quit
  4. Assist with quitting
  5. Arrange follow-up care and support.
399
Q

A child can still be vaccinated if:

A
  • Low grade fever
  • Col. runny nose or cough
  • Ear infection
  • Mild diarrhea
  • On antibiotics
400
Q

How is the Hb electrophoresis results in a healthy person ?

A
  • HbA1: 95 - 98 %
  • HbA2: 1.5 - 3.5 %
  • HbF: <2 %. (fetal Hb).
  • HbC: Absent
  • HbS: Absent
401
Q

How is the Hb electrophoresis results in sickle cell trait , sickle a-thalasemia ?

A
  • HbS: Present.
  • HbA1 and HbA2 > HbS
402
Q

How is the Hb electrophoresis results in sickle cell anemia and sickle beta-thalasemia ?

A
  • HbA : Absent
  • HbS: Present.
  • HbF: present

In sickle beta- thalasemia :
HbS > HbA

403
Q

How is the Hb electrophoresis results in beta-thalasemia minor ?

A

High HbA2

404
Q

What are the advised dietary measures in irritable bowel syndrome ?

A
  • Fiber supplement improve constipation and diarrhea.
  • Avoid caffeine
  • Avoid Legume to decrease bloating. “بقول”
  • Limit lactose and fructose.
405
Q

Name the major stress hormones and their role in the human body.

A
  1. Adrenaline :
    - Increases HR
    - Increases BP
    - Responsible for immediate reactions
    - Produced from adrenal gland.
  2. Cortisol:
    - It allows body to concentrate on stressful situations.
    - Increases sugar in bloodstream.
    - Suppress the unessential processes like digestion.
    - Produced in adrenla gland but it takes time.
406
Q

Which hormones are responsiable for aggressiveness and violence ?

A
  • Testosterone
  • Serotonin.
407
Q

What are the factors that promotes passage of meconium in utero? (Meconium Aspiration Syndrome MAS)

A
  1. Placental insufficiency
  2. Maternal HTN
  3. Preeclampsia
  4. Oligohydromnios
  5. Infection
  6. Acidosis
  7. Maternal drug abse (Cocaine and tobacoo).
408
Q

How to manage Meconium Aspiration Syndrome MAS ?

A
  • It is necessory to perform echocardiogram and cardiac examination to evaluate for congenital heart diseases and persistant pulmonary HTN of the newborn.
  • If baby is vigorous (normal respiration): bulb syringe used gently to clear secretions.
  • If baby in Not vigorous (depressed respiration): place baby on radient warmer, bulb syringe , new born resuscitation (warming, repositioning of head and stimulation).
  • If baby still not breathing , or HR < 100 bmp : use positive pressure ventilation.

Continue care in NICU:
- Maintain optimal thermal environment to minimize O2 consumption.
- Sedation may by used if baby is agitated.
- Umbilical artery catheter insertion to monitor blood pH.
- Surfactant therapy: decrease severity of disease and decrease hospitalization.
- Ventilation is commonly used.
- In persistant pulmonary HTN of the new born, Tx is : inhaled nitric oxide (Pulmonary vasodilator of choice).

409
Q

What is the most common organism that causes ventriculo-peritoneal shunt infection ?

A
  1. Staphylococci (Coagulase -ve).
  2. Staphylococcus epidermidis).
410
Q

What are the contra-indications of laparoscopic cholecystectomy ?

A
  1. Generalized peritonitis.
  2. Septic shock from cholangitis.
  3. Severe acute pancreatitis
  4. Untreated coagulopathy
  5. Lack of eqipments or surgeons
  6. Advanced liver cirrhosis and failure of hepatic function
  7. Suspected gallbladder cancer.
411
Q

Internal hemorrhoids treatment ?

A
  • Grade 1: Conservative medical therapy + avoid NSAIDs , spicy food and fatty food.
  • Grade 2 and grade 3 : Nonsurgical procedures:
  • Ruber band ligation (grade 2&3).
  • Coagulation, electrocautery and electrotherapy (grade 1 &2).
  • Sclerotherapy and cryotherapy (early internal hemorrhoids).
  • Laser therapy and radiowave ablation.
  • Grade 4 and severly symptomatic grade 3 : Surgical removal.
412
Q

Hepatocellular carcinoma treatment ?

A
  • Liver Transplantation (best management)
    Other options:
  • Resection
  • Radiofrequency ablation RFA
  • Systemic therapy with Sorafenib.
413
Q

When does liver resection is considered treatment of choice in patients with hepatocellular carcinoma ?

A

when the tumor is LESS than 5 cm in the ABSENCE of liver cirrhosis.

414
Q

One year of smoking cessation will decrease the risk of coronoray artery disease by how much ?

A

1/2

415
Q

What is the definition of steroid - resistant nephrotic syndrome ?

A

It is the absence of remission despite 4 weeks of therapy with daily Prednisolone at 9 dose of 2 mg/Kg/day.

416
Q

What is Boerhave syndrome ?

A

It is a spontaneous rupture of the esophagus which occur after a forcefull emesis.
It is a transmural perforation.

417
Q

How to differentiate between Boerhave syndrome and Mallory weiss syndrome ?

A

Boerhave syndrome :
- transmural perforation.
- Tx: combination of conservative and surgical:
* IV resusitation
* broad spectrum Abx
* surgical intervention.

Mallory weiss syndrome :
- non-transmural perforation.
- Supportive treatment

418
Q

What is the most common symptoms of enterocele ?

A

Feeling of heaviness or fullness in the vagina.

419
Q

Name the compartments of the leg.

A
  1. Anterior
  2. Lateral
  3. Deep posterior
  4. Superficial posterior.
420
Q

what are the degrees of the perianal tear ?

A

First degree: Injury to the perianal skin +/- vaginal mucose.

Second degree: Injury to the perineum involving perianal muscles but not the anal sphincter.

Third degree: Injury to the perineum involving anal sphincter.

Fouth degree: Injury to the perineum involving anal sphincter complex and anorectal mucose.

421
Q

What is Trichotillomania and whats its treatment ?

A

It is a hair pulling disorder

Tx:
- First line : behavioral treatemnt and intervention
- N-acetyl cysteine
- Olanzapine

422
Q

What is the most common cause of first trimester miscarrge ?

A

Chromosomal abnormalities.

423
Q

What is the Tx of neonatal seizures ?

A
  • Phenobarbital
  • Seizures persist : Phenytoin
  • IV benzodiazepine (Lorazepam or Midazolam).
424
Q

What are the Sx of intusseception ?
and how to Dx ?

A
  • Nonbilious vomiting until obstruction occur.
  • Abdominal pain : colicky, severe and intermittent.
  • Passage of blood and mucus: current jelly
  • Lethargy
  • Palpable abdominal mass.

Dx:
- Abdominal x-ray
- U/S : target sign and pseudokidney sign
- Contrast enema.

425
Q

how to manage a patient with low lying placenta if she is less than 37 weeks or if she is more then 37 weeks ?

A

if < 37 weeks: admit to the hospital
If > /= 37 weeks : CS

426
Q

What is the classic triad of renal cell carcinoma ?

A
  1. Gross hematuria
  2. Flank pain
  3. Palpable mass
427
Q

Which antibiotics to use if bacterial gastroenteritis is suspected ?

A
  • Fluroquinolone (ciprofloxacin ) for 5 days
  • TMP- SMX in an alternative.
428
Q

What is the most common viral pathogen of otitis media ?

A
  • Rhinovirus
  • RSV
429
Q

What is the most common cause of inspiratory stridor in neonatal period ? and how to manage it ?

A

Laryngomalacia

  • Place baby in prone position
  • elevate head
  • Observation
  • +/- PPI
  • It with resolve sponatneously after 12-18 months later.
430
Q

Cardiotocography CTG interpretation ?

A

VEAL CHOP

Variable decleration = Cord compression
Early decleration = Head compression
Accelaration = Ok
Late decleration = Placenta insufficiency.

431
Q

Pre-eclampsia with severe features ?

A
  1. SBP >/= 160 mmHg or DBP >/= 110 mmHg on 2 occasions, 4 hours apart.
  2. Impaired hepatic function (Liver enzymes increase 2X).
  3. Severe persistent upper quadrent pain or epigastric pain.
  4. Progressive renal insufficiency
  5. new onset cerebral or visual disturbance
  6. Pulmonary edema
  7. Thrombocytopenia.
432
Q

What are the risk factors of sudden infant death syndrome ?

A
  1. Prematurity (<37 weeks).
  2. Early bed sharing (<12 weeks)
  3. Alcohol use during pregnancy
  4. Sidt bedding (sheet bed cover).
  5. low birth weight
  6. Aboriginals ethincity
  7. Male
  8. No prenatal care
  9. Smoking in house
  10. Pron sleep position
  11. Poverty.
433
Q

What is the most common cause of Parkinsonism ?
and what is the gold standard treatment of Sx parkinsons ?

A

Idiopathic parkinson’s disease. Which is characterized by loss of Dopamine neurons in the substantia nigra.

gold standard treatment of Sx parkinsons is : Levodopa / Carbidopa.

434
Q

What is the empirical treatment of infective endocarditis in native vs prosthetic valves ?

A

Native valve infective endocarditis :
- Vancomycin + Gentamycin/Ceftriaxone

Prosthetic valve infective endocarditis :
Vancomycin + Gentamycin + Cefepime + Rifampin.

435
Q

What are the indications of spleenectomy ?

A

SHIRTS :

Splenic abscess/Splenomegaly
Hereditary spherocytosis
Immune Thrombocytopenia Purpura ITP
Rupture of spleen
Thrombotic Thrombocytopenic Purpura TTP
Splenic vein thrombosis.

436
Q

What are the side effectis of Digoxin ?

A
  1. AV block
    2.Tachyarrhythmias
  2. Bradyarrhythmias
  3. Blurred/yellow vision (Van - gogh syndrome)
  4. Anorexia , N/V.
437
Q

What is the classification criteria of Wegener’s granulomatosis ?

A
  1. Nasal/oral involvement “inflammation, ulcer , epistaxsis”
  2. Abnormal CXR “nodules, cavitations”
  3. Urinary sediment “ microscopic hematuria +/- RBC cust”
  4. Biopsy of involved tissue:
    • Lungs: granulomas
    • Kidney: necrotizing segmental glomerulonephritis.

C-ANCA and ESR used to monitor disease and response to Tx.

438
Q

At which age does stepping/walking/dance reflex dissappear ?

A

2 months

439
Q

Which imaging is used to screen and to confirm Abdominal aortic aneurysm AAA ?

A

Screen : U/S
Confirm: CT with IV contrast

440
Q

What are the surgical indications of Abdominal aortic aneurysm AAA ?

A
  1. Ruptured
  2. Symptomatic
  3. > 5 -5.9 cm
  4. Ascending thoracic aorta aneurysm
  5. Enlarging
  6. 4.5 - 5 cm in Marfan syndrome.
441
Q

What are the major and minor risk factors of peripheral vascular disease PVD ?

A

Major :
- Smoking
- DM
- Older age

Minor:
- HTN
- Hyperlipidemia
- Obesity
- Sedentary lifestyle
- PMHx or FHx of CAD/CVD.

442
Q

Athetoid cerebral palsy is associated with which condition ?

A

Kernicterus ( Brain damage due to hyperbilirubinemia).

443
Q

What is the management of pituitary adenoma ?

A
  • If asymptomatic + < 10 mm : Follow with MRI
  • If symptomatic or > 10 mm:
    medical treatment: Cabergoline - Bromocriptine or Bromocriptine (both dopamine agonist with S/E: N/V, orthostasis or nasal congestion.
  • If medical treatment failed / can’t tolerate meds/ recurrence: Trans-sphenoidal surgery
  • If medical and surgical therapy failed: Radiation.
444
Q

What is the classic triad of measles ?

A

3 Cs:
Cough
Coryza
Conjunctivitis

445
Q

What are the 2 vaccines that are recommended for all 65 years adults or older ?

A
  • PCV13 : Pneumococcal conjugate vaccine
    then
  • PPSV23 :Pneumococcal Polysaccharide Vaccine
    at least 1 year later.
446
Q

Best imaging study to diagnose diverticulitis ?

A

CT

447
Q

Which antibiotics are used to treat diverticulitis ?

A
  • Ciprofloxacin + Metronidazole OR
  • TMP-SMX + Metrinidazole OR
  • Amoxicillin - clavulanate OR
  • Moxifloxacin
448
Q

What is the treatment of diverticulitis with abscess ?

A
  • Abscess > 4 cm : Drain percutaneously
  • Abscess < 4 cm : Abx only.
449
Q

What is Sialadenitis?
Most common organism ?
Most common causes ?
Tx?

A
  • Sialadenitis is an acute inflammation of salivary glands.
  • Most common organism is staphylococcus aureus.
  • Most common causes:
    1. Post-op dehydration
    2. Radiation therapy
    3. Immunosuppression

Tx:
- Hydration
- Abx: Clindamycin
- warm compressors
- Message

450
Q

Which medication is used for mysthenia gravic symptomatic relief ?

A

Acetylcholinesterase inhibitors : Pyridostigmine

451
Q

What is the single most important preventive measure for coronary artery disease ?

A

Cessation of cigarette smoking

452
Q

What are the most common conginital urological anomalies ?

A
  • Hydronephrosis
  • Cryptochidism “Undescended testicles”
  • Hypospadius
  • Vesicurethral reflux
453
Q

How to Tx motion sickness ?

A
  1. Anticholinergic agents : Scopolamine
  2. Antihistamine : Dimenhydrinate.
454
Q

What is the treatment for hydatid cyst ?

A

Benzimidazole : Albendazole and Mebendazole.

455
Q

What is NEXUS criteria for c-spine imaging?

A
  1. Focal neurologic deficit.
  2. Midline spinal tenderness
  3. Altered level of consciousness.
  4. Intoxication
  5. Distracting injuries.

If any of the above is present, c-spine can not be cleared and further imaging is needed.

456
Q

What is the most common source of post-coital bleeding ?

A

The cervix.

457
Q

What is Kartagener syndrome ?

A

It is an autosomal recessive syndrome.
Triad:
1. Situs inversus
2. Chronic sinusitis
3. Bronchiectasis

Problem in movement of the cilia, leading to recurrent chest infections, ear/ nose / throat symptoms and infertility.

458
Q

SLE dx criteria ?

A

SOAP BRAIN MD

  • Serositis : pleurisy, Pericarditis on PEx or ECG or imaging.
  • Oral ulcers: oral/nasopharyngeal “palate” painless.
  • Arthritis: nonerosive, >/= 2 peripheral joints + tenderness & swelling.
  • Photosensitvty.
  • Blood disorders: Leukopenia , lymphoma , thrombocytopenia , hemolytic anemia.
  • Renal involvement : proteinuria
  • Antinuclear antibodies
  • Immunologic phenomena: dsDNA, Anitsmith Ab , Antiphospholipids.
  • Neurological disorder: Seizures / psychosis
  • Malar rash
  • Discoid rash .
459
Q

Which electrolyte disorder is associated with short bowel syndrome ?

A

Hypomagnesemia

460
Q

DM Dx criteria ?

A
  1. Fasting plasma glucose >/= 126 mg/dL (>/= 7 mmol/L).
  2. 2-hour plasma glucose during a 75 g oral glucose tolerance test (OGTT): >/= 200mg/dL (>/= 7 mmol/L).
  3. Random blood glucose with classic Sx: >/= 200 mg/dL (>/= 11.1 mmol/L).
  4. HbA1C: >/= 6.5%.
461
Q

What to give in :
- Oopherectomy
- Oopherectomy + hystrectomy ?

A
  • Oopherectomy : Estrogen + Progestin
  • Oopherectomy + hystrectomy : Estrogen.
462
Q

What is the mechanism of action of octreotide in variceal bleeding ?

A

acterotide causes splanchnic vasoconstriction –> decreases portal pressure and decreases or stopping variceal bleeding.

463
Q

What are the causes of cirrhosis ?

A
  • Alcohol
  • Primary biliary cirrhosis
  • Sclerosing cholangitis
  • Alpha - 1 antitrypsin deficiency
  • Hemochromatosis
  • Wilson disease.
464
Q

What is the most common cause of liver cirrhosis and what is the most common cause of liver transplant ?

A
  • most common cause of liver cirrhosis is alcohol
  • most common cause of liver transplant is chronic HCV..
465
Q

All clotting factors are made in the liver except :

A
  • Factor VIII
  • Von Willebrand factor

both made by endothelial cells.

466
Q

How to manage Spontaneous Bacterial Peritonitis SBP ?

A
  • Cefotaxime or ceftriaxone
  • Albumin infusion , decreases risk of hepatorenal syndrome.
  • Stop beta-blockers as its increases mortality.
467
Q

Interpret with Serum - Ascitic Albumin Gradient SAAG.

A

SAAG >/= 1.1 —> Increased hydrostatic pressure:
- SAAG >/= 1.1 and total protein < 2.5 g/dL : Portal HTN.
- SAAG >/= 1.1 and total protein > 2.5 g/dL: HF, Budd-chair.

SAAG < 1.1 = ascitic albumin is high : Cancer or infection:
- Total protein < 2.5 g/dL : nephrotic syndrome
_ Total protein > 2.5 g/dL :Carcinomatosis (Ovarian) or TB.

*** TB : peritoneum Bx : High lymphocytes.

468
Q

How to manage Cirrhosis ?

A
  • Spironolactone + Furosemide
  • Propranolol to prevent bleeding
  • In encephalopathy : Lactulose
  • In esophageal varices bleeding : Octereotide.
469
Q

Which meds should be stopped in ascitis ?

A
  • NSAIDs
  • ACEI
  • ARBs
470
Q

What is charcot’s triad ?

A

Acute cholangitis :
1. Fever
2. RUQ pain
3. Jaundice

471
Q

Reynold’s pentad ?

A

Acute suppurative cholangitis :
1. Fever
2. RUQ pain
3. Jaundice
4. Hypotension
5. Confusion

472
Q

How to diagnosis Hemolytic disease of the fetus and newborn (HDFN) ?

A
  1. Prenatal diagnosis:
    Imaging
    - Ultrasound: to determine hydrops fetalis.
    - Doppler sonography of fetal blood vessels: Increased flow rate indicates fetal anemia.

2.Postnatal diagnosis”
- If the newborn has signs of hemolysis, conduct a Coombs test (either direct or indirect).

  • Rh incompatibility: positive
  • ABO incompatibility: weak positive or negative
473
Q

What is coombs test ?

A

It is a test that detects antibodies and/or complement proteins on the surface of RBC; (direct test) or in serum (indirect test).

474
Q

What factors affect the prognosis of schizophrenia?

A

Poor prognosis :
- Early onset of illness,
- Family history of schizophrenia.
- Structural brain abnormalities.
- Prominent cognitive symptoms

Better prognosis:
For people living in low-income and middle-income countries.

475
Q

What are the signs of Compensated Shock?

A
  1. Cool extremities
  2. Weak thready peripheral pulse
  3. Delayed capillary refill
  4. Tachycardia in the absence of fever
  5. Narrowing pulse pressure (PP)
476
Q

What are the Morphological characteristics of malignant solitary pulmonary nodules ?

A
  • Large nodule size.
  • Irregular.
  • Spiculated margins.
  • Inhomogeneous density of nodule.
  • Thick walls in cavitary nodules.
477
Q

What ia an Aspergillus ?

A

It is a genus of fungi found ubiquitously in the environment. Transmitted via airborne exposure to mold spores. Risk factors for infection include destructive pulmonary pathology (e.g., COPD) and immunosuppression. Can cause invasive aspergillosis, allergic bronchopulmonary aspergillosis, and aspergilloma.

478
Q

What is the definition of the following terms ?
- Allergic bronchopulmonary aspergillosis (ABPA).
- Chronic pulmonary aspergillosis.
- Invasive aspergillosis.

A
  • Allergic bronchopulmonary aspergillosis (ABPA) :a hypersensitivity reaction caused by exposure to Aspergillus that mostly occurs in patients with cystic fibrosis or asthma.
  • Chronic pulmonary aspergillosis: a long-term Aspergillus infection of the lung which can manifest as:
  • Aspergilloma: an opportunistic infection of a pre-existing cavitary lesion (e.g. from previous tuberculosis)
  • Aspergillus nodule: single or multiple nodules with possible cavitation.
  • Chronic cavitary pulmonary aspergillosis: extensive fibrosis with tissue destruction of at least two lung lobes
  • Fibrotic pulmonary aspergillosis.
  • Chronic necrotizing pulmonary aspergillosis: a subacute and semi-invasive form of chronic pulmonary aspergillosis characterized by localized, slowly progressive, inflammatory destruction of lung tissue and commonly associated with alcohol use disorder.
  • Invasive aspergillosis: a severe form of Aspergillus infection which manifests with severe pneumonia and septicemia with potential involvement of other organs (e.g., skin, CNS) and mostly occurs in immunocompromised individuals.
479
Q

How to treat the following :
- Allergic bronchopulmonary aspergillosis (ABPA).
- Chronic pulmonary aspergillosis.
- Invasive aspergillosis.
??

A
  • Allergic bronchopulmonary aspergillosis (ABPA):
    1. Oral glucocorticoids.
    2. For acute or recurrent ABPA: itraconazole
    3. In the case of sinusitis:
    • Endoscopic drainage.
    • Surgical resection of nasal polyps.
    • Possible nasal wash with antimycotics.
  • Chronic pulmonary aspergillosis:
    1. Aspergilloma and aspergillus nodule: itraconazole OR voriconazole (should be used preoperatively and postoperatively).
    2.Necrotizing pulmonary aspergillosis: voriconazole.
    3. Other chronic pulmonary aspergillosis types:
    • Oral or intravenous voriconazole treatment in mild disease.
    • Posaconazole or amphotericin B in severe disease.
  • Invasive aspergillosis:
    1. Treatment of choice: IV voriconazole.
    2. Alternatives include:
    • Combination with caspofungin if monotherapy fails
    • IV caspofungin OR oral posaconazole.
    • IV amphotericin B OR IV isavuconazole., followed by oral itraconazole.
      3. In HIV- infected patients: antiretroviral therapy.
      4. If patient is taking immunosuppressants: decrease dose of immunosuppressants.
480
Q

What is the definitive treatment of Aspergilloma ?

A

surgical resection (e.g., lobectomy) of the aspergilloma

481
Q

Most common genotype of hepatitis C in saudi arabia?

A

HCV-4.

482
Q

Which antipsychotic is associated with weight gain?

A

Olanzapine and zotepine.

483
Q

Which contraceptives are best while breastfeeding?

A

progestin-only pill (POP).

484
Q

What is the recommendation for urinary tract infection (UTI) screening in pregnant women?

A

The recommendation is that all pregnant women have a screening urine culture at 16 weeks’ gestation. If the results are negative for a UTI, no additional cultures are indicated.

According to the US preventive services task force:
Screening for asymptomatic bacteriuria during pregnancy is done with a urine culture at 12 to 16 weeks of gestation or at the first prenatal visit.

485
Q
  1. What is Primary hyperaldosteronism?
  2. causes ?
  3. Physiology of aldosterone secretion?
  4. What is the mechanism of hypokalemia and metabolic alkalosis ?
  5. What are the clinical features of hyperaldosteronism?
A
  1. It is an autonomous overproduction of aldosterone in the zona glomerulosa of one or both adrenal glands.
  2. Most commonly due to the following conditions:
    - Aldosterone-producing adrenal adenoma (Conn syndrome) or aldosteronoma (50–70%) .
    - Bilateral idiopathic hyperplasia of the adrenal glands (30%).
    Less common causes include:
    - Unilateral hyperplasia of one adrenal gland.
    - Familial hyperaldosteronism.
    - Aldosterone-secreting carcinomas of the adrenal cortex
    - Ectopic aldosterone producing tumors (e.g., in the kidneys, ovaries).

3.
- Physiological aldosterone secretion is regulated by the renin-angiotensin-aldosterone system (RAAS) and occurs in response to the detection of low blood pressure in the kidneys.
- ↑ Aldosterone → ↑ open Na+ channels in principle cells of luminal membrane at the cortical collecting ducts
of the kidneys → ↑ Na+ reabsorption and retention → water retention → hypertension.

    • ↑ Na+ reabsorption → electronegative lumen → electrical gradient through open K+ channels → ↑ K+ secretion → hypokalemia.
    • Hypokalemia → metabolic alkalosis via two mechanisms (both of which decrease extracellular H+, thereby increasing extracellular pH):
    • Efflux of K+ from intracellular to extracellular space in exchange for H+.
      * ↑ H+ secretion in the kidney in order to enable ↑ K+ reabsorption.
      - Diabetes insipidus

: hypokalemia
→ desensitization of renal tubules to antidiuretic hormone (ADH) → polyuria and polydipsia.

    • Hypertension
    • Features of hypokalemia: Fatigue- Muscle weakness /cramping- Headaches - Paresthesia in severe cases due to metabolic alkalosis - Polyuria and polydipsia. - Palpitations - Constipation.
    • Absence of significant edema (due to aldosterone escape).
486
Q
  1. What is the screening test of hyperaldosteronism?
  2. What are the Confirmatory tests of hyperaldosteronism?
A
  1. Plasma aldosterone concentration to plasma renin activity (PAC/PRA ratio):
    - aldosterone is increased and renin is decreased.
    - ↑ PAC (> 15 ng/dL or 416 pmol/L) and ↓ PRA.
    - ↑ PAC/PRA ratio (ratio > 20).
    • Oral sodium loading test:
    • High-sodium diet (5000 mg) or oral sodium chloride tablets (2 g taken three times daily) for 3 days.
    • Followed by 24-hour urine measurements of aldosterone, sodium (to confirm appropriate sodium loading), and creatinine (to assess adequate urine collection.
      * Healthy individuals: RAAS is physiologically suppressed → inhibition of aldosterone secretion.
      * Primary hyperaldosteronism: failure to suppress aldosterone secretion (high urine aldosterone > 12 mcg/day and urine sodium > 200 mEq).
  • Saline infusion test:
    • Procedure: infusion of 2 L of normal saline over 4 hours.
    • Healthy individuals: RAAS is physiologically suppressed → inhibition of aldosterone secretion to plasma concentration < 5 ng/dL (139 pmol/L).
    • Primary hyperaldosteronism: failure to suppress aldosterone secretion (PAC > 10 ng/dL, or 277 pmol/L).
  • Fludrocortisone suppression test:
    • Procedure: administration of fludrocortisone (0.1 mg every 6 h) for a duration of 4 days (with simultaneous replacement of sodium chloride and potassium).
    • Healthy individuals: RAAS is physiologically suppressed → substantial decrease of aldosterone < 50 ng/mL or ≤ 6 ng/dL (measured in upright position at 10am on day 4).
    • Primary hyperaldosteronism: Failure to suppress the aldosterone secretion (serum levels > 50–60 ng/mL or > 6 ng/dL).
  • Captopril suppression test:
    • Procedure: PRA and PAC are measured at baseline and 2 hours after administration of a single dose of captopril (25–50 mg).
    • Healthy individuals: RAAS is physiologically suppressed → inhibition of aldosterone secretion (more than 30% suppression from baseline).
    • Primary hyperaldosteronism: failure to suppress the aldosterone secretion (less than 30% suppression from baseline).
  • Adrenal CT
  • Adrenal venous sampling.
487
Q

Which supplement should be given to infants who are on breast milk only ?

A

Breast milk contains very little iron; therefore, the American Academy of Pediatrics recommends that infants who only receive breast milk (exclusively breastfeed) will need a supplement of iron each day at a dose of 1 milligram of iron for each kilogram of body weight; this supplement of iron should start at 4 months

488
Q

When should an infant have Vit D supplement ?

A

Shortly after birth, most infants will need an additional source of vitamin D. To avoid developing a vitamin D deficiency, the American Academy of Pediatrics recommends breastfed and partially breastfed infants be supplemented with 400 IU per day of vitamin D beginning in the first few days of life.

489
Q

What is the recurrence rate of ectopic pregnancy ?

A

10–20%

490
Q

Which type of renal tubular acidosis RTA comes with Hypokalemia ?

A

type 1 RTA.

491
Q

Which type of renal tubular acidosis RTA comes with Hyperkalemia ?

A

Type 4 is also called hyperkalemic RTA.

492
Q

What is the active form of vitamin D ?
and what are the indications for prescribtion ?

A

Calcitriol (1,25‑dihydroxyvitamin D3).

Indicated to control:
- Hypocalcemia in patients on chronic renal dialysis.
- Secondary hyperparathyroidism in those with chronic kidney disease not yet on dialysis.
- Hypocalcemia in patients with hypoparathyroidism and pseudohypoparathyroidism.

493
Q

What is the clinical presentation of posterior shoulder dislocation?

A
  • The affected arm held in adduction and internal rotation.
  • Efforts by the examiner to abduct or externally rotate the extremity result in pain and very little movement.
  • A prominent acromion and coracoid are typically noted, with the humeral head palpable posteriorly.
494
Q

What is the most common pathogen of bacterial acute otitis media ?

A
  • Streptococcus pneumoniae
  • Haemophilus influenzae.
  • Moraxella catarrhalis
495
Q

What is the Effect of nitric oxide in meconium aspiration syndrome ?

A

In a severe form of meconium aspiration syndrome, we have shown that short-term therapy with inhaled nitric oxide, combined with conventional ventilatory therapy and treatment with surfactant, improves oxygenation, without a significant decrease in pulmonary arterial pressure.

496
Q

what is the difference between Osteomalacia and rickets ?

A
  • Osteomalacia: is a disorder of impaired mineralization of the osteoid.
  • Rickets: is a disorder of impaired mineralization of cartilaginous growth plates.
497
Q

What is the most common cause of both osteomalacia
and rickets ?

A

Vitamin D deficiency resulting from inadequate intake, malabsorption, or lack of exposure to sunlight.

498
Q

What is the Pathophysiology of both osteomalacia
and rickets in :
- Vitamin D deficiency and defective vitamin D metabolism.
- Vitamin D -independent forms ?

A

Vitamin D deficiency and defective vitamin D metabolism:
- Hypocalcemia → defective bone matrix mineralization (osteomalacia) or cartilaginous growth plate mineralization (rickets).
- Hypocalcemia → ↑ PTH levels → ↓ phosphate levels → impaired mineralization.

Vitamin D -independent forms:
- Phosphate deficiency → ↓ phosphate blood levels → defective bone matrix mineralization (osteomalacia) or growth plate mineralization (rickets).

499
Q

What are the clinical features of osteomalacia
and rickets ?

A

osteomalacia:
1. Occurs in adults and children.
2.Bone pain and tenderness.
3.Pathologic fractures.
4.Waddling gait and difficulty walking
5.Myopathy: Muscle weakness - Spasms- cramps.
6. Bone deformity only in very severe cases of osteomalacia.
7. Symptoms of hypocalcemia.

rickets
1. Only occurs in children (growth plates have not fused).
2.Bone deformities: Bending of primarily the long bones
- Distention of the bone-cartilage junctions:
* Rachitic rosary: bead-like distention of the bone cartilage junctions in the ribs.
* Marfan sign: distention of the bone-cartilage junctions in the joints.

  • Craniotabes: softening of the skull.
  • Deformities of the knee, especially genu varum.
  • Increased risk of fracture [10] [11]
  • Harrison groove: depression of the thoracic outlet due to muscle pulling along the costal insertion of the diaphragm.
  • Late closing of fontanelles
  • Impaired growth
  • Symptoms of hypocalcemia
500
Q

How are the labs in osteomalacia and rickets ?

A
  • ↓ Calcium and ↓ phosphate.
  • ↑ Alkaline phosphatase and ↑ PTH.
  • Vitamin D-dependent rickets type 1: ↓ calcitriol concentration
  • Vitamin D-dependent rickets type 2: ↑ calcitriol concentration.
501
Q

How is the imaging findings in osteomalacia and rickets?

A

osteomalacia :
- Looser zones (pseudofractures): transverse bands of radiolucency indicating defective calcification of osteoid.
- Milkman syndrome:
* Multiple, almost symmetrical pseudofractures
in the cortex of bones.
* Visible as radiolucent bands with decreased density on x-ray.
* Ribs, pubic rami, iliac bones, neck of femur, radii and ulnae, and scapula are predominantly affected.

Rickets:
- Growth plates in the metaphysis of the long bones are less defined and show cupping, stippling, and fraying.
- Wide epiphysis.
- In severe cases, Looser zones and fractures.
- Chest X-ray: prominent costochondral junctions.
- Evidence of bone deformities.

502
Q

How to treat osteomalacia and rickets?

A

Vitamin D deficiency: administration of vitamin D
- Also indicated in infants who are exclusively breastfed.
- The healing of both osteomalacia and rickets requires adequate daily intake of calcium.

Defective vitamin D metabolism or vitamin D‑independent forms: treatment of underlying disease.

503
Q

What is the difference between fetal demise or stillbirth and missed abortion ?

A

A death that occurs prior to 20 weeks’ gestation is usually classified as a spontaneous abortion; those occurring after 20 weeks constitute a fetal demise or stillbirth. Many states use a fetal weight of 350 g or more to define a fetal demise.

504
Q

What is the most common childhood leukemias ?

A

Acute lymphocytic leukemia (ALL).

505
Q

What is CHA2DS2-Vasc score?
And what its recommendation?

A

AF is a cardiac arrhythmia with the potential to cause thromboembolism.
Studies suggest that AF increases the risk of stroke five-fold.

Thus, it is important to determine which patients with AF may benefit from oral anticoagulant (OAC) and possibly aspirin therapy to reduce the risk of stroke.

OACs used:
- Warfarin, with a goal International Normalized Ratio (INR) of 2-3 and percent time in therapeutic INR (TTR) remains ≥ 70%
- Dabigatran
- Rivaroxaban
- Apixaban
- Edoxaban

Score
- CHF : 1 point
- HTN. : 1 point
- Age >= 75 years : 2 points
- DM : 1 point
- Stroke/ TIA/ Thromboembolic event : 2 points
- vascular disease: 1 point
- age 64- 75 years : 1 point.
- Sex (Female) : 1 point

Recommendations:
0 - None
1 - Aspirin or OAC
2 or more - OAC.

506
Q

Patients with large hemangiomas should be instructed to avoid what ?

A

Patients with large hemangiomas may need to be instructed to avoid trauma to the right upper abdominal quadrant

507
Q

What is Child-Pugh scoring system (also known as the Child-Pugh-Turcotte score) ?

A
  • Designed to predict mortality in patients with cirrhosis.
  • Three categories:
  • Class A: 5-6 points - good hepatic function.
  • Class B: 7-9 points - moderately impaired hepatic function.
  • Class C: 10-15 points- advanced hepatic dysfunction.

1.Total bilirubin, μmol/L (mg/dL)
* <34: 1 point
* 34-50: 2 points
* >50: 3 points

  1. Serum albumin, g/L
    * >35: 1 point
    * 28-35: 2 points
    * <28: 3 points

3.INR
* <1.7: 1 point
* 1.7-2.3: 2 points
* >2.3: 3 points

4.Presence of ascites
* none: 1 point
* mild: 2 points
* moderate to severe: 3 points

5.presence of hepatic encephalopathy
* none: 1 point
* grades I-II (or suppressed with medication): 2 points
* grades III-IV (or refractory): 3 points

Perioperative mortality:
* Child-Pugh A: 5%
may have up to 50% of liver resected
* Child-Pugh B: 10-15%
no more than 25% of liver resected
* Child-Pugh C: >25%
liver resection contraindicated

508
Q

Apgar score ?

A

The Apgar score is used as a part of early assessment of a newborn.
A score of 0, 1, or 2 is assigned to each of the 5 physical signs at 1 and 5 minutes after birth. The maximum score that can be assigned is 10. Scores ranging from 7-10 are considered normal. If the 5-minute Apgar score is abnormal (< 7), appropriate measures should be taken. Apgar scores should be assigned every 5 minutes until the infant is stabilized.

  1. Breathing effort:
    - Not breathing : 0
    - Slow or irregular: 1
    - Cries well :2
  2. Heart rate:
    - No heartbeat: 0
    - HR is less than 100 bpm: 1
    - HR 100 bpm : 2
  3. Muscle tone:
    - Loose and floppy : 0
    - Some muscle tone: 1
    - Active motion: 2
  4. Grimace response or reflex irritability is a term describing response to stimulation:
    - No reaction: 0
    - Grimacing: 1
    - Grimacing and a cough, sneeze, or vigorous cry: 2
  5. Skin color:
    - Pale blue: 0
    - Pink and the extremities are blue: 1
    - Entire body is pink : 2
509
Q
  1. What is Post-traumatic stress disorder (PTSD) ?
  2. Treatment ?
A
  1. distressing symptoms related to a specific traumatic event and lasting > 1 month following the event.

2.
- Psychotherapy: first-line treatment; with or without adjunctive pharmacotherapy.
* Trauma-focused cognitive-behavioral therapy:
**Exposure therapy; (e.g., showing war; veterans images of war, returning to the scene of an accident).
**
Cognitive processing therapy
* Eye movement desensitization and reprocessing: Under the guidance of a therapist, the patient recalls traumatic images while following the therapist’s fingers with their eyes from left to right.

  • Pharmacotherapy :
  • SSRIs, SNRIs (e.g., venlafaxine)
  • Prazosin: for nightmares
  • Consider atypical antipsychotics to augment SSRIs and SNRIs.
  • Benzodiazepines are rarely used for hyperarousal and anxiety. Their use should be closely monitored because many patients with PTSD are at risk for drug dependence.
510
Q

Name Selective serotonin reuptake inhibitors (SSRI) medications.

A
  • citalopram (Cipramil)
  • dapoxetine (Priligy)
  • escitalopram (Cipralex)
  • fluoxetine (Prozac or Oxactin)
  • fluvoxamine (Faverin)
  • paroxetine (Seroxat)
  • sertraline (Lustral)
  • vortioxetine (Brintellix)
511
Q

What is the Most common bone cancer ?

A

Osteosarcoma – the most common type, which mostly affects children and young adults under 20.

Ewing sarcoma – which most commonly affects people aged between 10 and 20.

512
Q

What is the Infectious agent of croup or bronchiolitis?

A
  • Respiratory syncytial virus (RSV),.
  • Parainfluenza viruses.
  • Influenza
  • Adenoviruses
    are the major causative agents.
  • Parainfluenza type 1 virus is the most common cause of croup (laryngotracheobronchitis).
  • RSV the most common cause of bronchiolitis.
513
Q

What is the skin disease associated with celiac disease ?

A

Dermatitis herpetiformis, also known as DH and Duhring’s disease.

514
Q

Black pigment gallstones usually occour in which type of patients ?

A

Black pigment gallstones occur disproportionately in individuals with high heme turnover. Disorders of hemolysis associated with pigment gallstones include:

  • Sickle cell anemia
  • Hereditary spherocytosis
  • Beta-thalassemia.
    In cirrhosis, portal hypertension leads to splenomegaly
515
Q

Folic acid dose in Folic Acid Deficiency or in Females of childbearing potential ?

A

400 mcg/day PO
(0.4mg)

516
Q

What is the clinical presentation of anterior cerebral artery (ACA) occlusions in ischemic stroke?

A
  1. Disinhibition and speech perseveration
  2. Primitive reflexes (eg, grasping, sucking reflexes)
  3. Altered mental status
  4. Impaired judgment
  5. Contralateral weakness (greater in legs than arms)
  6. Contralateral cortical sensory deficits
  7. Gait apraxia
  8. Urinary incontinence
517
Q

Where is the lesion of Parkinson’s disease ?

A

Substantia nigra

518
Q

What is the risk of GERD post gastric sleeve ?

A

Incidence could increase up to 35%

519
Q

What is the mode of inheritance in cystic fibrosis?

A

Autosomal recessive manner

520
Q

How to adjust SCA treatment during pregnancy ?

A
  • If you take Hydroxyurea on a regular basis, for example, you should stop taking it (three) months before pregnancy, or as soon as you are aware of it.
  • Angiotensin-converting enzyme inhibitors and angiotensin receptor blockers should be stopped before conception.
  • Take a high dose of folic acid (5mg daily) throughout pregnancy to strengthen blood cells.
  • Take a low dose of aspirin (75mg daily) from 12 weeks of gestation in an effort to reduce the risk of developing pre-eclampsia and the risk of toxaemia of pregnancy.
  • You can take painkillers (e.g. Paracetamol).
  • Avoid some painkillers (e.g. Ibuprofen) which may affect the fetus.
  • Women with SCD should be advised to receive prophylactic low-molecular-weight heparin during antenatal hospital admissions.
521
Q

How to treat Candida infections of the mouth, throat, and esophagus ?

A

antifungal medicine:

  • clotrimazole,
  • miconazole
  • nystatin
  • fluconazole in severe cases or if the disesa in esophagus.

for 7 to 14 days

522
Q

When to give pneumococcal vaccine after splenectomy?

A

Ideally patients should receive pneumococcal vaccine from 4 to 6 weeks before elective splenectomy or initiation of chemotherapy or radiotherapy.

If it is not possible, vaccination should be administered at least 2 weeks pre-operatively in elective cases or at least 2 weeks post-operatively in emergency cases.

523
Q

What are the types of headache and what are the symptoms of each one ?

A
  1. Tension headaches:
    Tension headaches are the most common type of headache:
    - Consistent without throbbing.
    - Mild to moderate.
    - On both sides of the head (bilateral).
    - Responsive to over-the-counter treatment.
    - Worse during routine activities (such as bending over or walking upstairs).
  2. Migraines:
    Migraines are the second most common type of primary headaches:
    - Moderate to severe pain.
    - Nausea or vomiting.
    - Pounding or throbbing pain.
    - Pain that lasts four hours to three days.
    - Sensitivity to light, noise or odors.
    - Stomach upset or abdominal pain.
  3. Cluster headaches
    Are the most severe type of primary headache. Cluster headaches come in a group or cluster, usually in the spring or fall. They occur one to eight times per day during a cluster period, which may last two weeks to three months. The headaches may disappear completely (go into remission) for months or years, only to recur later. The pain of a cluster headache is:
    - Intense with a burning or stabbing sensation.
    - Located behind one of your eyes or in the eye region, without changing sides.
    - Throbbing or constant.
524
Q

What are the treatment options of cluster headaches ?

A

ACUTE ATTACK TREATMENT:
- Oxygen in non-rebreathing mask at a high rate of oxygen flow.

  • Triptans:
  • Sumatriptan 6 mg SubQ
  • sumatriptan 20 mg nasal sprays.
  • zolmitriptan 5 mg nasal sprays.
  • Dihydroergotamine IM
  • Lidocaine nasal drops (one mL of 4% lidocaine) can be repeated once after 15 minutes if necessary.

PREVENTIVE TREATMENTS:
- Verapamil, traditionally started at a low dose, which might be increased to achieve treatment outcomes. EKG’s done prior to starting treatment and before the dose is increased.

  • Bridge Therapy: Prednisone as a short-term bridge for 10-21 days, while waiting for the prevention medications to begin to work. It is often given in a gradually decreasing dosage over several weeks. Prednisone should be used no more than once a year to avoid serious side effects.
  • Lithium Carbonate.
  • Topiramate
525
Q

What is Isotretinoin, and is it safe to be taken in pregnancy ?

A

Isotretinoin is used to treat severe recalcitrant nodular acne.

  • Isotretinoin must not be taken by patients who are pregnant or who may become pregnant.
  • You must use two acceptable forms of birth control for 1 month before you begin to take isotretinoin, during your treatment and for 1 month after your treatment.
  • Isotretinoin interferes with the action of micro-dosed progestin (‘minipill’) oral contraceptives.
  • Do not donate blood while you are taking isotretinoin and for 1 month after your treatment.
526
Q

Colonoscopy in ulcerative colitis ?

A

Patients with ulcerative colitis should receive an:
- Initial screening colonoscopy 8 years after a diagnosis of pancolitis.
and
- 12 to 15 years after a diagnosis of left-sided disease.
and then subsequently
- every 1 to 3 years.

527
Q

How to treat atrial fibrillation ?

A
  1. RATE CONTROL:
    - Beta blockers (e.g., metoprolol, esmolol [Brevibloc], propranolol).
    - nondihydropyridine calcium channel blockers (e.g., diltiazem, verapamil).
    - Amiodarone offers another choice for rate control when beta blockers and calcium channel blockers do not work, but its delayed action, potential toxicity, and drug interactions severely limit its use.

** Beta blockers and calcium channel blockers are contraindicated in patients with preexcitation (Wolff-Parkinson-White syndrome).
**
Non-cardioselective beta blockers are also contraindicated in patients with acute heart failure, severe chronic obstructive pulmonary disease, and asthma.

  1. ANTICOAGULATION: depends on CHAD score.
  2. CARDIOVERSION
    4.ABLATION THERAPY
528
Q

What is a Thumb-printing’ radiologic sign ?

A

Thumb-printing is a radiologic sign indicating bowel-wall thickening due to edema that is often a finding in patients with mesenteric ischemia.

529
Q

Definition of flail chest.

A

Definition: 3 or more contiguous ribs are fractured in two or more places.

530
Q

How to treat and manage Lead Toxicity ?

A

In children:
- BLL < 5 µg/dL:
* Repeat BLL in 6–12 months if the child is at high risk for lead exposure or if risk profile increases.
* For children initially screened before 12 months of age, consider retesting in 3–6 months for children at high risk.

  • BLLs: 5- 9 µg/dL:
    • nutritional counseling related to calcium and iron intake.
    • Follow up BLLs.
  • BLLs: 10 - 19 µg/dl:
    • Consider checking blood iron levels.
    • Repeat BLLs within 3 months.
  • BLLs: 20 - 44 µg/dl:
    • Neurodevelopmental assessment.
    • Assessment of blood iron levels.
    • Abdominal X-rays with bowel decontamination if indicated.
    • BLL 20–24 µg/dl : BLL checked in 1 to 3 months
    • BLL 25 - 44 µg/dl: retesting must be done within one month.
  • BLLs 45 - 69 µg/dl:
    • oral chelation therapy.
    • Environmental assessment.
    • Remediation within 48 hours.
    • If a lead-safe environment cannot be assured: hospitalization.
  • BLLs greater than 70 µg/dL:
    • medical emergencies, the risk of encephalopathy is high and treatment is required.

Chelation agents:
- Succimer is a water-soluble, oral chelating agent that is appropriate for use with BLLs higher than 45 µg/dL.

  • D-penicillamine is a second-line oral chelating agent.
  • Edetate (EDTA) calcium disodium (CaNa2 EDTA):
    • parenteral chelating agent.
    • Should never be used in patients with lead encephalopathy, because it does not cross the blood-brain barrier.
    • Life-threatening hypocalcemia has been reported.
  • Dimercaprol (also referred to as British antilewisite [BAL]):
    • Parenteral chelating agent.
    • Agent of first choice for patients with lead encephalopathy + high BLLs (ie, > 100 µg/dL), it is used in conjunction with CaNa2 EDTA.

*** In adults, consider chelation therapy for patients with blood lead levels BLLs of 70 µg/dL or higher. Also consider chelation therapy in symptomatic adults with BLLs exceeding 50 µg/dL

531
Q

Most common pathogen in Prosthetic valve endocarditis?

A
  • S aureus.
  • S epidermidis.
532
Q

Which muscles of pelvic floor is affected in urge urinary incontinence?

A

Detrusor muscle

533
Q

Name the muscles of the pelvic floor.

A

Levator Ani Muscles:
- pubococcygeus
- puborectalis.
- iliococcygeus.

534
Q

cervical insufficiency definition.

A

The inability of the uterine cervix to retain a pregnancy in the absence of the signs and symptoms of clinical contractions, or labor, or both in the second trimester.”

535
Q

What are the indication for cervical cerclage ?

A
  • History: ≥1 of the following:
    1. Second trimester pregnancy losses related to painless cervical dilation and no history of labor or abruption.
    2. Previous second trimester cerclage for painless cervical dilation.
  • PEx: Also known as ‘physical examination–indicated cerclage’, ‘rescue cerclage’ and ’emergency cerclage’
    Patient presents with painless second trimester cervical dilation (cervical dilatation >1cm on exam).
  • Ultrasound: Cervical length shortening and history of preterm birth:
    1. Singleton pregnancy
    2. Prior spontaneous preterm birth (<34 weeks)
    3. Cervical length: <25 mm (at <24 weeks).
536
Q

Is Celiac Screening for Kids With Type 1 Diabetes Adequate?

A

The International Society for Pediatric and Adolescent Diabetes (ISPAD) recommends celiac screening
“at diagnosis and then annually for 5 years.”

537
Q

How to treat depression in children ?

A
  • PSYCHOTHERAPY
  • Fluoxetine (Prozac), citalopram (Celexa), and sertraline (Zoloft) as first-line treatments for moderate to severe depression in children.
  • Escitalopram (Lexapro) is also licensed for the treatment of depression in adolescents 12 years and older
538
Q

Is ACE inhibitors safe in pregnancy ?

A

It is well accepted that angiotensin-converting enzyme (ACE) inhibitors are contraindicated during the second and third trimesters of pregnancy because of increased risk of fetal renal damage.
First-trimester use, however, has not been linked to adverse fetal outcomes.

539
Q

Omeprazole side effects long term use ?

A
  • bone fractures.
  • gut infections.
  • vitamin B12 deficiency
540
Q

What are the chemical mediators in thermal burns ?

A

Prostaglandins, histamine, and bradykinin.

541
Q

Rectal adenoma colonoscopy follow up?

A

Patients with one or two tubular adenomas that are smaller than 10 mm should have a repeat colonoscopy in five to 10 years.
Repeat colonoscopy at five years is recommended for patients with nondysplastic serrated polyps that are smaller than 10 mm

542
Q

Name the medications that causes Low efficacy of clopidogrel .

A

Low efficacy of clopidogrel is anticipated in the presence of:
- omeprazole
-esomeprazole
- morphine
- grapefruit juice
- scutellarin
- fluoxetine
- azole antifungals
- calcium channel blockers
- sulfonylureas
- ritonavir

543
Q

What it is the antibiotic of choice in patients with acute otitis media ?

A

High-dose amoxicillin (80 to 90 mg per kg per day) is the antibiotic of choice for treating acute otitis media in patients who are not allergic to penicillin.

544
Q

Miscarriage rates by age ?

A

The average risk of miscarriage by the age of the mother is as follows:
- Under 35 years old: 15 percent chance of pregnancy loss.
- Between 35–45 years old: Between 20 and 35 percent chance of pregnancy loss.
- Over 45 years old: About a 50 percent chance of pregnancy loss.

545
Q

What is the risk of having an ectopic pregnancy again?

A

The chances of having another ectopic pregnancy are higher if you’ve had one before, but the risk is still small (around 10%).

546
Q

What is the treatment of croup ?

A

A single dose of dexamethasone (0.15 to 0.60 mg per kg usually given orally) is recommended in all patients with croup, including those with mild disease.

Nebulized epinephrine is an accepted treatment in patients with moderate to severe croup.

547
Q

What is the type of HBV vaccine ?

A

Recombinant

548
Q

What is the most common pathogen of Acute otitis externa (swimmer’s ears) ?

A

The acute form is caused primarily by bacterial infection, with:
- Pseudomonas aeruginosa and
- Staphylococcus aureus
the most common pathogens.

549
Q

At which week does elective caesareans are usually done ?

A

39th week of pregnancy.

550
Q

how to treat lymphangitis ?

A
  • Analgesics can help to control pain.
  • Anti-inflammatory medications can help to reduce inflammation and swelling.
  • Antibiotics, used in the treatment of group A beta-hemolytic streptococci (GABHS) and S aureus infections:
    • Dicloxacillin
    • Cephalexin
    • Cefazolin
    • Cefuroxime
    • Ceftriaxone
    • Clindamycin
    • Nafcillin
    • Trimethoprim and sulfamethoxazole (TMP/SMZ)
551
Q

Which antibiotic is used in prophylaxis in prosthetic valve before dental procedure ?

A

The guidelines recommend 2 grams of amoxicillin given orally as a single dose 30-60 minutes before the procedure as the drug of choice for infective endocarditis prophylaxis. Amoxicillin has been shown to be effective in reducing bacteraemia related to dental procedures

552
Q

Which tocolytic drug increases postnatal patent ductus arteriosus ?

A

Indomethacin

553
Q

How to treat UTI in pediatrics ?

A

The recommended initial antibiotic for most children with UTI is:
- trimethoprim/sulfamethoxazole (Bactrim, Septra). Alternative antibiotics include:
- amoxicillin/clavulanate (Augmentin) or
- cephalosporins, such as cefixime (Suprax), cefpodoxime, cefprozil (Cefzil), or cephalexin (Keflex).

554
Q

Crohn’s disease patient. post bowel resection , c/o diarrhea with normal labs.
Which medication to give and why?

A

Patients with crohn’s disease usually have the disease in the ileum, so bowel resection mostly is in the ileum.

Patients with terminal ileal disease or previous terminal ileal resection may not absorb bile acids normally, and this abnormality can lead to secretory diarrhea in the colon. These patients may benefit from bile acid sequestrants (eg, cholestyramine or colestipol).

555
Q

What is Congenital adrenal hyperplasia (CAH) and what is the mode of transmission.

A

It is one of the hereditary diseases with autosomal recessive inheritance resulting from a deficiency of steroid 21-hydroxylase (21-OH). It is characterized by cortisol deficiency, with or without aldosterone deficiency, and androgen excess.

556
Q

What are the indications for cesarean delivery in patients with eclampsia?

A

Irrespective of gestational age, a prolonged induction with clinically significant worsening of maternal cardiovascular, hematologic, renal, hepatic, and/or neural status is generally an indication for cesarean delivery when the anticipated delivery time is remote.

557
Q

Who is a candidate for expectant management in pre-eclampsia ?

and who is a candidates for delivery (expectant management not advised) in pre-eclampsia ?

A

Expectant management :
1. Gestational hypertension or preeclampsia without severe features <37w0d.
2. Reassuring antenatal testing
3. Intact membranes
4.No vaginal bleeding
5.No evidence of active preterm labor.

Delivery:
1. Severe range hypertension unresponsive to antihypertensive agent(s).
2. Persistent headache or persistent RUQ/epigastric pain unresponsive to treatment.
3.Visual disturbance or altered sensorium or motor deficit.
4. Stroke or MI.
5.HELLP syndrome
6.Worsening renal function (Cr above 1.1 or double the baseline).
7.Pulmonary edema
8. Eclampsia
9. Placental abruption or bleeding in the absence of placenta previa.
10.Abnormal antenatal testing.
11.Fetal demise.
12.Fetal lethal anomaly or extreme prematurity.
13. UA Doppler REDF

Expectant Management for Severe Preeclampsia:
** ≥34w0d: Delivery is recommended:
- Do not delay delivery to administer steroids in late preterm
**
<34w0d: Expectant management for women who are clinically stable:
- Associated with higher GA (on average 1-2 weeks) at delivery( Improved neonatal outcomes).
- Requires close maternal and fetal monitoring with serial laboratory testing.
- Deliver if maternal or fetal status deteriorates
- Corticosteroid administration is recommended
“May not always be advisable” to delay delivery when indicated to provide full steroid course

558
Q
  1. Definition of fibroids.
  2. What are the symptoms of fibroids?
A
  1. Definition:
    It is a benign (not cancer) growths that develop from the muscle tissue of the uterus. They also are called leiomyomas or myomas.
  2. symptoms of fibroids:
    - Changes in menstruation:
    • Longer, more frequent, or heavy menstrual periods
    • Menstrual pain (cramps)
    • Vaginal bleeding at times other than menstruation
    • Anemia (from blood loss)
      - Pain
    • In the abdomen or lower back (often dull, heavy and aching, but may be sharp)
    • During sex
      - Pressure
    • Difficulty urinating or frequent urination
    • Constipation, rectal pain, or difficult bowel movements
    • Abdominal cramps
      - Enlarged uterus and abdomen
      - Miscarriages
      - Infertility
      Fibroids also may cause no symptoms at all. Fibroids may be found during a routine pelvic exam or during tests for other problems.
559
Q

What is the first- line conservative surgical therapy for the management of symptomatic intracavitary fibroids.

A

Hysteroscopic myomectomy.

560
Q

When to offer uterine artery occlusion in patients with uterine fibroids?

A

Uterine artery occlusion by embolization or surgical methods may be offered to selected women with symptomatic uterine fibroids who wish to preserve their uterus. Women choosing uterine artery occlusion for the treatment of fibroids should be counselled regarding possible risks, including the likelihood that fecundity and pregnancy may be impacted.

561
Q

What is Wilson disease?
What is the mainstay of therapy for Wilson disease?

A

It is a rare autosomal recessive inherited disorder of copper metabolism that is characterized by excessive deposition of copper in the liver, brain, and other tissues. Wilson disease is often fatal if not recognized and treated when symptomatic.

Mainstay of therapy for Wilson diseas is D-penicillamine and trientine.

562
Q

What are the most common intra-abdominal tumors in children ?

A
  • Neuroblastoma.
  • Wilms’tumor.
  • Hepatoblastoma.
  • Teratoma.
  • Granulosa cell tumor.
563
Q

Haloperidol antidote ?

A

Bromocriptine or Ropinirole may be used to treat the extrapyramidal effects caused by haloperidol, acting as dopamine receptor agonists.

564
Q

When to screen for group b streptococcus in pregnancy?

A

between 36 and 38 weeks of pregnancy. In this test, a swab is used to take a sample from the vagina and rectum.

565
Q

How to approach bladder truama ?

A
  • Most extraperitoneal bladder leaks can be effectively managed with maximal bladder drainage per urethral or suprapubic catheter.
  • Depending on the presumed size of the bladder defect, the bladder should be drained for 10 to 14 days and then assessed for healing via cystogram.
  • In a trauma situation, blood at the urethral meatus is an absolute indication for retrograde urethrography.
  • Only after urethral injury is excluded should urethral catheter placement be attempted. In the setting of a posterior urethral injury, placement of a suprapubic (cystotomy) tube, via an open or percutaneous approach, is generally pursued.
  • Posterior urethral injury is a contraindication to urethral catheter insertion. Such an injury should be suspected if blood is present at the urethral meatus, in all pelvic fractures, or if a high-riding prostate is found on digital rectal examination.
566
Q

Eradication of Helicobacter pylori definition ?

A

The presence of negative tests for H pylori 4 weeks or longer after the end of antimicrobial therapy.

567
Q

Pansystolic murmur DDx ?

A

They are usually due to regurgitation in cases such as
- mitral regurgitation
- tricuspid regurgitation
- ventricular septal defect (VSD).

568
Q

What is the definition of Antepartum haemorrhage (APH) ?

A

bleeding from or in to the genital tract, occurring from 24+0 weeks of pregnancy and prior to the birth of the baby.

569
Q

Antidote of heparin ?

A

protamine sulfate

570
Q

Antidote of Warfarin ?

A

vitamin K

571
Q

What are the complications of pharyngeal pouch surgery?

A
  1. Recurrent laryngeal nerve damage.
  2. Pouch perforation.
  3. Mediastinitis.
  4. Pharyngeal fistula.
  5. Wound infection.
  6. Pharyngeal stenosis.
  7. Recurrence of pouch.
572
Q

What is the standard initial treatment of antiphospholipids syndrome ?

A

Heparin and warfarin.

573
Q

Absence seizure treatment ?

A
  • Ethosuximide (Zarontin).
  • Lamotrigine (Lamictal).
  • Valproic acid (Depakene).
  • Divalproex sodium (Depakote)
574
Q

What are the categorized of prevention ?

A
  1. Primary Prevention—intervening before health e ects occur, through measures such as vaccinations, altering risky behaviors (poor eating habits, tobacco use), and banning substances known to be associated with a disease or health condition.
  2. Secondary Prevention—screening to identify diseases in the earliest stages, before the onset of signs and symptoms, through measures such as mammography and regular blood pressure testing.
  3. Tertiary Prevention—managing disease post diagnosis to slow or stop disease progression through measures such as chemotherapy, rehabili- tation, and screening for complications.
575
Q

What is Charcot’s triad ?

A

Acute cholangitis triad :
1. fever
2.RUQ pain
3.jaundice

576
Q
  1. What is Human echinococcosis ?
  2. What are the forms of echinococcosis ?
  3. Signs and symptoms ?
A
  1. It is a zoonotic disease (a disease that is transmitted to humans from animals) that is caused by parasites, namely tapeworms of the genus Echinococcus.
  2. There are 4 forms:
    - cystic echinococcosis ( hydatid disease or hydatidosis), caused by Echinococcus granulosus.
    - alveolar echinococcosis, caused by infection with E. multilocularis.
    - the 2 others are :polycystic caused by: E. vogeli; and
    unicystic caused by E. oligarthrus.
  3. Signs and Sx:
    Cystic echinococcosis / hydatid disease:
    - Development of one or more hydatid cysts located most often in the liver and lungs, and less frequently in the bones, kidneys, spleen, muscles and central nervous system.
    - Liver: Abdominal pain, nausea and vomiting.
    - Lung: chronic cough, chest pain and shortness of breath.

Alveolar echinococcosis
- slow development of a primary tumour-like lesion which is usually located in the liver. Clinical signs include weight loss, abdominal pain, general malaise and signs of hepatic failure.
- Larval metastases may spread either to organs adjacent to the liver (for example, the spleen) or distant locations (such as the lungs, or the brain) following dissemination of the parasite via the blood and lymphatic system. If left untreated, alveolar echinococcosis is progressive and fatal.

577
Q
  1. How to diagnose echinococcosis (hydatid disease and Alveolar echinococcosis) ?
  2. Treatment ?
A
  1. Dx:
    - US
    - CT
    - MRI
  2. Treatment:
    - percutaneous treatment of the hydatid cysts with the PAIR (Puncture, Aspiration, Injection, Re-aspiration) technique:
    • Non-echoic lesion ≥ 5 cm in diameter .
    • Cysts with daughter cysts and/or with detachment of membranes
    • Multiple cysts if accessible to puncture
    • Infected cysts
  • Surgery
  • Anti-infective drug treatment:
    *Albendazole and mebendazole are the only anthelmintics effective against cystic echinococcosis.
    • Albendazole is the drug of choice.
  • Watch and wait.
578
Q

What is the classification of BI-RADS (Breast Imaging-Reporting and Data System) ?

A
  • BI-RADS 0:
  • incomplete
  • Need to review prior studies and/or complete additional imaging.
  • BI-RADS 1:
  • negative
  • symmetrical and no masses, architectural distortion, or suspicious calcifications.
  • Continue routine screening.
  • BI-RADS 2:
  • benign
  • 0% probability of malignancy.
  • Continue routine screening.
  • BI-RADS 3:
  • probably benign
  • <2% probability of malignancy
  • Short-term follow-up mammogram at 6 months, then every 6 to 12 months for 1 to 2 years.
  • BI-RADS 4:
  • suspicious for malignancy
  • 2-94% probability of malignancy
  • for mammography and ultrasound
  • these can be further divided:
    BI-RADS 4A: low suspicion for malignancy (2-9%)
    BI-RADS 4B: moderate suspicion for malignancy (10-49%)
    BI-RADS 4C: high suspicion for malignancy (50-94%)
  • Perform biopsy, preferably needle biopsy.
  • BI-RADS 5:
  • highly suggestive of malignancy
  • > 95% probability of malignancy
  • Biopsy and treatment, as necessary.
  • BI-RADS 6:
  • known biopsy-proven malignancy.
  • Assure that treatment is completed.
579
Q

How to monitor AAAs ?

A
  • Patients with AAAs 3.0 to 3.9 cm in diameter should be monitored with ultrasonography every two to three years.
  • Patients with AAAs 4.0 to 5.4 cm in diameter should be monitored with ultrasonography or computed tomography every six to 12 months.
  • > 5.4 cm : Surgical consultation for elective repair.
580
Q

Which type of imaging is used to screen for AAA ?

A

US abdomen.

581
Q

What are the risk factors of Abdominal Aortic Aneurysm ?

A
  • Atherosclerosis
  • Cerebrovascular disease
  • Coronary artery disease
  • First-degree relative with abdominal aortic aneurysm
  • History of other vascular aneurysms.
  • Hypercholesterolemia
  • Hypertension
  • Male sex
  • Obesity
  • Older age
  • Tobacco use
582
Q

What are the indications of surgery in Abdominal Aortic Aneurysm AAA ?

A
  • When the aneurysm exceeded 5.5 cm.
  • Expanded by more than 1 cm per year.
  • Became tender or symptomatic.
583
Q

How to treat Abdominal Aortic Aneurysm AAA ?

A
  • Smoking cessation (because smoking causes increase in size of aneurysm).
  • Beta blockers are known to improve perioperative mortality for AAA repair. But their effects on AAA enlargement are not signicant.
  • Surgically when indicated:
    • Open repair
    • Endovascular repair.
584
Q

What is Bethesda classification ?

A

Bethesda classification system for thyroid fine needle aspirates comprises of six (6) categories in pathological reporting of thyroid FNA’s:

  • Category I: non diagnostic :
    Repeat FNA with US guidance.
    Risk of malignancy: 1-4
  • Category II: benign:
    Clinical follow up
    Risk of malignancy:0-3
  • Category III: atypia of undetermined significance:
    Repeat FNA.
    Risk of malignancy: 5 - 15
  • Category IV: suspicious for follicular neoplasm:
    Surgical lobectomy.
    Risk of malignancy: 15 -30
  • Category V: suspicious for malignancy:
    Near - total thyroidectomy or surgical lobectomy.
    Risk of malignancy: 60- 75
  • Category VI: malignant:
    Near - total thyroidectomy.
    Risk of malignancy: 97 - 99
585
Q

What is the favorable place for AV fistula ?

A

The radial cephalic arteriovenous at the wrist is the recommended first choice for hemodialysis access.

586
Q

What are the treatment recommendations for localized resectable disease of GISTs ?

A
  • Surgical resection is the primary treatment; for tumors < 2 cm with no high-risk features (eg, irregular border, cystic spaces, ulceration, echogenic foci, heterogeneity), consider endoscopic surveillance at 6-12 month intervals.
  • Consider preoperative imatinib only if surgical morbidity could be reduced by downstaging the tumor preoperatively.
  • Testing the tumor to confirm it has a genotype that is likely to respond to imatinib (especially KIT exon 11) is recommended prior to starting preoperative imatinib.
  • Consider postoperative imatinib for at least 36 months for high-risk tumors.
  • Patients with small esophagogastric or duodenal nodules < 2 cm undergo endoscopic ultrasound assessment and then annual follow-up, reserving excision for patients whose tumor increases in size or becomes symptomatic.
587
Q

What is the definitive treatment of sigmoid and cecal volvulus ?

A

Surgery.

588
Q

How to approach sigmoid volvuls ?

A
  • In patients with no evidence of peritonitis or ischemic bowel: resuscitation + detorsion of the sigmoid volvulus by sigmoidoscopy or colonoscopy and concomitant rectal tube placement. The bowel is then prepared, and surgery is undertaken electively during the same hospitalization.
  • Inability to detorse the sigmoid volvulus endoscopically is an indication for immediate surgical intervention.
  • If the patient has evidence of peritonitis or ischemic bowel, emergency surgery is indicated, and the operative procedure is chosen on the basis of intraoperative findings.
  • Radiologic diagnoses of cecal volvulus or cecal bascule are considered indications for surgical intervention because the obstruction in these conditions cannot be reliably reduced with colonoscopy. Right hemicolectomy with primary ileocolic anastomosis is the procedure of choice. However, a single attempt at colonoscopic decompression is reasonable.
  • For sigmoid volvulus, the currently accepted surgical procedures include sigmoid resection with primary anastomosis and resection and the Hartmann procedure. Primary anastomosis is performed if the divided bowel ends are viable, peritoneal contamination is not evident, and the patient is hemodynamically stable. If evidence of ischemic bowel or gross peritoneal contamination is observed or if the patient is hemodynamically unstable, a Hartmann procedure is safer.
589
Q

Infected pseudocyst management ?

A

Percutaneous catheter drainage is the procedure of choice for treating infected pseudocysts, allowing for rapid drainage of the cyst and identification of any microbial organism.

590
Q

Most common site of thoracic aortic blunt trauma?

A

Usually occurs at the aortic isthmus just distal to the left subclavian artery.

591
Q

What is Cholecystokinin ? and when does it increases ?

A

It is a peptide hormone of the gastrointestinal system responsible for stimulating the digestion of fat and protein.

CCK is the primary hormone responsible for gallbladder contraction and has potent effects on pancreatic secretion, gastric emptying, and satiet.

So it will increase with biliary colic.

592
Q
  1. What is Angiodysplasia ?
  2. How to investigate ?
  3. Management ?
A
  1. Angiodysplasia is the most common vascular lesion of the gastrointestinal tract, and this condition may be asymptomatic, or it may cause gastrointestinal (GI) bleeding.The vessel walls are thin, with little or no smooth muscle, and the vessels are ectatic and thin.
  2. Labs :
    - CBC
    - Serum iron level
    - Stool for occult blood.

Imaging/procedures:

  • Initially: Endoscopy (colonoscopy, with or without upper endoscopy, and wireless video capsule endoscopy).
  • Endoscopy is the most common method of diagnosing angiodysplasia in both the upper and lower GI tract.
  • Angiography (usually preceded by a bleeding scan to guide angiography).
  • If massive bleeding: deep small bowel enteroscopy, or even intraoperative enteroscopy.
  1. Management:
    - Conservative approach to patients who are hemodynamically stable is recommended, because most bleeding angiodysplasias will cease spontaneously.
  • Asymptomatic angiodysplasias which are found incidentally doesn’t need to be treated.
  • Gastric and duodenal angiodysplastic lesions:
    **endoscopic obliteration techniques:
    • Monopolar electrocautery.
    • Heater probe.
    • Sclerotherapy.
    • Band ligation.
    • Argon and neodymium:yttrium-aluminum-garnet (Nd:YAG) lasers.
      ** Partial or complete gastrectomy.
  • Acute arterial bleeding: Emergency embolization.
  • Cecum and right colon:
  • Endoclips.
  • Right hemicolectomy for angiodysplasia is a second-line therapy after endoscopic ablation.
  • Surgical resection is the definitive treatment for angiodysplasia.
  • Surgical resection is preferred for the acute management of severe hemorrhage or for the management of recurrent hemorrhage over a relatively short period accompanied by a large transfusion requirement.
593
Q

What is the relationship between splenectomy and insulin ?

A

Increasing evidence suggests that the spleen harbors stem cells that act as precursors to insulin-producing pancreas cells. Additionally, small studies with short-term follow-up associate splenectomy with increased rates of diabetes mellitus.

594
Q

How to repair CBD injury ?

A

The most popular surgical repair is the Roux-en-Y hepaticojejunostomy.

There are many described techniques for complex biliary injury repairs including:
- primary repair
- primary end to end anastomosis of bile duct.
- choledochoduodenostomy.
- cholecystojejunostomy.

595
Q
  • What are the symptoms of superior laryngeal nerve injury?
  • What are the symptoms of recurrent laryngeal nerve injury?
A
  • superior laryngeal nerve injury:
    changes the pitch of the voice and causes an inability to make explosive sounds due to paralysis of the cricothyroid muscle.
  • recurrent laryngeal nerve injury:
    unilateral vocal cord paralysis. Patients with this typically complain of new-onset hoarseness, changes in vocal pitch, or noisy breathing.
596
Q

Barrett’s esophagus management

A

According to SMLE Q notes:
- Barrett alone (metaplasia):
PPIs and rescope every 2–3 years.

  • Low-grade dysplasia:
    PPIs and rescope every 6–12 months.
  • High-grade dysplasia:
    Ablation with endoscopy: photodynamic therapy, radiofrequency ablation, endoscopic mucosal resection)
597
Q

When does ovulation occur in relation to the LH peak?

A

Ovulation is considered to occur 28-36 hours after the beginning of the LH rise.

or 8-20 hours after the LH peak.

598
Q
  1. What is Cholesterol embolism?
  2. When to suspect a Cholesterol embolism ?
  3. Workup ?
  4. How to manage ?
A
  1. Cholesterol embolism is a type of embolism resulting from fracture of an atherosclerotic plaque.
  2. Cholesterol embolism syndrome (CES) should be suspected in a patient who:
    - develops worsening renal function,
    - hypertension
    - distal ischemia
    - acute multisystem dysfunction after an invasive arterial procedure.
    - May occur spontaneously.
  3. Workup:
    * Demonstration of cholesterol crystals in occluded arterioles is the only definitive test for cholesterol embolism

labs:
- CBC
- Chemistry: High renal function
- Urinalysis.
- Tissue-specific laboratory tests: elevated creatine kinase (CK) level; myocardial, pancreatic, and hepatobiliary involvement produces increases in cardiac enzymes, amylase, and hepatobiliary enzymes
- Inflammatory mediators

Imaging:
- Angiography: to rule out more treatable causes of tissue ischemia, but it may induce atheroembolism.
- Transesophageal echocardiography (TEE).
- CT
- MRI

  1. Management:
    Medical : Supportive
    Surgical:
    - Surgical therapy (eg, aortic aneurysm resection) may be necessary to remove the source of atheroembolic material.
    - Stent-grafting may be a less invasive method to reduce embolization risk.
599
Q

What are the additional risks to the woman and baby in SCA pregnancy ?

A
  • Increased incidence of perinatal mortality.
  • Premature labour.
  • Fetal growth restriction.
  • Acute painful crises during pregnancy.
  • Increase in spontaneous miscarriage.
  • Antenatal hospitalisation.
  • Maternal mortality.
  • Delivery by caesarean section.
  • Infection.
  • Thromboembolic events.
  • Antepartum haemorrhage.
  • +/- Increased risk of pre-eclampsia and pregnancy-induced hypertension.
600
Q

What is the role of blood transfusion during SCA pregnancy?

A
  • Routine prophylactic transfusion is not recommended during pregnancy for women with SCD.
  • Haemoglobin under 6 g/dl or a fall of over 2 g/dl from baseline is often used as a guide to transfusion requirement.
  • Exchange transfusion is also indicated for acute stroke.
  • Blood should be matched for an extended phenotype including full rhesus typing (C, D and E) as well as Kell typing if necessary.
  • Blood used for transfusion in pregnancy should be cytomegalovirus negative.
601
Q

What is the optimal management of acute painful crisis during pregnancy?

A
  • Pregnant women presenting with acute painful crisis should be rapidly assessed by the multidisciplinary team and appropriate analgesia should be administered.
  • Pethidine should not be used because of the associated risk of seizures.
  • The requirement for fluids and oxygen should be assessed, and fluids and oxygen administered if required.
  • Thromboprophylaxis should be given to women admitted to hospital with acute painful crisis.
  • Women admitted with sickle cell crisis should be looked after by the multidisciplinary team, involving obstetricians, midwives, haematologists and anaesthetists.
602
Q

How to treat Traveler’s Diarrhea?

A
  • Antibiotics (usually a quinolone) should be used to reduce the duration and severity of traveler’s diarrhea.
  • Bismuth subsalicylate Two 262-mg tablets , four times daily for up to three weeks.

**Antibiotic prophylaxis should not be used routinely in persons at risk of developing traveler’s diarrhea.

603
Q

What are the Common Causes of Traveler’s Diarrhea?

A

Bacteria:
- Enterotoxigenic Escherichia coli
- Other E. coli types (e.g., enteroaggregative E. coli)
- Campylobacter
- Salmonella (non-typhoid)
- Shigella
- Aeromonas
- Vibrio (non-cholera)
Parasites
- Giardia lamblia
- Entamoeba histolytica
- Cyclospora cayetanensis
- Cryptosporidium parvum
Viruses
- Rotavirus
- Noroviruses

604
Q

What are the types of colorectal surgeries and thier indications ?

A
  1. Right hemicolectomy:
    - The ileocolic, right colic, and right branch of the middle colic vessels are divided and removed with the contiguous mesentery.
    - ileocolic anastomosis is typically performed.
  2. Extended right hemicolectomy:
    - The ileocolic, right colic, and middle colic vessels with their contiguous mesentery are divided and removed. The inferior mesenteric vein may be divided and included in the specimen.
    - Anastomosis between the ileum and the distal transverse colon.
  3. Transverse colectomy:
    - The transverse colon is resected along with the middle colic vessels and its mesentery.
  4. Left hemicolectomy:
    - Appropriate for tumors in the distal transverse or descending colon and for selected patients with proximal sigmoid colon cancer.
    - The left branch of the middle colic vessels, the inferior mesenteric vein, and the left colic vessels along with their mesenteries are included with the specimen.
  5. Sigmoid colectomy:
    - For sigmoid colon cancers.
    - The inferior mesenteric artery is divided at its origin, and dissection proceeds just under the superior rectal vessels toward the pelvis until adequate margins are obtained.
  6. subtotal colectomy:
    - Removing the ascending (right side) colon and the descending (left side) colon. The small intestine will be attached to the sigmoid colon or the rectum. You may need to have an ostomy pouch.
  7. low anterior resection:
    - removes the rectum and the sigmoid colon.
  8. abdomino-perineal resection:
    - removes the rectum and anus. Since these parts of the colon are removed, you will no longer have an anus. You will need to have an ostomy pouch.
  9. Hartmann’s procedure/proctosigmoidectomy:
    - surgical resection of the rectosigmoid colon with closure of the anorectal stump and formation of an end colostomy
605
Q

Which parts of the colon are retroperitoneal and which are intraperitoneal structures ?

A

Retroperitoneal:
- Right colon
- Left colon

Intraperitoneal:
- Transverse colon
- Sigmoid

606
Q

How long is the rectum ?

A

12 cm to 15 cm.

607
Q

What is Goodpasture syndrome ?

A

It is a diffuse pulmonary hemorrhage and acute or rapidly progressive glomerulonephritis.
Goodpasture disease is a term used to describe glomerulonephritis, with or without pulmonary hemorrhage, and the presence of circulating anti–glomerular basement membrane (anti-GBM) antibodies.

608
Q

How to diagnose and treat Goodpasture syndrome ?

A

kidney biopsy is the best method for detecting anti-GBM antibodies in tissues.

Treatment:
The treatment of choice is plasmapheresis in conjunction with prednisone and cyclophosphamide.

The three main goals for the treatment are:
1. Rapidly remove circulating antibody, primarily by plasmapheresis.
2. Stop further production of antibodies using immunosuppression with medications, namely, corticosteroids (e.g., prednisone) and cyclophosphamide.
The duration of the immunosuppressive treatment varies but is typically 6 months for corticosteroids and 3 months for cyclophosphamide.

  1. Remove offending agents that may have initiated the antibody production.
609
Q

What is benign rolandic epilepsy?

A

It is the most common form of childhood epilepsy. It is referred to as “benign” because most children outgrow the condition by puberty. This form of epilepsy is characterized by seizures involving a part of the brain called the rolandic area.

EEG usually showes : Centrotemporal Spikes

610
Q

Patient suspected to have TB pleural effusion whats is the most sensitive test that supports the diagnosis ?

(According to SMLE notes)

A

High pleural fluid Adenosine deaminase ADA levels.

611
Q

Name the nerves that could be injured during breast surgery including mastectomy ?

A
  • Intercostobrachial nerve
  • Pectoral nerves.
  • Segmental intercostal nerves.
612
Q

What are the symptoms of intercostobrachial nerve injury ?

A

Constant paresthesias and dull, aching, or burning pain of the upper arm, shoulder, and axilla, and occasionally of the more anterior chest wall.

613
Q

basal cell carcinoma common site ?

A

Basal cell carcinoma occurs most often on areas of the skin that are exposed to the sun, such as your head and neck.

614
Q

What are the Recommendations during pregnancy in patients with SLE ?

A
  • Hydroxychloroquine (plaquenil) – We continue use of hydroxychloroquine (HCQ) during pregnancy in all patients with SLE.
  • Low-dose aspirin started at 12 weeks gestation, to reduce the risk of preeclampsia and its sequelae.
  • NSAID use is not associated with congenital anomalies, but consider avoiding these medications during the first trimester.(FDA) recommends that the lowest dose and shortest duration of NSAID be used between 20 and 30 weeks of gestation.
  • Glucocorticoids are used with the lowest possible dose of prednisone, ideally less than 10 mg/day.
  • Azathioprine is compatible with pregnancy.
  • Cyclosporine used when maternal benefit outweighs fetal risk.
  • Antihypertensive medications – Methyldopa, labetalol, nifedipine, and hydralazine are the most commonly used antihypertensives in pregnancy.
    Angiotensin-converting enzyme (ACE) inhibitors and angiotensin II receptor blockers are contraindicated during pregnancy.
  • Diuretics should be used with caution. Nitroprusside is the agent of last resort for urgent control of refractory severe hypertension; its use should be limited to a short period of time in an emergency situation.
  • Methotrexate – Methotrexate is teratogenic and should not be used during pregnancy.
  • Leflunomide: should be avoided during pregnancy.
  • Mycophenolate Mofetil (MMF) : should be avoided during pregnancy as it is associated with an increased risk of malformations and first-trimester pregnancy loss. The most frequent malformations include external ear and other facial malformations such as cleft palate and lip.
615
Q

What is the most common obstetric complication in women with SLE ?

A

Preterm birth.

**The presence of lupus nephritis and active disease are the strongest predictors for early delivery.

616
Q

Name the Fetal complications during pregnancy in patients with SLE ?

A
  • Miscarriage
  • Stillbirth
  • Growth restriction
  • Neonatal lupus (NL) syndromes
    -Complications of prematurity.
617
Q

How to manage Lupus Nephritis ?

A
  • Hydroxychloroquine
  • Glucocorticoids plus either cyclophosphamide intravenously or mycophenolate mofetil orally.
  • Angiotensin-converting enzyme inhibitors or angiotensin-receptor blockers should be administered if proteinuria reaches or exceeds 0.5 g/day.
  • Patients with class V lupus nephritis are generally treated with prednisone for 1-3 months, followed by tapering for 1-2 years if a response occurs.
  • Patients with end-stage renal disease require dialysis and are good candidates for kidney transplantation. Hemodialysis is preferred to peritoneal dialysis.
618
Q

Which vaccines are contraindicated in patients with SLE ?

A
  • Chickenpox and herpes zoster vaccines.
  • MMR vaccine
  • Oral polio vaccine.
  • Oral typhoid vaccine
  • Yellow fever vaccine.

**Live influenza vaccines in general

619
Q

How to manage surgical menopause ?

A
  • Estrogen only MHT (Menopausal Hormone Therapy) is generally prescribed for those women who have had a hysterectomy.
  • Women who retain their uterus should use an oestrogen and progestogen combination preparation.
  • For women who have had both hysterectomy and bilateral oophorectomy for endometriosis, use low dose oestrogen only preparations in younger women and to discontinue oestrogen if symptoms of endometriosis recur and consider using a non-hormonal agent to treat hot flushes.
620
Q

When do you see an Egg-on-a-string sign on CXR ?

A

In transposition of the great arteries (TGA).

621
Q

What is Milk-alkali syndrome ?

(From SMLE notes)

A
  • Excessive consumption of calcium carbonate.
  • Presents with hypercalcemia, metabolic alkalosis, and acute kidney injury.
  • PTH usually suppressed.
622
Q

How to diagnose post-streptococcal GN by total complement activity ?

(From SMLE notes)

A

Depressed C3 and CH50 in the first two weeks of the disease course.

623
Q

How is the level of :
- Ca
- P
- Vit D
- PTH
in Primary , secondary and tertiary hyperparathyroidism ?

A

Primary hyperparathyroidism:
- Ca: High
- P : low
- Vit D : high
- PTH: High or normal

Secondary hyperparathyroidism:
- Ca: Low or normal
- P : Low or high
- Vit D: Low
- PTH: high

Tertiary hyperparathyroidism :
- Ca: High
- P : High
- Vit D : low
- PTH: Very High

624
Q

What are the Jones criteria for diagnosis of acute rheumatic fever (ARF)?

A

The Jones criteria are used to diagnose rheumatic fever.

The 5 major criteria consist of the following:
1.Carditis
2.Polyarthritis
3.Chorea
4.Erythema marginatum
5.Subcutaneous nodules

The minor criteria include the following:
1. Fever
2. Arthralgia
3. Elevated erythrocyte sedimentation rate or C-reactive protein level
4. Prolonged PR interval on electrocardiogram (ECG).

The presence of 2 major manifestations or of 1 major and 2 minor manifestations, supported by evidence of a preceding GAS infection by positive throat swab or culture results or by high serum ASO titers, strongly suggests ARF.

625
Q

What are the Contraindications to Routine Circumcision ?

A
  • Hypospadias.
  • Chordee.
  • Penile Torsion.
  • Webbed Penis.
  • Buried Penis.
  • Urethral Hypoplasia.
  • Epispadias.
  • Ambiguous Genitalia (including bilateral cryptorchidism or micropenis).
  • Any current illness or medical condition that requires monitoring
  • Age less than 12 - 24 hours
  • Known bleeding diathasis (e.g. hemophilia or thrombocytopenia)
  • Disorders of the skin or connective tissue that would impair normal healing
626
Q

What is the definition and management of prolonged second stage of labor ?

A

Diagnosis : Failed delivery of the baby after 3 hours in a primipara and after 2 hours in a multipara (an extra hour may be added if an epidural was adminstered).

Mx:
- Oxytocin if uterine contractions are inadequate and progress is > 1 cm after 60 -90 minutes of pushing.

  • Trial of forceps or vacuum delivery if the fetal head is engaged AND maternal contractions are adequate.
  • CS if fetal head is not engaged.
627
Q

What is the cause of endometrial hyperplasia ?

A

Increased estrogen stimulation leads to excessive proliferation of the endometrium like in :
- PCOS
- Estrogen producing ovarian tumors.
- Follicle persistence in anvulatory cycles
- Hormone replacement therapy without progestrin administration (progestrin prevent endometrial hyperplasia).

628
Q

Secondary prevention for patients after a myocardial infarction recommendations ?

A

Lifestyle changes:
- exercise
- Stop smoking
- eat a Mediterranean-style diet.
- consume at least 7 g of omega 3 fatty acids (from two to four portions of oily fish) a week.
- To keep weekly alcohol consumption within safe limits (no more than 21 units a week for men, 14 units for women) and to avoid binge drinking (more than three drinks in 1-2 hours).
- maintain a healthy weight.

Advise patients against taking:
- Supplements containing β carotene (may increase risk of cardiovascular death)
- Vitamin E or C supplements (no evidence of benefit)
- Folic acid supplements (no evidence of benefit).

Medications:
- ACEI: (reduces mortality, the risk of myocardial infarction, and, in selected patients, the risk of developing heart failure).
- Aspirin (reduces cardiovascular mortality and morbidity).
- β blocker (reduces total mortality and cardiovascular morbidity)
- Statin.

** After a non-ST elevation myocardial infarction , treat patients with both clopidogrel and low dose aspirin for 12 month.

** After an ST elevation myocardial infarction , treat patients for at least four weeks if this combination has been started within the first 24 hours (reduces total mortality and the risk of myocardial infarction and stroke). Thereafter, continue standard treatment, including low dose aspirin without clopidogrel, unless there are other indications to continue both.

629
Q

What are the guidelines of aortic stenosis ?

A
  1. Asymptomatic patients with high-gradient severe aortic stenosis:
    No intervention is appropriate OR Intervention may be considered for those with:
    - Left ventricular ejection fraction (LVEF) of at least 50%.
    - Vmax 4.0-4.9 m/s.
    - Negative exercise stress.
    - No predictors of rapid progression.
    - high-risk profession or lifestyle and low surgical risk.
  2. Low-flow (LF) low-gradient (LG) severe aortic stenosis and reduced LVEF (< 50%):
    - intervention is appropriate OR
    - balloon aortic valvotomy .
  3. Symptomatic severe aortic stenosis and high or extreme surgical risk:
    - Transcatheter aortic valve replacement (TAVR).
    - surgical AVR (SAVR).
630
Q
  1. What is Myasthenia gravis (MG) ?
  2. Pathophysiology ?
  3. Signs & symptoms ?
  4. Diagnosis ?
  5. Management ?
A
  1. Myasthenia gravis is a relatively rare acquired, autoimmune disorder caused by an antibody-mediated blockade of neuromuscular transmission resulting in skeletal muscle weakness.
  2. The autoimmune attack occurs when autoantibodies form against the nicotinic acetylcholine postsynaptic receptors at the neuromuscular junction of skeletal muscles.
  3. Signs & symptoms:
    - clinical hallmark of MG is the presence of fluctuating fatigable muscle weakness that worsens with activity and improves on rest.
    - ocular symptoms with or without generalized weakness.
    - Limb weakness may be more severe proximally than distally.
  4. Diagnosis :
    - anti–acetylcholine receptor (AChR) antibody test.
  5. Management:
    - Symptomatic therapy
    - Anticholinesterase (AchE) inhibitors
    - Pyridostigmine is used for maintenance therapy.
    - Neostigmine is generally used only when pyridostigmine is unavailable
    - Rapidly acting or short-term immunomodulating agents:
    Intravenous immune globulin (IVIg)
    - Plasmapheresis
    - Long-term immunosuppression:
    • Prednisone
    • Azathioprine
    • Mycophenolate mofetil
    • Cyclosporine
    • Tacrolimus
    • Methotrexate
    • Rituximab
    • Cyclophosphamide is used in refractory/severe MG..
631
Q

What are the factors that may trigger or worsen exacerbations of Myasthenia gravis ?

A
  • Warm weather
  • Surgery
  • Immunization
  • Emotional stress
  • Menstruation
  • Intercurrent illness (eg, viral infection)
  • Tapering of immunosuppression
  • Pregnancy and postpartum period
  • Worsening of chronic medical illnesses (cardiac, renal, autoimmune, etc.)
  • Medication (eg, aminoglycosides, ciprofloxacin, telithromycin. clindamycin, phenothiazines, chlorpromazine, diazepam, halothane, ketamine, lidocaine, procain, non-depolarizing neuromuscular blocking agents, chloroquine, procaine, lithium, phenytoin, beta-blockers, procainamide, propafenone, bretylium, qunidine, calcium channel blockers, d-penicillamine, high-dose prednisone, magneisum)
632
Q

What is the most common cause of persistent hyperparathyroidism ?

A

Missed parathyroid adenoma.

633
Q

Which nerve is injured in foot drop ?

A

peroneal nerve

634
Q

What is The American Association for the Surgery of Trauma (AAST) splenic injury scale ?

A

Grade I:
- subcapsular hematoma <10% of surface area.
- parenchymal laceration <1 cm depth
- capsular tear.

Grade II:
- subcapsular hematoma 10-50% of surface area.
- intraparenchymal hematoma <5 cm
- parenchymal laceration 1-3 cm in depth.

Grade III:
- subcapsular hematoma >50% of surface area
- ruptured subcapsular or intraparenchymal hematoma ≥5 cm
- parenchymal laceration >3 cm in depth.

Grade IV:
- Any injury in the presence of a splenic vascular injury or active bleeding confined within splenic capsule.
- parenchymal laceration involving segmental or hilar vessels producing >25% devascularisation.

Grade V
- shattered spleen
- any injury in the presence of splenic vascular injury with active bleeding extending beyond the spleen into the peritoneum.

635
Q

How to manage spleen injuries ?

A
  • All patients with suspect splenic injury should be assessed, resuscitated, and treated according to ATLS principles.
  • Patients who are haemodynamically unstable or with a grade 5 injury (a shattered spleen or major hilar vascular injury) need urgent laparotomy.
  • Haemodynamically stable patients with grade 1–3 injuries without active extravasation can be treated conservatively.

They should be resuscitated using the principle of permissive hypotension, admitted to a high dependency area for observation, and have serial abdominal examinations for any evidence of deterioration. The patient should be placed on strict bed rest and have a repeat CT scan at 1-week post-injury.

Any evidence of increasing tenderness or peritonitis, there should be a low threshold for re-imaging and / or laparotomy (as associated injuries such as small bowel injuries are easily missed on initial CT imaging).

All patients who are treated conservatively should receive prophylactic vaccinations (against Strep Pneumoniae, Haemophilus Influenzae B (HIB) and Meningococcus).

If there is evidence of active extravasation of the contrast during the arterial phase of the CT scan, the patient should undergo embolisation (if locally available) or laparotomy with splenectomy.

636
Q

What is Sandifer syndrome ?

A

It is a condition that involves spasmodic torsional dystonia with arching of the back and rigid opisthotonic posturing, associated with symptomatic gastroesophageal reflux, esophagitis, or hiatal hernia.
in infants

637
Q

Which antibiotics to give in pregnants with Group B Streptococcus(GBS) infection?

(According to SMLE notes)

A

Penicillin G or Ampicillin.
In case of penicillin-allergy we use Cefazolin.

638
Q

What is the standard imaging study for the diagnosis of urethral injury ?

(According to SMLE notes)

A

Retrograde urethrography.

639
Q

Glasgow Coma Scale

A

Eye Opening Response:
• Spontaneous–open with blinking at baseline: 4 points
• To verbal stimuli, command, speech: 3 points
• To pain only (not applied to face): 2 points
• No response: 1 point

Verbal Response
• Oriented: 5 points
• Confused conversation, but able to answer: questions 4 points
• Inappropriate words: 3 points
• Incomprehensible speech: 2 points
• No response: 1 point

Motor Response
• Obeys commands for movement :6 points
• Purposeful movement to painful stimulus: 5 points
• Withdraws in response to pain: 4 points
• Flexion in response to pain (decorticate posturing): 3 points
• Extension response in response to pain (decerebrate posturing): 2 points
• No response: 1 point

640
Q
  1. What is Henoch-Schönlein purpura (HSP) ?
  2. What is the hallmark of the disease ?
  3. Signs and symptoms?
A

1.It is an acute immunoglobulin A (IgA)–mediated disorder characterized by a generalized vasculitis involving the small vessels of the skin, the gastrointestinal (GI) tract, the kidneys, the joints, and, rarely, the lungs and the central nervous system (CNS).

2.The hallmark of the disease is Rash involving the legs.

  1. Signs and symptoms:
    - Headache
    - Anorexia
    - Fever
    - Rash, involving the legs
    - Abdominal pain and vomiting (35-85%)
    - Joint pain (60-84%), especially involving the knees and ankles
    - Subcutaneous edema (20-50%)
    - Scrotal edema (2-35%)
    - Bloody stools.
641
Q

What is the most common vasculitis in childhood ?

A

Henoch-Schönlein purpura (HSP).

642
Q

How to diagnose Henoch-Schönlein purpura (HSP) ?
+ How to manage Henoch-Schönlein purpura (HSP) ?

A

Diagnosis:
Presence of palpable purpura or petechiae—with lower limb predominance and without thrombocytopenia or coagulopathy+ at least 1 of the following:

  • Diffuse abdominal pain of acute onset.
  • Any biopsy sample showing leukocytoclastic vasculitis, or proliferative glomerulonephritis with predominant IgA deposition in a kidney biopsy sample in patients with atypical distribution of purpura.
  • Acute arthritis or acute arthralgia in any joint.
  • Kidney involvement (any hematuria or proteinuria) and IgA deposition.

Management:
- Treatment remains primarily supportive in most cases, though pharmacotherapy, plasmapheresis, and surgical interventions may also be considered in select cases.

  • Supportive measures may include the following:
    • Ensuring adequate hydration.
    • Monitoring for GI and kidney complications.
    • Treating minor symptoms of arthritis, edema, fever, or malaise
    • Eating a bland diet
    • Discontinuing any drugs suspected of playing a causative role
  • Analgesics for joint and soft tissue discomfort.
  • Steroids
  • Plasmapheresis
  • Surgery
643
Q

What are the risk factors of placenta previa ?

A
  • Advancing maternal age (>35 y)
  • Infertility treatment
  • Multiparity.
  • Multiple gestation
  • Short interpregnancy interval
  • Previous uterine surgery, uterine insult or injury
  • Previous cesarean delivery.
  • Previous or recurrent abortions
  • Previous placenta previa (4-8%)
  • Nonwhite ethnicity
  • Low socioeconomic status
  • Smoking
  • Cocaine use
644
Q

What are the complications of polyhydramnios ?

A
  • Maternal dyspnea/respiratory distress
  • Preterm labor
  • Premature rupture of membranes
  • Abnormal fetal presentation
  • Umbilical cord prolapse
  • Postpartum hemorrhage
  • Fetal macrosomia due to maternal diabetes mellitus
  • Hypertensive disorders of pregnancy
  • Urinary tract infections
  • Amniotic fluid embolism
  • Increased maternal uterine tone
    -Transient fetal respiratory distress.
  • Placental abruption
  • Fetal-maternal hemorrhage
  • Maternal Rh sensitization
  • Fetal pneumothorax.
645
Q

What is the first-line treatment for dysmenorrhea caused by endometriosis ?

A

Combined oral contraceptives

646
Q

How to calculate a death rate ?

A

The number of deaths recorded is divided by the number of people in the population, and then multiplied by 100, 1,000 or another convenient figure

647
Q

How to treat acute poststreptococcal glomerulonephritis (APSGN) ?

A
  • The major goal is to control edema and blood pressure.
  • Restrict salt and water.
  • Diuretics (Loop diuretics): If significant edema or hypertension develops.

(Loop diuretics (furosemide) are preferred over thiazides (hydrochlorothiazide or chlorthalidone)

  • Antihypertensives may be needed in the chronic phase if the blood pressure remains high:
    • For HTN not controlled by diuretics, the usual 2nd-line choices are CCB, ACEIs, or ARBs.
    • Although ACEIs and ARBs carry the risk of hyperkalemia and temporarily impairing recovery of renal function.
      *For malignant hypertension, intravenous nitroprusside or other parenteral agents are used.
648
Q

How and when to screen for DM in pregnancy ? and how to manage ?

A

one-step 75 g oral glucose tolerance test for all women not already known to be diabetic at 24–28 weeks of gestation.

Diabetes is diagnosed where one or more threshold value is exceeded:
- fasting ≥5.1 mmol/l
- 1-hour ≥10.0 mmol/l
- 2-hour ≥8.5 mmol/l

Management:
- Lifestyle advice including dietary modification is the primary intervention in all women diagnosed with gestational diabetes. - If failed: oral hypoglycaemic agents or insulin (Both glibenclamide and metformin are effective treatments for gestational diabetes).

649
Q

How to diagnose COPD ?

A

Spirometry is required to make the diagnosis; a postbronchodilator FEV 1/FVC < 0.70 confirms the presence of persistent airflow limitation.

650
Q

How to manage COPD ?

A
  • Bronchodilators:
    They work by dilating airways, thereby decreasing airflow resistance. This increases airflow and decreases dynamic hyperinflation. These drugs provide symptomatic relief but do not alter disease progression or decrease mortality.
  • Beta2 agonists and cholinergic/muscarinic antagonists:
  • Tiotropium
  • Aclidinium
  • Umeclidinium bromide
  • Revefenacin
  • Phosphodiesterase inhibitors
  • Smoking cessation is key.
  • Pneumococcal and influenza vaccinations decrease the incidence of lower respiratory tract infections.
651
Q

How to manage COPD exacerbation ?

A
  • Short-acting beta2-agonists:
    The recommended initial bronchodilators to treat an exacerbation are short-acting beta2-agonists, with or without short-acting anticholinergics.
  • Systemic corticosteroids:
    Systemic corticosteroids can improve lung function and oxygenation. They also shorten recovery time and hospital duration. The duration of systemic corticosteroid therapy should not exceed 5-7 days.
  • Noninvasive mechanical ventilation:
    The first mode of ventilation used in COPD with acute respiratory failure and without contraindications is noninvasive mechanical ventilation. It improves gas exchange, reduces the work of breathing, decreases the need for intubation, decreases hospitalization duration, and improves survival.
  • Long-acting bronchodilator:
    As soon as possible before hospital discharge, initiate maintenance therapy with a long-acting bronchodilator.
652
Q

What is the definition of COPD exacerbation ?

A

A COPD exacerbation is defined as acute respiratory symptom worsening with the need for additional therapy. Several factors can lead to an exacerbation, the most common being respiratory tract infections.

653
Q

What are the indications for intensive care admission in patients with COPD ?

A
  • Confusion
  • Lethargy
  • Respiratory muscle fatigue.
  • Worsening hypoxemia
  • Respiratory acidosis (pH < 7.30)
  • Clinical concern for impending or active respiratory failure.
654
Q

What is the definition of prolonged latent stage of labor ?

A

The first stage of labor is divided into latent and active phases. According to Friedman et al., latent stage considered to be prolonged if takes:
>20 hours for nulliparous women, and
>14 hours for multiparous women.

655
Q

Transient ischemic attack (TIA) investigations ?

A
  • Rule out metabolic or drug-induced causes.
  • Fingerstick blood glucose test.
  • CBC, coagulation profile and serum electrolytes levels.
  • ECG) with rhythm strip.
  • Brain imaging is recommended within 24 hours of symptom onset:
    • MRI with diffusion-weighted imaging (DWI) is preferred.
    • Noncontrast CT of the head is a reasonable first choice when MRI is not readily available.
  • The cerebral vasculature should be imaged on an urgent basis, preferably at the same time as the brain.
656
Q

What is (AbcD2 score) ?
and how to investigate according to the score ?

A

It is a tool used to assist with predicting the risk of stroke in TIAs at 48 hours.

Age:
- 60 years or older (1 point)
Blood pressure:
- Systolic ≥140 mmHg (1 point)
- Diastolic ≥90 mmHg (1 point)
Clinical features:
- Any unilateral weakness (2 points)
- Speech impairment without weakness (1 point)
Duration:
- 60 minutes or more (2 points)
- 10–59 minutes (1 point)
Diabetes mellitus (1 point).

ABCD2 >4 = high risk, ≤4 = low risk (maximum is 7).

Management:
ABCD2 score 4 or less:
- CT brain scan to exclude haemorrhage and other space occupying lesions (eg. tumour).
- Carotid duplex ultrasound.
as soon as possible (within 48–72 hours).

657
Q

When to consider hospitalization for patients with TIA ?

A

It is reasonable to hospitalise patients with TIA if they present within 72 hours of the event and any of the following criteria are present:

• ABCD2 score of equal to or greater than 3
• ABCD2 score of 0–2 and uncertainty that diagnostic workup can be completed within 2 days as an outpatient.
• ABCD2 score of 0–2 and other evidence that indicates the patient’s event was caused by focal ischemia.12

658
Q

TIA management ?

A
  1. Lifestyle modifications
    • Quit smoking.
    • Adopt a low fat, low sodium diet.
    • Exercise regularly ,and
    • Avoid excessive alcohol consumption.
  2. Antiplatelets
    - Combination of aspirin and dipyridamole shown to be more effective than aspirin alone.
    - Low dose aspirin and modified release dipyridamole or clopidogrel alone should be started.
  3. Anticoagulation:
    After a CT has excluded haemorrhage, warfarin should be started in all TIA patients who have :
    - Atrial fibrillation
    -Cardio-embolic stroke from valvular heart disease, or
    - Recent myocardial infarction.
    - Even in patients with non-rheumatic atrial fibrillation.
  4. Cholesterol lowering agent:
    By diet and statins.
  5. Blood pressure lowering:
    - Angiotensin converting enzyme inhibitors (ACEI) and diuretics separately and together, have the most evidence.
659
Q

How much should B-hcg decrease after methotrexate in ectopic pregnancy ?

A

If the serum hCG did not decline by at least 15 percent between days four and seven, a second dose of methotrexate was given. Continued failure of the serum hCG to decline by at least 15 percent during any successive week mandated an additional dose of methotrexate. If after three doses the hCG level did not decline or cardiac activity was present, the ectopic pregnancy was treated surgically.

660
Q

What are the risk factors of abortion ?

A
  • Age : Women older than age 35 have a higher risk of miscarriage.
  • Previous miscarriages.
  • Chronic conditions.
  • Uterine or cervical problems.
  • Smoking, alcohol and illicit drugs.
  • Weight. Being underweight or overweight.
  • Invasive prenatal tests.
661
Q

What is Alport syndrome ?

A

It is a genetic condition characterized by kidney disease, hearing loss, and eye abnormalities.

Can’t Pee, Can’t see, can’t hear a bee !

662
Q

How to manage functional or simple ovarian cysts (thin-walled cysts without internal structures) which are less than 50 mm (5 cm) maximum diameter ?

A

Usually resolve over 2–3 menstrual cycles without the need for intervention.

Women with small (less than 50 mm diameter) simple ovarian cysts generally do not require follow-up as these cysts are very likely to be physiological and almost always resolve within 3 menstrual cycles.

663
Q

Which blood tests should be measured in all women under age 40 with a complex ovarian mass ? and why ?

A

Lactate dehydrogenase (LDH), α-FP and hCG should be measured in all women under age 40 with a complex ovarian mass because of the possibility of germ cell tumours.

664
Q

What is the role of ultrasound in the assessment of suspected ovarian masses?

A

A pelvic ultrasound is the single most effective way of evaluating an ovarian mass with transvaginal ultrasonography being preferable due to its increased sensitivity over transabdominal ultrasound.

665
Q

How to manage simple ovarian cysts of 50–70 mm in diameter ?

A

yearly ultrasound follow-up.

666
Q

How to manage simple ovarian cysts with a larger diameter >70 mm ?

A

considered for either further imaging (MRI) or surgical intervention.

667
Q

Is the laparoscopic approach better for the elective surgical management of ovarian masses?

A

The laparoscopic approach for elective surgical management of ovarian masses presumed to be benign is associated with lower postoperative morbidity and shorter recovery time and is preferred to laparotomy in suitable patients.

668
Q

Should an ovarian cyst be aspirated?

A

Aspiration of ovarian cysts, either vaginally or laparoscopically, is less effective and is associated with a high rate of recurrence.

669
Q

What is the :
- Appearance
- WBC
- RBC
- Protein
- Glucose
- Opening pressure
in a normal CSF ?

A
  • Appearance: clear and colourless
  • White blood cells (WBC):
    0 – 5 cells/µL
    no neutrophils present, primarily lymphocytes
    normal cell counts do not rule out meningitis or any other pathology.
  • Red blood cells (RBC): 0 – 10/mm³
  • Protein: 0.15 – 0.45 g/L (or <1% of the serum protein concentration)
  • Glucose: 2.8 – 4.2 mmol/L (or ≥ 60% plasma glucose concentration)
  • Opening pressure: 10 – 20 cm H2O
670
Q

What is the :
- Appearance
- WBC
- Protein
- Glucose
- Opening pressure
in a Bacterial meningitis ?

A
  • Appearance: cloudy and turbid
  • Opening pressure: elevated (>25 cm H₂O)
  • WBC: elevated >100 cell/µL (primarily polymorphonuclear leukocytes (>90%))
  • Glucose level: low (<40% of serum glucose)
  • Protein level: elevated (>50 mg/dL)
671
Q

What is the :
- Appearance
- WBC
- Protein
- Glucose
- Opening pressure

in a Viral meningitis ?

A
  • Appearance: clear
  • Opening pressure: normal or elevated
  • WBC: elevated (50 – 1000 cells/µL, primarily lymphocytes, can be PMN (polymorphonuclear) early on)
  • Glucose level: normal (>60% serum glucose, however, may be low in HSV infection)
  • Protein level: elevated (>50 mg/dL)
672
Q

What is the :
- Appearance
- WBC
- Protein
- Glucose
- Opening pressure
in a Fungal meningitis ?

A
  • Appearance: clear or cloudy
  • Opening pressure: elevated
  • WBC: elevated (10 – 500 cells/µL)
  • Glucose level: low
  • Protein level: elevated
673
Q

What is the :
- Appearance
- WBC
- Protein
- Glucose
- Opening pressure

in a Tuberculosis meningitis ?

A
  • Appearance: opaque, if left to settle it forms a fibrin web.
  • Opening pressure: elevated
  • WBC: elevated (10 – 1000 cells/µL, early PMNs then mononuclear)
  • Glucose level: low
  • Protein level: elevated (1-5 g/L)
674
Q

What is the :
- Appearance
- WBC
- Protein
- Glucose
- Opening pressure
in Guillain Barre syndrome

A
  • Appearance: clear or xanthochromia
  • Opening pressure: normal or elevated
  • WBC: normal
  • Glucose level: normal
  • Protein level: elevated (>5.5 g/L)
675
Q

What are the risk factors for abruptio placentae?

A
  • Maternal hypertension : Most common cause.
  • Maternal trauma.
  • Cigarette smoking
  • Alcohol consumption
  • Cocaine use
  • Short umbilical cord
  • Sudden decompression of the uterus (eg, premature rupture of membranes, delivery of first twin).
  • Retroplacental fibromyoma
  • Retroplacental bleeding from needle puncture.
  • Idiopathic.
  • Previous placental abruption
  • Chorioamnionitis.
  • Prolonged rupture of membranes (24 h or longer)
  • Maternal age 35 years or older
  • Maternal age younger than 20 years
  • Male fetal sex
  • Low socioeconomic status
  • Elevated second trimester maternal serum alpha-fetoprotein.
  • Subchorionic hematoma.
676
Q

The Tanner Stages in females ?

A

Stage I (Preadolescent):
- There is no sexual hair.
- Breast: Only the papilla is elevated above the level of the chest wall.

Stage II :
- Sparse, long, pigmented, downy hair, which is straight or only slightly curled, appears.
- Mainly along the labia.
- Breast: (Breast Budding) - Elevation of the breasts and papillae may occur as small mounds along with some increased diameter of the areolae.

Stage III:
- Considerably darker, coarser, and curlier sexual hair appears.
The hair has now spread sparsely over the junction of the pubes.
- Breast: The breasts and areolae continue to enlarge, although they show no separation of contour.

Stage IV:
- The hair distribution is adult in type but decreased in total quantity.
- There is no spread to the medial surface of the thighs.
- Breast: The areolae and papillae elevate above the level of the breasts and form secondary mounds with further development of the overall breast tissue.

Stage V:
- Hair appears to have an inverse triangle of the classically feminine type.
- There is spread to the medial surface of the thighs but not above the base of the inverse triangle.
- Breast: Mature female breasts have developed. The papillae may extend slightly above the contour of the breasts as the result of the recession of the aerolae.

677
Q

The Tanner Stages in males ?

A

Stage I (Preadolescent):
- There is no androgen-sensitive pubic hair.
- The testes, scrotal sac, and penis have a size and proportion similar to those seen in early childhood.

Stage II:
- There is sparse development of long pigmented downy hair, which is only slightly curled or straight.
- The hair is seen chiefly at the base of penis.
- There is enlargement of the scrotum and testes and a change in the texture of the scrotal skin. The scrotal skin may also be reddened.

Stage III :
- The pubic hair is considerably darker, coarser, and curlier.
- The distribution is now spread over the junction of the pubes.
- Further growth of the penis has occurred, initially
in length, although with some increase in circumference. There also is increased growth of the testes and scrotum.

Stage IV :
- The hair distribution is now adult in type but still there is no spread to the medial surface of the thighs.
- The penis is significantly enlarged in length and circumference, with further development of the glans penis. The testes and scrotum continue to enlarge, and there is distinct darkening of the scrotal skin.

Stage V :
- Hair distribution is adult in quantity and type and is described in the inverse triangle. There can be spread to the medial surface of the thighs.
- The genitalia are adult with regard to size and shape.

678
Q

How to treat breast Lobular carcinoma in situ ?

A

Observation and close follow-up care with or without tamoxifen and bilateral mastectomy with or without reconstruction.
There is no evidence of therapeutic benefit from local excision, axillary dissection, radiotherapy, or chemotherapy.

679
Q

Diarrhea comes with which acid base disturbance ?

A

Metabolic acidosis with hypokalemia.

Gastrointestinal losses, from diarrhea, vomiting, or nasogastric suctioning, also are common causes of hypokalemia.

680
Q

How does the diagnosis of spontaneous rupture of the membranes is made ?

A

by maternal history followed by a sterile speculum examination.
If, on speculum examination, no amniotic fluid is observed, clinicians should consider performing an insulin‐like growth factor‐binding protein 1 (IGFBP‐1) or placental alpha microglobulin‐1 (PAMG‐1) test of vaginal fluid to guide further management.

**The role of ultrasound assessment of amniotic fluid volume is unclear.

681
Q

How to manage preterm prelabour rupture of the membranes ?

A
  • An antibiotic (preferably erythromycin) should be given for 10 days or until the woman is in established labour (whichever is sooner).
  • Women who have PPROM between 24+0 and 33+6 weeks’ gestation should be offered corticosteroids; steroids can be considered up to 35+6 weeks’ gestation.
  • Women whose pregnancy is complicated by PPROM after 24+0 weeks’ gestation and who have no contraindications to continuing the pregnancy should be offered expectant management until 37+0 weeks.
  • In women who have PPROM and are in established labour or having a planned preterm birth within 24 hours, intravenous magnesium sulfate should be offered between 24+0 and 29+6 weeks of gestation.
682
Q

Should tocolytic agents be used preterm prelabour rupture of the membranes ?

A

Tocolysis in patients with PPROM is not recommended.

683
Q

Tumor screening recommendations in Beckwith-Wiedemann Syndrome ?

A

Ultrasounds
every 3 months until age 8 years.

Alpha-fetoprotein (AFP) measurements:
At intervals ranging from 6 weeks to 3 months until age 4 years.

684
Q

What are the risk factors for endometrial carcinoma?

A
  • Age. Uterine cancer most often occurs in women over 50.
  • Obesity
  • Nulliparity
  • An individual who has had a late menopause (aged >52y).
  • Things that affect hormone levels, like:
    • taking estrogen after menopause
    • birth control pills
    • tamoxifen
    • the number of menstrual cycles (over a lifetime)
    • pregnanc
    • certain ovarian tumors
    • polycystic ovarian syndrome (PCOS).
  • DM
  • Diet: Women who eat high in animal fat may have an increased risk of uterine cancer.
685
Q

What are the complications associated with DM in pregnancy ?

A
  • Preeclampsia
  • Miscarriage
  • Fetal Growth restriction
  • Macrosomia
686
Q

How to diagnose fetal anemia ?

A
  • Prenatal ultrasound: signs of fetal heart failure or unusual blood flow in a vessel.
  • Maternal blood testing can detect specific antibodies that may cause anemia in the baby.
  • Amniocentesis:
    test the amniotic fluid to determine how red blood cells are breaking down in the fetal circulatory system.
  • Fetal blood sampling tests blood from the umbilical vein to look for anemia. This test is similar to amniocentesis, but the needle is guided by ultrasound into the umbilical vein.
687
Q

What is Toxic synovitis/Transient Synovitis ?
Important Hx ?
Dx ?
TX?

A

It is the most common cause of acute hip pain in children aged 3-10 years. The disease causes arthralgia and arthritis secondary to a transient inflammation of the synovium of the hip.

Important history:
- Recent history of an upper respiratory tract infection, pharyngitis, bronchitis, or otitis media.
- usually afebrile or low garde fever.

Diagnosis :
- WBC: may be slightly elevated.
- Erythrocyte sedimentation rate: may be slightly elevated.
- C-reactive protein (CRP) level rises within 6 hours after the onset of septic arthritis of the hip and peaks at 2 days.

  • Biopsy: nonspecific inflammation and hypertrophy of the synovial membrane.
  • US: effusion that causes bulging of the anterior joint capsule. Synovial fluid has increased proteoglycans.

Treatment:
- Apply heat and massage.
- If the diagnosis of transient synovitis is uncertain or the patient is uncomfortable, hospitalize for observation and traction.
- Home treatment also can include traction.
- Skin traction of the hip in 45° of flexion minimizes intracapsular pressure.
- Treatment with ibuprofen may shorten the duration of symptoms
- Advise bedrest for 7-10 days.
- Advice not to bear weight on the affected limb.
- Advise to avoid full unrestricted activity until the limp and pain have resolved.

688
Q

What are the Complications of blood transfusion ?

A

Early:
1- Haemolytic reactions
2- Non-haemolytic febrile reactions
3- Allergic reactions to proteins, IgA
4- Transfusion-related acute lung injury
5- Reactions secondary to bacterial contamination
6- Circulatory overload
7- Air embolism
8- Thrombophlebitis
9- Hyperkalaemia
10- Citrate toxicity
11- Hypothermia
12- Clotting abnormalities (after massive transfusion).

Late :
1. Transmission of infection
        * Viral (hepatitis A, B, C, HIV, CMV)
        * Bacterial (Treponeum pallidum, Salmonella)
        * Parasites (malaria, toxoplasma)
2. Graft-vs-host disease
3.Iron overload (after chronic transfusions)
4. Immune sensitization (Rhesus D antigen)

Hypocalcemia in massive blood transfusion.

689
Q

Which diagnostic tests are used to diagnose Wilson Disease ?

A
  • complete blood count
  • serum ceruloplasmin
  • copper levels
  • ocular slit-lamp examination
  • A 24-hour urinary copper excretion.
690
Q

What is Milk-alkali syndrome ?

A

Milk-alkali syndrome is caused by the ingestion of large amounts of calcium and absorbable alkali, with resulting hypercalcemia. If unrecognized and untreated, milk-alkali syndrome can lead to metastatic calcification and renal failure.

691
Q

When to use Leukotriene inhibitors in asthma patients ?

A

Leukotriene inhibitors are alternative treatments in exercise-induced asthma and can be of benefit for children when oral therapy is preferred over inhalers. Leukotriene inhibitors are effective in the treatment of allergic rhinitis but are less effective than intranasal corticosteroids.

692
Q

When does thoracotomy is indication after chest tube insertion ?

A

Thoracotomy is indicated when total chest tube output exceeds 1500 mL within 24 hours, regardless of injury mechanism.

THE INDICATIONS for thoracotomy after traumatic injury typically include shock, arrest at presentation, diagnosis of specific injuries (such as blunt aortic injury), or ongoing thoracic hemorrhage.

693
Q

What is the benefit of acyclovir in herpes outbreak during pregnancy ?

A

Acyclovir (Zovirax) and valacyclovir (Valtrex) are the drugs most commonly used for herpes outbreak suppression and treatment during pregnancy. These drugs help reduce how active the virus is. They also help speed up lesion healing.

(to lessen the severity of herpes zoster virus infection.).

694
Q

What is Walled-off pancreatic necrosis ?
How to treat it ?

A

Walled-off pancreatic necrosis (WOPN) is a late complication of acute pancreatitis, although it can occur in chronic pancreatitis or as a result of pancreatic trauma.

WOPN usually occurs four weeks after the episode of acute pancreatitis. Before this time, it is referred to as an acute necrotic collection (ANC).

Tx :
- Percutaneous catheter drainage.
- Endoscopic retrograde cholangiopancreatography (ERCP) drainage (+/- necrosectomy) .

695
Q

What is the definition of the following terms:
- Acute peripancreatic fluid collections (APFC)
- Pseudocysts
- Acute necrotic collections (ANCs)
- Walled-off necrosis (WON or WOPN):

A

1.Fluid collections associated with interstitial edematous pancreatitis (i.e. minimal or no necrosis):
- Acute peripancreatic fluid collections (APFC):
in the first 4 weeks: non-encapsulated peripancreatic fluid collections.
- Pseudocysts:
develop after 4 weeks; encapsulated peripancreatic or remote fluid collections.

  1. Fluid collections associated with necrotizing pancreatitis:
    - Acute necrotic collections (ANCs):
    in the first 4 weeks; non-encapsulated heterogeneous non-liquefied material.
    - Walled-off necrosis (WON or WOPN):
    develop after 4 weeks; encapsulated heterogeneous non-liquefied material.
696
Q

What are the causes of Sustained fetal hypoxia and subsequent bradycardia ?

A

Sustained fetal hypoxia and subsequent bradycardia include:
- Uterine hyperstimulation (by tachysystole or hypertonus).
- Maternal hypotension (positional, procedural or anaesthetic)
- Sustained umbilical cord compression, including cord prolapse, or a rapid descent of the fetal head through the pelvis.

697
Q

What is the best method of evacuating a molar pregnancy?

A
  • Suction curettage is the method of choice of evacuation for complete molar pregnancies.
  • Suction curettage is the method of choice of evacuation for partial molar pregnancies except when the size of the fetal parts deters the use of suction curettage and then medical evacuation can be used.
  • Anti-D prophylaxis is required following evacuation of a molar pregnancy.
698
Q

What is G6PD Deficiency ?

A

Glucose-6-phosphate dehydrogenase deficiency, the most common enzyme deficiency worldwide.
It is an X-linked inherited disorder.

699
Q

How to diagnose Glucose-6-phosphate dehydrogenase deficiency ?

A

Rapid fluorescent spot test

700
Q

What are the complications of Glucose-6-phosphate dehydrogenase deficiency ?

A
  • Neonatal hyperbilirubinemia.
  • Acute hemolysis.
  • Chronic hemolysis.
701
Q

Name the Medications That Should Be Avoided By Persons with G6PD Deficiency.

A
  1. Dapsone: Antimicrobial for treatment of leprosy.
  2. Flutamide: Antiandrogen for treatment of prostate cancer.
  3. Mafenide cream (Sulfamylon): Topical antimicrobial.
  4. Methylene blue (Urolene Blue): Antidote for drug induced methemoglobinemia.
  5. Nalidixic acid (NegGram): Antibiotic used primarily for urinary tract infections.

6.Nitrofurantoin (Macrodantin): Antibiotic used primarily for urinary tract infections.

  1. Phenazopyridine (Pyridium): Analgesic for treatment of dysuria.
  2. Primaquine: Antimalaria agent
  3. Rasburicase (Elitek): Adjunct to antineoplastic agents
  4. Sulfacetamide (Klaron): Antibiotic (ophthalmic and topical preparations).
  5. Sulfamethoxazole (Gantanol): Antibiotic used in combination preparations (i.e., trimethoprim-sulfamethoxazole [TMP-SMX; Bactrim, Septra])

12.Sulfanilamide (AVC): Antifungal agent for treatment of vulvovaginalCandida albicans infection

702
Q

How to treat G6PD Deficiency ?

A
  • Avoidance of oxidative stressors.
  • Blood transfusion if needed.
  • Splenectomy generally is NOT recommended
703
Q

How to treat endometriosis ?

A

If fertility not desired:
- Analgesics (nonsteroidal anti-inflammatory drugs [NSAIDs] or acetaminophen) PLUS Oral contraceptive pills (OCPs).

No improvement:
- Gonadotropin-releasing hormone analogues (GnRHas; e.g., leuprolide, goserelin , triptorelin, nafarelin.

No improvemnt:
laprascopy and surgical treatment (exision on lesions).

If still no improvement:
hysterectomy and oophorectomy

Other medications can be used :
- Androgenic agents (e.g., danazol [Danocrine]).
- Progestogens (e.g., medroxyprogesterone acetate [Provera]).
- Antiprogestogens (e.g., gestrinone).

704
Q

What are the side effects of tocolytics ?

A
  • Tachycardia.
  • Hypotension.
  • palpitations.
  • shortness of breath
  • chest pain
  • pulmonary edema
  • hypokalemia
  • hyperglycemia
705
Q

What are the contents of Foramen ovale ?

A

OVALE

O: otic ganglion (inferior).
V: V3 cranial nerve (mandibular division of the trigeminal nerve).
A: accessory meningeal artery.
L: lesser petrosal nerve.
E: emissary veins.

706
Q

What is the clinical presentation of Organophosphate Toxicity ?

A

Muscarinic effects:
- SLUDGE (salivation, lacrimation, urination, diarrhea, GI upset, emesis)
- DUMBELS (diaphoresis and diarrhea; urination; miosis; bradycardia, bronchospasm, bronchorrhea; emesis; excess lacrimation; and salivation).

Nicotinic effects:
- muscle fasciculations
- cramping
- weakness
- diaphragmatic failure.
- Autonomic nicotinic effects include hypertension, tachycardia, mydriasis, and pallor.

CNS effects:
- Anxiety
- Emotional lability
- Restlessness
- Confusion
- Ataxia
- Tremors
- Seizures
- Coma
- Apnea

707
Q

Organophosphate Toxicity treatment ?

A

Atropine.

708
Q

What should we give after endoscopic treatment of high-risk peptic ulcers ?

A

High-dose intravenous proton pump inhibitors for 72 hours after endoscopic therapy for a bleeding ulcer reduce recurrent bleeding risk and surgery.

709
Q

BUN in pregnancy ?

A

A low BUN level can occur normally in the second or third trimester of pregnancy.

710
Q

What are the medications that are commonly used for left ventricular dysfunction ?

A
  1. Diuretics or water pills: Treats swelling of feet and abdomen.
  2. Beta-blockers: Slows heart rate and regulates blood pressure
  3. ACE inhibitors, ARB, ARNI: Widen blood vessels

4.Mineralocorticoid receptor antagonist (spironolactone and eplerenone): Rids the body of excess salt and fluid but retains potassium

  1. Nitrates and Hydralazine: Relaxes and further widens blood vessels

6.Digoxin: Assists in the pumping process of the heart

711
Q

What is the next step after finding Lymphatic Malformation (Cystic Hygroma) ?

A

antenatal chromosomal analysis (Chorionic villus sampling).

712
Q

What are the indications for long term oxygen therapy in COPD patients ?

A
  • A resting PaO2 = 7.3 kPa (55 mm Hg) or SaO2 88% or less while being at rest in a stable clinical condition
  • A resting PaO2 = 8.0 kPa (59 mm Hg) or SaO2 89% or less if there is evidence of cor pulmonale, right heart failure or polycythemia (hematocrit greater than 55%) while being in a stable clinical condition
713
Q

Which medications in the drug class Antihypertensive agents are used in the treatment of Systemic Sclerosis?

A
  • Reserpine:
    Reserpine depletes norepinephrine and epinephrine. This effect, in turn, depresses sympathetic nerve functions, decreasing the heart rate and lowering the arterial blood pressure.
  • Methyldopa (Aldomet):
    stimulates central alpha-adrenergic receptors, resulting in decreased sympathetic outflow. Treatment results in inhibition of vasoconstriction.
714
Q

Name the medications that are used to quit smoking and thier side effects.

A

Varenicline (Chantix):
- Nausea/Vomiting
- Headache
- Trouble sleeping, unusual dreams, or sleepwalking
- Constipation
- Gas
- Changes in taste
- Skin rashes
- Seizures
- Heart or blood vessel problems (mostly in people who already have these problems)
- Mood or behavior changes, such as depression, hallucinations, delusions, aggression, hostility, agitation, anxiety, panic, or even suicidal thoughts.

Bupropion (Zyban):
- This drug should not be taken if you have or have ever had:
* Seizures (it can cause or worsen seizures)
* Heavy alcohol use
* Cirrhosis
* A serious head injury
* Bipolar (manic-depressive) illness
* Anorexia or bulimia (eating disorders)
- Side effects:
* Dry mouth
* Stuffy nose
* Trouble sleeping and nightmares
* Tiredness
* Constipation
* Nausea
* Headaches
* High blood pressure
* Seizures
* Feeling depressed, anxious, agitated, hostile, aggressive, overly excited or hyperactive, or confused; or having suicidal thoughts.

Nortriptyline:
- fast heart rate
- blurred vision
- trouble urinating
- dry mouth
- constipation
- weight gain or loss
- low blood pressure when they stand up.

Clonidine:
- It’s FDA-approved to treat high blood pressure
- Side effects: constipation, dizziness, drowsiness, dry mouth, and unusual tiredness or weakness.

715
Q

What are the first-line treatments for smoking cessation?

A

Nicotine replacement and sustained release bupropion

716
Q

Which treatments for smoking cessation is safe in CVD ?

A

Nicotine patches have been studied extensively in patients with stable CVD and have been shown to be safe.

717
Q

What is the Differences between Kwashiorkor and Marasmus ?

A
  • Causes:
    Kwashiorkor : Deficiency of proteins.
    Marasmus: Deficiency of both proteins and calories.
  • Age factors:
    Kwashiorkor: Between the age of 6 months and 3 years of age.
    Marasmus: Between the age of 6 months and 1 year of age
  • Oedema
    Kwashiorkor: Present
    Marasmus: Absent
  • Subcutaneous fat:
    Kwashiorkor: Present.
    Marasmus: Absent
  • Weight loss:
    Kwashiorkor: There is some weight loss.
    Marasmus: There is severe weight loss.
  • Symptoms
    Kwashiorkor: The thinning of muscles and limbs.
    Marasmus: The thinning of limbs.
  • Fatty liver cells
    Kwashiorkor: There is an enlargement in the fatty liver cells.
    Marasmus: There is no enlargement in the fatty liver cells.
  • Appetite:
    Kwashiorkor: Voracious feeder.
    Marasmus: Poor appetite.
  • The texture of the skin:
    Kwashiorkor: Flaky paint appearance on the skin
    Marasmus: Dry and wrinkled skin.
  • Requirement of Nutrition:
    Kwashiorkor: Adequate amounts of proteins.
    Marasmus: Adequate amounts of proteins, carbohydrates and fats.
718
Q

Polyhydramnios is associated with which genetic disorders ?

A

Down syndrome (Trisomy 21) and Edward’s syndrome (Trisomy 18), but only when the baby also has a duodenal atresia or other blockage in the gastrointestinal tract.

719
Q

When should a women stop having cervical cancer screening?

A

You should stop having cervical cancer screening after age 65 years if:

  • You do not have a history of moderate or severe abnormal cervical cells or cervical cancer, and
  • You have had either three negative Pap test results in a row or two negative co-test results in a row within the past 10 years, with the most recent test performed within the past 5 years.
720
Q

streptokinase antidote ?

A
  • Aminocaproic acid.
  • Aprotinin
  • Tranexamic acid
721
Q

What is the most effective treatment for OSA ?

A

CPAP (Continuous Positive Airways Pressure)

722
Q

What is the most effective treatment for OSA ?

A

CPAP (Continuous Positive Airways Pressure)

723
Q

What are the recommended Drug Therapy in Patients with Bipolar Disorder ?

A
  • Lithium
  • Valproic acid (Depakene).
  • Carbamazepine (Tegretol):
    in patients with structural central nervous system disease or renal disease.
724
Q

What is the cause of presence of subcutaneous emphysema after an ERCP ?

A

It is a rare but well-recognized complication. The most common cause is duodenal perforation which results in retroperitoneal collection of free air.

725
Q

How to investigate for painless hematuria ?

A
  • computed tomography intravenous pyelogram, and
  • urine cytology, and
  • urine microscopy , an d
  • culture and blood tests (full blood examination, renal function and,
  • in men, prostate-specific antigen
726
Q

What is the defintion of postoperative adynamic ileus or paralytic ileus ?

A

Ileus that persists for more than 3 days following surgery is termed postoperative adynamic ileus or paralytic ileus.

727
Q

What is the test that has the best specificity and sensitivity for differentiating between postoperative ileus and other conditions ?

A

CT scanning with Gastrografin.

728
Q

In a plain radiograph,how long does it take a contrast medium in patients with paralytic ileus to reach the cecum?

A

On plain abdominal radiographs, ileus appears as copious gas dilatation of the small intestine and colon.
With enteroclysis, the contrast medium in patients with paralytic ileus should reach the cecum within 4 hours; if the contrast medium remains stationary for longer than 4 hours, mechanical obstruction is suggested.

729
Q

How to manage Postoperative Ileus ?

A
  • Management of ileus starts with correction of underlying medical condition, electrolyte abnormalities, and acid base abnormalities.
  • Most cases of postoperative ileus resolve with watchful waiting and supportive treatment.
  • Intravenous hydration.
  • For patients with vomiting and distention, use of a nasogastric tube provides symptomatic relief.
  • Discontinue medications that produce ileus (eg, opiates).The use of postoperative narcotics can be diminished by supplementation with nonsteroidal anti-inflammatory drugs (NSAIDs.
  • advisable to delay oral feeding until ileus resolves clinically.
  • Chewing gum may constitute a form of sham feeding that stimulates gastrointestinal motility
730
Q

What is the Diagnostic work-up for localised CRC (colorectal cancer ) ?

A

Local assessment :
- Complete colonoscopy.

Imaging work-up:
- CT scan: Lung, Abdominal , and Pelvic with IV contrast.
- CT colonography (when complete colonoscopy is not feasible).
- MRI abdominal (to clarify ambiguous lesions or define pT4b).

Laboratory work-up:
- Complete blood count.
- Coagulation.
- Liver function panel.
- Kidney function panel.
- Albumin.
- CEA.

731
Q

What is the Diagnostic work-up for localised CRC (colorectal cancer ) ?

A

Local assessment :
- Complete colonoscopy.

Imaging work-up:
- CT scan: Lung, Abdominal , and Pelvic.
- CT colonography (when complete colonoscopy is not feasible).
- MRI abdominal (to clarify ambiguous lesions or define pT4b).

Laboratory work-up:
- Complete blood count.
- Coagulation.
- Liver function panel.
- Kidney function panel.
- Albumin.
- CEA.

732
Q

What is the treatment of hyperplastic or adenomatous polyps and non-invasive (pTis, i.e. intra- epithelial or intramucosal) adenocarcinomas ?

A

Endoscopic resection.

733
Q

What is the management of locally infiltrative colon cancers ?

A

Wide resection of the involved bowel segment and its lymphatic drainage. The extent of the colonic resection is determined by the blood supply and distribution of regional lymph nodes. The resection should include a segment of colon of at least 5 cm on either side of the tumour.

734
Q

What is the diagnostic algorithm for localised colon cancer ?

A

Positive screening test / suspicious test –> Colonoscopy

  • No findings : refer to screening program.
  • Hyperplastic polyp –> remove polyp –> refer to screening program.
  • Adenoma –> remove adenoma –> repeat colonoscopy within 1 to 2 years.
  • Adenocarcinoma –> pT1 without risk factors –> remove the adenocarcinoma –> repeat colonoscopy within 1 to 2 years. .
  • Adenocarcinoma –> pT1 with risk factors or >pT1 –> presurgical diagnostic work up.
735
Q

Is Anti-D recommended after removal of molar pregnancy ?

A

Anti‐D prophylaxis is recommended following removal of a molar pregnancy.

736
Q

What is the Classification Criteria for traumatic brain injury severity based on Glasgow Coma Scale, Loss of Consciousness, Post-traumatic Amnesia and structural imaging?

A

Mild:
- GCS: 13-15
- LOC: <20 min to 1 hour
- Structural imaging: Definition Dependent.

Moderate:
- GCS: 9-12
- LOC: 1 hour - 24 hours
- Structural imaging: Normal or Abnormal.

Severe:
- GCS: 3-8
- LOC: > 24 hours
- Structural imaging: Normal or Abnormal.

737
Q

How many doses of varicella vaccine should be given ?

A

The varicella vaccine is given in two doses.
- A child should have the first shot at ages 12-18 months. The second shot should be given at ages 4-6 years.

  • Older children and adults should have two shots, with four to eight weeks between the first and second shot.
738
Q

What is the most common opportunistic infection in patients with HIV ?

A

Pneumocystis jirovecii (formerly Pneumocystis carinii).

739
Q

What are the Guidelines for Management of Incidentally Detected Pulmonary Nodules in Adults ?

A

In low risk patients + single nodule:
- <6 mm : No routine follow-up.

  • 6–8 mm: CT at 6–12 months, then consider CT at 18–24 months.
  • > 8 mm: Consider CT at 3 months, PET/CT, or tissue sampling.

In high risk patients + single nodule:
- < 6 mm: Optional CT at 12 months.

  • 6 -8 mm: CT at 6–12 months, then CT at 18–24 months.
  • > 8 mm: Consider CT at 3 months, PET/CT, or tissue sampling.

In low risk patients + multiple nodules:

  • <6 mm : No routine follow-up.
  • 6 - 8 mm: CT at 3–6 months, then consider CT at 18–24 months.
  • > 8 mm: CT at 3–6 months, then
    consider CT at 18–24 months.

In high risk patients + multiple nodules:

  • < 6 mm: Optional CT at 12 months.
  • 6-8 mm: CT at 3–6 months, then at 18–24 months.
  • > 8 mm: CT at 3–6 months, then at 18–24 months.
740
Q

What are the Guidelines for Management of Incidentally Detected Pulmonary Nodules in Adults ?

A

In low risk patients + single nodule:
- <6 mm : No routine follow-up.

  • 6–8 mm: CT at 6–12 months, then consider CT at 18–24 months.
  • > 8 mm: Consider CT at 3 months, PET/CT, or tissue sampling.

In high risk patients + single nodule:
- < 6 mm: Optional CT at 12 months.

  • 6 -8 mm: CT at 6–12 months, then CT at 18–24 months.
  • > 8 mm: Consider CT at 3 months, PET/CT, or tissue sampling.

In high risk patients + single nodule:

741
Q

Which SSRIs that has the most side effects ?

A

Citalopram appears to be the best-tolerated SSRI, followed by fluoxetine, sertraline, paroxetine, and fluvoxamine. The latter 2 drugs are associated with the most side effects and the highest discontinuation rates because of side effects in clinical trials.

742
Q

Which type of malignancy is associated with Hashimoto’s thyroiditis ?

A

malignant lymphoma of the thyroid.

743
Q

Most common site of fibroid in uterus ?

A

Intramural fibroids are the most common type of fibroid. These types appear within the muscular wall of the uterus.

744
Q

congenital heart disease associated with infective endocarditis ?

A
  1. ventricular septal defect (31 %).
  2. tetralogy of Fallot (19 %).
  3. atrioventricular septal defect
745
Q

What are the causes of functional hyposplenism ?

A
  • Sickle-cell anemia
  • alcoholic liver disease
  • celiac disease
  • bone marrow transplantation
  • inflammatory bowel disease.
746
Q

Best procedure to diagnose tracheomalacia ?

A

Three-phase dynamic bronchoscopy.

747
Q

How to confirm ovulation ?

A
  • Urine test kits to measure LH levels
  • Transvaginal ultrasound
  • Endometrial biopsy.
  • Blood tests to measure hormone levels.
  • Basal body temperature (BBT) chart.
748
Q

Gonorrhea swab site ?

A

Male : urethra
Female: cervix

749
Q

What are the complications of cystic fibrosis ?

A
  • Allergic bronchopulmonary aspergillosis (ABPA): it’s an allergic reaction in the lungs to the fungus Aspergillus.
  • Bronchiectasis: a widening of the airways in the lungs caused by chronic inflammation or obstruction of the airways.
750
Q

What is sandpaper rash ?

A

The strep bacteria make a toxin (poison) that causes a bright red, bumpy rash. The rash spreads over most of the body and is what gives scarlet fever its name. It often looks like a bad sunburn with fine bumps that may feel rough like sandpaper, and it can itch.

(Streptococcus pyogenes, which is also called group A Streptococcus or group A strep, cause scarlet fever).

751
Q

SLE confirmatory tests ?

A

The antinuclear antibody (ANA) test is the most sensitive test for SLE and is therefore the best screening assay for ruling out its presence.

Anti-native (N)-DNA and anti-Sm (Smith antigen) tests are highly specific for SLE and have strong confirmatory powers, even in a patient unlikely to have the disease.

752
Q

What is the most common obstetric cause for DIC ?

A

The three most common causes of obstetrical DIC :
1. PPH
2. placental abruption.
3. preeclampsia.

753
Q

What is the :
- Gene defect
- Mode of inheritance
- Deficient enzyme
- Role of enzyme
- Effect of enzyme deficiency
- Age of onset
- Clinical features
- Diagnostics
- Treatment
of Hereditary fructose intolerance ?

A
  • Gene defect: Chromosome 9q
  • Mode of inheritance: AR
  • Deficient enzyme: Aldolase B
  • Role of enzyme: Converts fructose - 1- phosphate to glyceraldehyde and dihydroxyacetone phosphate.
  • Effect of enzyme deficiency:
    Accumulation of fructose -1- phosphate —>
    –> decrease in available phosphates –> inhibition of glycogenolysis and gluconeogenesis —> hypoglycemia.
  • Age of onset : When a child starts to consume food that contains sucrose.
  • Clinical features:
    Bloating, sweating, vomiting, failure to thrive, jaundice that can progress to cirrhosis, bleeding tendency, sever hypoglycemia (seizures, hypotonia , poor feeding, cyanosis, irritability), hepatomegaly, proximal tubular dysfunction —> renal failure.
  • Diagnostics :
  • Detection of reducing substances (fructose) in urine.
  • Definitive Dx: Enzymes assay in liver Bx and DNA testing for genetic defect.
  • Abnormal LFTs
  • Treatment :
    Life long adherence to a fructose-free, Sorbitol-free and Sucrose-free diet.
754
Q

What is the effect of MgSO4 on CTG ?

A

a small negative effect on FHR, variability, and accelerative pattern

755
Q

oxytocin effect on CTG ?

A

variable deceleration - transient bradycardia

756
Q

Why does amenorrhea cause osteoporosis?

A

Amenorrhea or “ oligomenorrhea “ (meaning a cycle length longer than 36 days or skipping flow for up to 6 months) causes bone loss because estrogen levels drop and remain low. Progesterone levels are also low therefore bone isn’t built as it should be.

757
Q

What is the PPD interpretation in patients with HIV infection?

A

CDC and the American Thoracic Society recommend that tuberculin reactions greater than or equal to 5 mm be considered positive for persons who are HIV-seropositive (regardless of Bacillus of Calmette and Guerin {BCG} vaccination status) and that such persons be considered for isoniazid prophylaxis

758
Q

Elevated IgE antibody indicate what ?

A

IgE antibodies are normally found in small amounts in the blood, but higher amounts can be a sign that the body overreacts to allergens. This can lead to an allergic reaction. IgE levels can also be high when the body is fighting off an infection from a parasite or with some immune system conditions.

759
Q

What is the best predictors of mortality in COPD patients ?

A

The patient’s age and baseline postbronchodilator FEV1 are the best predictors of mortality.

760
Q

what are the renal abnormalities associated with eclampsia ?

A

Eclampsia-associated renal abnormalities can include decreases in:
- glomerular filtration rate
- renal plasma flow
- uric acid clearance
- proteinuria.

761
Q

What is the the antidote for tPA ( Tissue plasminogen activator) in case of toxicity?

A

Aminocaproic acid.

762
Q

What is Acute colonic pseudo-obstruction (Ogilvie’s syndrome)?

Etiology ?
management ?

A

Definition: Acute colonic pseudo-obstruction (Ogilvie’s syndrome) is a disorder characterized by acute dilatation of the colon in the absence of an anatomic lesion that obstructs the flow of intestinal contents.

Etiology:
- in hospitalized or institutionalized patients in association with a severe illness or after surgery and in conjunction with a metabolic imbalance or administration of culprit medication
- the most common predisposing conditions were nonoperative trauma, infection, and cardiac disease.

Management:
- The goal is to decompress the colon to minimize risk of perforation and ischemia.
- Monitors + serial PEx + X-rays Q12-24 hours to evaluate colonic diameter.
- Initially: conservative IF:
* No significant abdominal pain.
* Diameter less then 12 cm.
* No signs of peritonitis.
- IF diameter is more than 12 cm or failed conservative Mx for24-48 hour:
* Neostigmine
* Colonoscopic decompression.
* Surgical decompression : If all above failed / have evidence of peritonitis or perforation.
* If pt not a candidate or surgery , Percutaneous colostomy can be used.

763
Q

common pelvic organ prolapse ?

A

cystocele ( bladder)

764
Q

Treatment of community acquired pneumonia ?

A

Either one of the following:

  1. Macrolides
    - Azithromycin
    - Clarithromycin (Biaxin)
    - Erythromycin
  2. Fluoroquinolones:
    - Gatifloxacin (Tequin)
    - Levofloxacin (Levaquin)
    - Moxifloxacin (Avelox)
  3. Doxycycline:
765
Q

Splenic injury grades ?

A
  • grade I
    subcapsular hematoma <10% of surface area
    parenchymal laceration <1 cm depth
    capsular tear
  • grade II
    subcapsular hematoma 10-50% of surface area
    intraparenchymal hematoma <5 cm
    parenchymal laceration 1-3 cm in depth
  • grade III
    subcapsular hematoma >50% of surface area
    ruptured subcapsular or intraparenchymal hematoma ≥5 cm
    parenchymal laceration >3 cm in depth
  • grade IV
    any injury in the presence of a splenic vascular injury* or active bleeding confined within splenic capsule
    parenchymal laceration involving segmental or hilar vessels producing >25% devascularisation.
  • grade V
    shattered spleen
    any injury in the presence of splenic vascular injury* with active bleeding extending beyond the spleen into the peritoneum.
766
Q

What are the medical treatment options of acute abnormal uterine bleeding ?

A

• IV conjugated equine estrogen: Primary choice for heavy bleeding (Once bleeding has slowed, patients can be transitioned to an oral taper using Premarin pills or more commonly COCs.)

• Combined oral contraceptives (OCs): Primarily for less severe bleeding.

• Oral progestins: Primarily for less severe bleeding and when estrogen is contraindicated.

•Tranexamic acid (TXA): is also an option for acute heavy menstrual bleeding (HMB).
**Therapeutic D&C should be reserved as a last resort for the rare patient who continues to have life-threatening bleeding despite high-dose of estrogen administration

767
Q

What are the options for medical management of chronic abnormal uterine bleeding?

A
  • Levonorgestrel-Releasing IUD (Mirena): First line treatment for heavy menstrual bleeding (HMB).
  • Combined oral contraceptives (OCs).
  • Depot medroxyprogesterone acetate.
  • Norethindrone acetate (progestins).
  • Tranexamic acid (TXA).
  • NSAID:
    • Mefenamic acid
    • Naproxen
    • Ibuprofen
    • Flurbiprofen
  • GnRH agonists
768
Q

What are the causes of abnormal uterine bleeding?

A

Structural causes PALM:
- Polyps
- Adenomyosis
- Liomyoma
- Malignancy & hyperplasia

Nonstructural causes COIEN:
- Coagulopathy
- Ovulatory
- Endometrial
- Iatrogenic
- Not yet classified

769
Q

Regarding Copper IUD & Levonorgestrol -Releasing intrauterine device :
- for how long they are used ?
- what are the side effects ?

A
  • Copper IUD:
    Approved for 10 continuous years.
    S/E: heavy menstrual, bleeding, and pain.
  • Levonorgestrel -Releasing intrauterine device :
    Approved for up to five years
    S/E: diminished menstrual bleeding and amenorrhea, and it’s used to treat heavy bleeding’s.
770
Q

What is DMPA injections ? (Contraceptives) and what are their side effects?

A

Depo Medroxyprogesterone acetate injections are progestin only contraceptives.

S/E :
1. Irregular bleeding (usually decreases with each injection).

  1. Bone Loss (When the injections are stopped, at least some and sometimes all of the bone that is lost is gained back.)

DOES NOT CAUSE WEIGHT GAIN.

771
Q

What is the first line contraceptive method that is recommended by the American Congress of Obstetricians and gynecologists ? Why ?

A

LARC METHODS: long-acting reversible contraception.

Because they’re:
- highly effective
- immediately reversible with a rapid return to fertility after removal.
- Very few medical contraindications to LARC exist.

The LARC methods include :
1. single-rod etonogestrel subdermal implant
2. the Copper T380A intrauterine device
3. levonorgestrel intrauterine systems (LNG-IUS).

772
Q

OCP and hypertension, when to use and when it’s contraindicated?

A

SBP < 140/90 mm Hg:
may use any hormonal contraceptive method.

  • SBP : 140-159 mm Hg or DBP: 90-99 mm Hg :
    OCP should not be used unless no other method is appropriate.

SBP =>160 mm Hg DBP => 100 mmHg or with vascular disease:
should not use combined hormonal contraceptives.

773
Q

Progestin are only given to women with an intact uterus to prevent ….. . ?

A

endometrial hyperplasia and cancer

774
Q

Name the medication’s that are used for emergency contraception

A
  • Levonorgestrel
  • Ulipristal acetate
  • Combined oral contraceptive pills.
775
Q

What is the mechanism of action of emergency contraceptives ( Levonorgestrel & Ulipristal acrtate) ?

A

Prevent ovulation

776
Q

What is the mechanism of action of copper IUD and levonorgestrel IUD ?

A

Inhibition of fertilization.

777
Q

What is the initial treatment of endometriosis related pain?

A

Estrogen plus progestin - containing agents.

778
Q

How much folic acid is recommended for pregnant women?

A
  • 600 micrograms of folic acid each day.
  • a daily prenatal vitamin with at least 400 micrograms starting at least 1 month before pregnancy and during the first 12 weeks of pregnancy is recommended.
  • History of a child with an NTD should take 4 milligrams (mg) of folic acid each day as a separate supplement at least 3 months before pregnancy and for the first 3 months of pregnancy.
779
Q

For how long it’s recommended to avoid pregnancy after receiving a live attenuated vaccine ?

A

It’s recommended to avoid pregnancy for one month, following each dose of a live attenuated vaccine .

780
Q

What is the recommendation regarding Tetanus, diphtheria & Pertussis vaccination in pre/peri/postnatally ?

A
  • it is routinely recommended during pregnancy.
  • If the pregnant woman has already received the full three-dose series, she can also receive a single dose during 27 to 36 weeks of gestation.
  • it is indicated in each pregnancy, even if consecutive pregnancies occur within 12 months.
  • if pregnant woman never took the vaccine before she should take the three dose series at time: 0, 4 weeks later and at 6 to 12 months after the initial does.
  • if the vaccine was never given during pregnancy, it should be administered immediately postpartum.
781
Q

Name the live viruses vaccines that are contraindicated during pregnancy.

A
  • Varicella-zoster
  • Measles, mumps, rubella (MMR)
  • Polio
  • Chickenpox
  • Yellow fever.
  • Intranasal Influenza
782
Q

Name the contraindicated vaccines during nursing and breast-feeding, and mentioned the reason.

A

Smallpox : will cause Vaccinia

Yellow fever : will cause meningoencephalitis.

783
Q

Which vaccinations are recommended to be administered to women post partum.

A
  • Measls, Mumps & Rubella : during hospital admission (before discharge)
  • Varicella: first dose in the hospital, and the second dose is giving 4 to 8 weeks later.
784
Q

What is the term for ectopic pregnancy that occurs with an intra-uterine pregnancy?

A

Heterotopic pregnancy

785
Q

What are the risk factors of tubal pregnancy?

A
  1. Risk of ectopic pregnancy:
    - history of one previous ectopic ->10%
    - a history of two or more previous ectopic) -> morethan 25%
  2. Abnormal fallopian tube anatomy.

3.Previous damage to the fallopian tubes, secondary to surgeries for a prior tubal pregnancy, for fertility restoration, or for sterilization.

  1. Prior sexually transmitted disease or other tubal infection.
  2. Women who become pregnant through the use of assisted reproductive technology(IVF).

6.Women with a history of infertility

7.Congenital fallopian tube anomalies

8.Other less significant risk factors include a history of cigarette smoking and age older than 35 years.

786
Q

Transvaginal Ultrasound (TVUS)

  • During endometrial cavity evaluation, an intrauterine gestational sac is usually visible between:
A

4.5 and 5 weeks.

787
Q

Transvaginal Ultrasound (TVUS):
During endometrial cavity evaluation, The yolk sac appears between :

A

5 and 6 weeks.

788
Q

Transvaginal Ultrasound (TVUS)
During endometrial cavity evaluation,
a fetal pole with cardiac activity is first detected at:

A

5.5 to 6 weeks

789
Q

What are the indications of medical management of ectopic pregnancy?

A

Methotrexate MTX is indicated if the following are present:

• Hemodynamic stability.
• Serum beta-human chorionic gonadotropin (hCG) concentration ≤5000 milli-international units/mL.
• No fetal cardiac activity detected on transvaginal ultrasound (TVUS).
• Ectopic mass size less than 4 cm
• Patients are willing and able to comply with post-treatment follow-up and have access to emergency medical services within a reasonable time frame in case of a ruptured.

790
Q

What are the absolute contraindications to methotrexate MTX therapy in ectopic pregnancy?

A

Absolute Contraindications to Methotrexate MTX therapy (ACOG):

• Intrauterine pregnancy.
• Evidence of immunodeficiency.
• Moderate to severe anemia, leukopenia, or thrombocytopenia.
• Sensitivity to methotrexate
• Active pulmonary disease.
• Active peptic ulcer disease.
• Clinically important hepatic dysfunction.
• Clinically important renal dysfunction.
• Breastfeeding.
• Ruptured ectopic pregnancy.
• Hemodynamically unstable patient.
• Inability to participate in follow-up.

791
Q

What are the relative contraindications of methotrexate therapy in ectopic pregnancy ?

A

Relative Contraindications to Methotrexate MTX therapy (ACOG):

• Embryonic cardiac activity detected by transvaginal ultrasonography
• High initial hCG concentration >5000 milli-international units/mL.
• Ectopic pregnancy greater than 4 cm in size as imaged by transvaginal ultrasonography
• Refusal to accept blood transfusion

792
Q

What is the protocol in methotrexate MTX treatment in ectopic pregnancy?

A
  1. Single dose regimen:
    - Methotrexate 50 mg/m2 IM on Day 1
    - B-HCG post treatment Day 4 and Day 7.
    - if decreased >15% , measure b-hcg weekly till normal.
    - If decreased < 15% , give Methotrexate again same dose and repeat b-hcg level
    - if still the same => Surgical Mx.
  2. Two-dose regimen:
    - MTX 50 mg/m2 IM on Day 1 + Day 4
    -if decreased >15% , measure b-hcg weekly till normal.
    - if decreased <15% , MTX 50 mg/m2 IM on day 7 and check b-hcg on day 11.
    - if still same , give a 4th dose MTX 50mg/m2 IM on day 11 and check b-hcg on day 14.
    - if still same surgical Tx.
  3. Fixed multiple dose regimen:
    - MTX 1mg/kg IM on Day1,3,5,7 & alternate with folinic acid on day:2,4,6,8.
  • measure bhcg on MTX days and continue till level os back to normal.
  • if b-hcg didn’t decrease after 4 doses => surgical Mx.

-if b-hcg decreases more than 15% discontinue MTX and check level Qweek.

793
Q

What are the risk factors of Hydatiform Mole ?

A
  • The main risk factors for HM are extremes of maternal age and a history of previous mole.
  • Prior molar pregnancy:
    • The risk for repeat molar pregnancy after the first mole is approximately 1 to 1.5 percent (approximately 10 to 15 times the risk for the general population).
    •The recurrence rate after two molar pregnancies has been reported to range from 11 to 25 percent.
  • Extremes of maternal age (≤15 and >35 years)
  • Asian and American Indian ancestry
794
Q

What is the clinical presentation of
Hydatiform moles ?

A

Common features:
- Vaginal bleeding
- Pelvic pressure or pain
- Enlarged uterus (greater than normal)
- Hyperemesis gravidarum

Less common or late features:
- Hyperthyroidism - Due to elevation of hCG >100,000 mIU/mL for several weeks. These patients may present with tachycardia, warm skin, and tremor. Laboratory evidence of hyperthyroidism is commonly detected in asymptomatic patients with HM.

  • Ovarian theca lutein cysts - Are a form of ovarian hyperstimulation resulting from high circulating levels of hCG and prolactin.
  • Preeclampsia <20 weeks of gestation
    Passage of hydropic vesicles from the vagina.
795
Q

How to diagnose hydatiform moles ?

A
  1. Markedly increased B-hcg level.
  2. If b-Hcg >100,000 ==> Transvaginal U/S.
  3. Definitive diagnosis: histological based upon uterine evacuation specimen.
796
Q

What are the findings in transvaginal ultrasound in complete mole and partial mole?

A

• Complete mole:
1. Absence of an embryo or fetus.
2. Absence of amniotic fluid.
3.Central heterogeneous mass with numerous discrete anechoic spaces - This has classically been described as a “snowstorm or Swiss cheese pattern”
4. Ovarian theca lutein cysts.

•Partial mole:
1.Diagnosed as a missed or incomplete abortion in 15 to 60%.
2.A fetus may be identified, may be viable, and is often growth restricted. 3.Amniotic fluid is present, but the volume may be reduced.
4. Placenta with one or more abnormal findings - Enlarged, cystic spaces (“Swiss cheese
pattern”) and/or increased echogenicity of chorionic villi.
5.Increased transverse diameter of the gestational sac.
6. Theca lutein cysts are usually absent.

797
Q

What is the management of hydatiform moles ?

A
  • Molar evacuation by suction curettage is usually the preferred treatment.
  • Hysterectomy is a reasonable alternative for patients who:
  1. have completed childbearing.
  2. particularly those with a known or presumptive complete mole and the following risk factors for gestational trophoblastic neoplasia:
    •Signs of trophoblastic proliferation (uterine size greater than gestational age, serum human chorionic gonadotropin [hCG] levels > 100,000 milli-international units/mL, ovarian theca lutein cysts >6 cm in diameter)
    • Age >40 years
798
Q

Post treatment management of hydatiform moles ?

A

POSTTREATMENT MANAGEMENT
- Anti-D immune globulin if Rh D-negative (in case of partial mole because it has fetal tissues).
- Initiate effective contraception.
- Review pathology report.
- Serum hCG levels: within 48 hours of evacuation, weekly until undetectable, then monthly for 6 months.

799
Q

Several factors predispose a patient to trophoblastic neoplasia following molar evacuation:

A
  1. complete moles have a 15 to 20% incidence of malignant sequelae, compared with 1 to 5% following partial moles.
  2. older maternal age(>40)
  3. B-hCG levels > 100,000 mlU/mL,
  4. uterine size that is large for gestational age
  5. theca-lutein cysts >6 cm.
  6. slow decline in B-hCG levels.
800
Q

What are the common sites of metastasis of gestational trophoblastic neoplasia?

A
  1. Pulmonary 80%.
  2. vagina 30%.
  3. CNS 10%
  4. Hepatic 10%
  5. Other
801
Q

What is the most common and the most aggressive histological type of gestation trophoblastic neoplasia GTN ?

A

Choriocarcinoma.

802
Q

What are the criteria for diagnosis of gestational trophoblastic neoplasia?

A
  1. Plateau of serum B-hcg level (-/+10> ) for four measurements during a period of 3 weeks or longer–days 1, 7, 14, 21.
  2. Rise of serum B-hcg level >10 % during three weekly consecutive measurements or longer, during a period of 2 weeks or more : days I, 7, 14.
  3. Serum B-hcglevel remains detectable for 6 months or more.
  4. Histological criteria for choniocancinoma
803
Q

What is the management of gestational trophoblastic neoplasia GTN ?

A

**Chemotherapy alone is usually the primary treatment:

  1. Single-agent chemotherapy for nonmetastatic or low-risk metastatic neoplasia:
    Monotherapy protocols with either methotrexate or actinomycin D.
  2. Combination chemotherapy is given for high-risk disease.
804
Q

Which type of contraception is preferred after remission of gestational trophoblastic neoplasia?

A

Estrogen- Progestin contraceptives.

805
Q

What are the indications for cervical cerclage in women with singleton pregnancies ?

A

History:
- history of one or more second trimester pregnancy losses related to painless, cervical dilation, and in the absence of labor or abruptio placentae.
- prior cerclage due to painless, cervical dilation in the second trimester.

PEx:
Painless, cervical dilation in the second trimester .

U/S finding with a history of prior preterm birth:

  • Shorts cervical length (less than 25 mm ) before 24 weeks of gestation {with a prior spontaneous preterm birth, at less than 34 weeks of gestation}.
806
Q

When should we begin transvaginal ultrasound for cervical length screening and how to approach according to the cervical length?

A

For women with a singleton pregnancy and a history of prior spontaneous preterm birth we begin TVUS cervical length screening at:
14 to 16 weeks of gestation,

and if her Cenvical length:
🔺>25mm: we perform serial examination.
🔺<25mm: Cervical cerclage placement before 24 weeks of gestation.

807
Q

What is the difference between chronic hypertension in pregnancy, gestational hypertension and preeclampsia?

A

🔺Chronic Hypertension in Pregnancy:
HTN before 20 week of gestation OR
persists longer than 12 weeks postpartum.

🔺Gestational Hypertension:
Pregnancy-induced hypertension with onset after 20 weeks’ gestation without proteinuria or end-organ dysfunction.

🔺Preeclampsia:
New onset of hypertension with proteinuria (=> 0.3 g or protein/creatinine ratio (=> 0.3 (mg/mg)) or end-organ dysfunction after 20 weeks of gestation.

808
Q

What is the criteria for the diagnosis of pre-eclampsia ?

A

🔺SBP >= 140mmHg or DBP >= 90 mmHg on at least 2 occasions at least 4 hours apart after 20 weeks of gestation in a previously normotensive patient.
+ 1 or more of :

  • Proteinuria >= 0.3 g in a 24-hour urine specimen or protein/creatinine ratio >=0.3 (mg/mg) (30 mg/mmol) in a random urine specimen or dipstick >=2+ if a quantitative measurement is unavailable.
  • Platelet count <100,000/microL
  • Serum creatinine > 1.1 mg/dL (97.2 micromol/L) or doubling of the creatinine concentration in the absence of other renal disease.
  • Liver transaminases at least twice the upper limit of the normal concentrations for the local laboratory
  • Pulmonary edema
  • New-onset and persistent headache not accounted for by alternative diagnoses and not responding to usual doses of analgesics.
  • Visual symptoms (eg. blurred vision, flashing lights or sparks, scotomata)
809
Q

What are the risk factors of preeclampsia?

A

1- Nulliparity
2- Multifetal gestations
3- Preeclampsia in a previous pregnancy.
4- Chronic hypertension
5- Pregestational diabetes
6- Gestational diabetes.
7- Thrombophilia
8- Systemic lupus erythematosus.
9- Prepregnancy body mass index greater than 30.
10- Antiphospholipid antibody syndrome.
11- Maternal age ≥35 years or <18 years.
12- Kidney disease.
13- Assisted reproductive technology (IVF).
14 - Obstructive sleep apnea.
15 - Obesity (BMI>30).
16 - Hydatidiform mole.
17 - Family history of preeclampsia.

810
Q

How to prevent preeclampsia?

A

High risk patients should receive a low dose of aspirin (81 mg /day )for preeclampsia prophylaxis initiated between 12 weeks and 28 weeks of gestation (optimally before 16 weeks of gestation )and continuing until delivery.

811
Q

What is the management of preeclampsia?

A

🔺 <37 weeks of gestation:
Expectant management + oral labetalol or nifedipine.

🔺≥ 37 weeks of gestation:
IOL + oral labetalol or nifedipine.

🔺 ≥ 34 weeks of gestation with pretem labor or PPROM:
IOL + oral labetalol or nifedipine.

812
Q

How to diagnose preeclampsia with severe features ?

A

The presence of one or more of the following indicates a diagnosis of preeclampsia with severe features:

  1. Severe blood pressure elevation (SBP ≥ 160 or DBP ≥ 110mmHg).
  2. Symptoms of CNS dysfunction.
  3. hepatic abnormality.
  4. thrombocytopenia <100,000 plt/microL
  5. Renal abnormality.
  6. Pulmonary edema.
813
Q

What is the management of preeclampsia with severe features?

A

🔺< 34 weeks of gestation:
Expectant management : admission+ corticosteroids+ magnesium sulfate (seizure prophylaxis) + IV labetalol.

🔺 ≥ 34 weeks of gestation:
Induction of labor after stabilizing the mother + magnesium sulfate (seizure prophylaxis) + IV labetalol.

814
Q

What is HELLP syndrome?
how to make the diagnosis ?
what is the clinical presentation?
and what is the management in pregnant women?

A

🔺 A life-threatening form of preeclampsia characterized by Hemolysis, Elevated Liver enzymes, and Low Platelets.

🔺 To make the diagnosis:
- Lactate dehydrogenase (LDH) elevated to 600 IU/L or more,
- Aspartate aminotransferase (AST) and alanine aminotransferase (ALT) elevated more than twice the upper limit of normal
-Platelets count less than 100,000

🔺Clinical presentation:
- Right upper quadrant pain and generalized malaise (90%)
- Nausea and vomiting (50%)

🔺Management:
Women with HELLP syndrome should be delivered regardless of their gestational age

815
Q

What is the management of eclampsia?

A
  • ABC
  • calling for help
  • prevention of maternal injury.
  • placement in lateral decubitus position.
  • prevention of aspiration
  • administration of oxygen
  • monitoring vital signs including oxygen saturation.
    -Anti-convulsive Therapy:
    • Magnesium sulfate IV 4-6 g loading dose over 20-30 minutes, followed by a maintenance dose of 1-2 g/hour.

• Diazepine and phenytoin are justified only in the context of antiepileptic treatment or when magnesium sulfate is contraindicated or unavailable.

  • Delivery once the mother is stabilized.
816
Q

🔺dose of magnesium sulfate in eclampsia?
🔺side effects?
🔺How to monitor ?
🔺what to do if toxicity occurs?
🔺 what are the contraindications?

A

Most eclamptic seizures are self-limited. Magnesium sulfate is not necessary to arrest the seizure but to prevent recurrent convulsions.)

🔺Dose: [IV] administration of a 4-6 g loading dose over 20-30 minutes, followed by a maintenance dose of 1-2 g/hour.

🔺Side effects: Deep tendon reflexes are lost, warmth and flushing, respiratory depression, and cardiac arrest.

🔺Monitoring: respiration status, tendon reflexes and measuring urine output (because magnesium sulfate is excreted almost exclusively in the urine).

🔺When toxicity occurs?
The infusion should be stopped emergency correction with calcium gluconate 10% solution, 10 mL IV over 3 minutes, along with furosemide intravenously to accelerate the rate of urinary excretion.

817
Q

What is the first line and second line IV treatment of hypertension in pregnancy?

A

First line treatment labetalol
Second line treatment hydralazine .

818
Q

What is the first line and second line oral treatment of hypertension in pregnancy?

A

First line: labetalol, 100 - 200 mg b.i.d.
Second line: Nifedipine 30 to 60 mg once daily.
Third line : Methyldopa 250 mg b.i.d. or TID .

819
Q

What is the clinical manifestation that follows preeclampsia?

A
  • Fetal growth restriction(IUGR).
    -Oligohydramnios.
    -Placental abruption.
    -Nonreassuring fetal status.
    -Increased risk of spontaneous or indicated preterm delivery.
820
Q

What is the most common complication of diabetes in pregnancy ?
And why patients with gestational diabetes are not at risk of having this complication?

A

Congenital malformation. Because elevated glucose values are associated with congenital anomalies.
Women, with true gestational diabetes are not at increased risk of having an infant with congenital malformation, because the onset of the disorder is after organogenesis, and they don’t experience diabetes - related vasculopathy, because of the short duration of the disorder.

821
Q

Women with GDM have a higher risk of developing which disorder.

A

Macrosomia, then hypertension, and preeclampsia.

822
Q

At which gestational age does nuchal translucency scan done to detect chromosomal abnormalities?

A

Between 10+3 and 13+ 6 gestational weeks.

823
Q

What is the recommended dose of folic acid supplementation in high-risk moderate risk, and low risk pregnant patients?

A

Low risk: 0.4 mg
Moderate risk : 1 mg
High risk: 4 mg.

824
Q

Congenital malformations other than NTDs:

We suggest a higher dose (1 mg per day) of periconceptional/first-trimester folic acid supplementation for women with a history of one of the following malformations in themselves or their partner, a prior offspring, or a first- or second-degree relative:

A

•Cleft lip/palate
•Congenital heart defects
•Limb reduction defects
•Urinary tract defects
•Congenital hydrocephalus

825
Q

At what gestation age, we start diabetes screening in a pregnant woman

A

At or beyond 24 weeks of gestation.

826
Q

How to approach a patient with the following antenatal ultrasound finding:
Reversed end diastolic blood flow?

A
  • If the Patient is less than 32 weeks : expectant management until 32 weeks, then deliver immediately.
  • if the patient is 32 weeks or more deliver immediately .
827
Q

What are the five parameters of biophysical profile?

A
  1. body movements
  2. muscle tone
  3. breathing movements
  4. amniotic fluid
  5. heartbeat.

Each of these 5 areas is given a score of either 0 (abnormal) or 2 (normal).

828
Q

A prophylactic dose of 300 µg of anti-D immune globulin (Rh -ve mother with a Rh +ve baby) can prevent Rh D alloimmunization after exposure to up to:

A

30 mL of Rh D-positive fetal whole blood or 15 mL of fetal RBCs

829
Q

Which finding in urine analysis will confirm the diagnosis of hyperemesis gravidarum,

A

Urine ketones .

830
Q

What are the indications of surgical intervention in adnexal cyst in pregnancy?

A
  1. Symptomatic masses (acute abdomen).
  2. If suspected malignancy based on:
  • US finding suggest malignancy:
    • cyst greater than 10cm.
    • papillary or solid component.
    • irregularity.
    • presence of ascitis
    • high color doppler flow
  • serum marker testing. Or both.
831
Q

How to manage a newborn who has a hepatitis B positive mother?

A
  • hepatitis B vaccine
  • immunoglobulin

Within 12 hours of delivery.

832
Q

Group B streptococcus screening in pregnancy?

A

Routine antepartum GBS vagina and rectal cultures on all pregnant women at 35 to 37 weeks

833
Q

Down syndrome prenatal screening: ?

Down syndrome definitive prenatal diagnosis:?

A

Down syndrome prenatal screening:
- Combined test “US determination of nuchal translucency NT + determination of biochemical markers associated with aneuploidy.

Down syndrome definitive prenatal diagnosis:
- Chorionic villi sampling (CVS) or Amniocentesis

834
Q

Went to screen for Group B streptococcus in pregnant women ?

A

“Routine antepartum GBS vagina and rectal cultures on all pregnant women at 35 to 37 weeks.”

835
Q

At Which gestational age does the following procedures should be performed in ?
- Chorionic Villi sampling.
- Amniocentesis.

A
  • Chorionic Villi sampling: first trimester ( 10-13 weeks)
  • Amniocentesis : second trimester (ACOG: 15-20 weeks) & (Uptodate : 15-17+6).
836
Q

The type & frequency of pregnancy related conditions that triggered DIC are :

A
  • Placental abruption (37%)
  • Postpartum hemorrhage (PPH) (29%)
  • Preeclampsia/eclampsia/HELLP syndrome (14%)
  • Acute fatty liver (8%)
  • Amniotic fluid embolism(6%)
    Pregnancy-related sepsis (6%)
837
Q

Tocolysis choice in preterm labor?

A

Tocolysis choice in Preterm Labor:
🔺First-line:
• Indomethacin (24-<32 weeks)
• Nifedipine (32-34 weeks or women who have a contraindication to indomethacin)
🔺Second-line: if the first-line drug does not inhibit contractions, we discontinue it and begin therapy with another agent.
• Nifedipine (24-32 weeks)
• Terbutaline (32-34 or for those who received nifedipine as a first-line agent at 24 to 32 weeks)